Thursday, September 9, 2010

'Space' Warfare XII: Surface Warfare


Ask, and ye shall (sometimes) receive: A reader emailed asking me to discuss future ground war. This I will take a bit more broadly as warfare fought on habitable, shirtsleeves planets, including sea and air operations. Warfare on non-habitable planets is an ambiguous case, with features of boarding operations in space.

I intended to deal first with the space context. But y'all want grunts, preferably in power armor, supported by coolific armored vehicles and aircraft, with subs and trimaran assault cruisers out to sea. Which brings us to something that has not been tested yet. What happens when post industrial forces fight each other?

We don't know, but we have seen this movie before, in flickery black and white a hundred years ago. Industrial age Western armies had shown how well they could scythe down waves of natives, usually. The general prognosis was that 1900-modern weapons were so accurate and effective that when turned on each other they would pretty much wipe each other out, and quickly.

The Next War would be a come-as-you-are war, settled in months if not weeks by whoever ran out of arms and ammo first, if it weren't won a week earlier by strategy and tactical execution. 1870 was the prelude; 1861-65 merely an example of a semi-modern war fought entirely by blundering amateurs.

It did not work out that way in 1914, so I hesitate to say it would work out that way now, or in 2114. What might happen, in fact, is broadly what happened in 1914: Everyone goes to ground.

The general principle of future surface war, it seems to me, is that if you are caught out in the open you are headed for the celestial choir. This goes for guerillas, it goes for power armor troops, it goes for laser armed tanks, trimaran cruisers, aircraft, and spacecraft in low orbit. Give precision weapons a clear target and they will take it out.

Thirty meter mecha, sad to say, make for very clear targets.

Reconnaissance robotics, on the other hand, will be hard to take out. They can be very small and stealthy, making the Predator look like a B-36. So you should have plenty of scouts, including a robotic fly on the wall of the other side's headquarters. Your intel problem is noise - the more raw intel, the more noise. Any AI good enough to cut through it is an intelligence officer, not a piece of equipment.

A tank backed into the underbrush is still effective, because it is hard to find, and you may only find it when it opens up on you. A tank on the move has a target painted on it. This, I think, is the real advantage of power armor troops: Compared to tanks they are stealthy, and can slip through environments where a tank would draw attention and fire.

I expect power armor to be relatively light. At minimum you want enough to stop small arms fire, shrapnel, and the like. The maximum of useful armor is reached when a hit would kill you anyway, like getting hit by the equivalent of a truck. Against lasers this may mean the point at which you cook inside your armor, not good.

Future war may well be 'slow,' because the mobility of power armor troops is essentially foot mobility, with enhancements like powered roller skates. Mobility is limited behind the front as well, because truck convoys will be conspicuous targets even hundreds of km behind the lines. Logistics too will have to be stealthy.

It is easier to have a Ho Chi Minh Trail in the jungle, so one thing timeless will be the supreme importance of ground and the physical ecosystem. This of course gets interesting on habitable planets other than Earth.

Some kinds of fortifications might remain valid, basically because dirt absorbs a lot of damage points. Yes, there are bunker busters and Thor bolts, but the point is that such big powerful weapons are costly to deploy, carried by vulnerable platforms, and can be engaged by defensive fire. This could be the saving of large naval surface combatants, hard to sink except by massive attack that overwhelms their defenses.

The one way to achieve rapid, heavy movement, whether logistic or an actual assault, is to ramp up the noise level so high that the enemy's sensors are saturated, and nothing (you hope) is in 'plain sight.' If you are right you get blitzkrieg; if wrong you get the Somme.

Large scale surface war may thus have an alternating rhythm - weeks or months of stalking, skulking, and skirmishes along the front, interrupted by episodes of sheer rock & roll, perhaps to cover the fast movement of a truck convoy up to the front, where it will disperse itself and go to ground.

All of this takes place, or doesn't, against the background of nuclear weapons. The constraints on mobility in 'conventional' warfare could make it indecisive enough for the great powers to engage in it without risking a nuclear exchange. As in the 18th century they would be fighting for provinces, not national survival.


Now for the space context. If suitable planets are limited - say, Earth and terraformed Mars or Venus - politically balkanized planets are to be expected, unless you go mid 20th century retro and have the American Empire a Federation. Certainly on Earth itself you can plausibly expect Great Powers, with great power militaries.

In a few-worlds setting, space itself will be off in the background. India is not going to get in a major tussle with Olympus Mons; both have bigger problems much closer to home. And India and China are not going to take their arguments to the asteroid belt, at least not in a big way, because money spent on deep space forces comes out of much more critical surface, air, and Earth orbital forces.

This can have advantages for space-centric settings, because you can let the major Earth powers stalemate each other, keeping them off the deep space chessboard.

In the classic operatic setting of many colony worlds, it could be a different matter. Uniform planets are rightly bashed, and I've bashed them myself. But in such a setting I think politically unified planets will be common, perhaps the norm. In the colonization era everyone can have their own planet, and later on, even if local fissures develop - and they will - any planet that can present a united front enjoys a huge advantage in interstellar power politics.

Or putting it another way, any planet that cannot present a united front is at a huge disadvantage, drawing plot complications like flies. And here we are.


I could make the many and salient arguments for peace, but I know they would fall on deaf ears, so we'll go straight to comments.


The image comes from this futurist blog.

828 comments:

1 – 200 of 828   Newer›   Newest»
Unknown said...

Reproducing a comment I made on sfconsim-l several weeks ago:

It occurs to me that advanced man carried weapons for use in war may become a largely obsolete field within our lifetimes.

Instead, I'd expect within the next 10-20 years to see more and more small sized drones taking over the battlefield, anything from rat sized up sensors + bomb pack on up. I'd expect these to be either semi-autonomous or controlled from the rear. Thus there'd be no need for man carried arms, because there wouldn't be any men there.

In the (few?) instances where forward control or guidance was warranted, I'd expect the controllers to be either in forward command posts or vehicles. Again, I'd expect little need for personal assault weapons, at most a side arm would be appropriate in these instances.

Special forces might be one of the few exceptions. However, depending on the mission it's possible that even special forces would be accompanied by a number of drones. I'd expect them to have a wearable set of drone controls, and again operate slightly behind the armed drones.

I note that today even special forces frequently guides in air, missile, or artillery as one of their roles. I can only see the trend of men operating as guidance for or controllers of remote weapons systems accelerating.

So, instead of considering the design of a man carried assault weapon, it might be germane to consider the design of a small drone integrated laser assault weapon.

I'd envision, for instance, a convey taking a few people out to meet community leaders to be a few core vehicles accompanied by a small cloud of a few dozen flying, skittering, and crawling drones; from very small sensor only platforms through something perhaps dog sized carrying a single weapons system to horse or mule sized heavy weapons platforms. By the time an enemy reaches the central vehicle(s) they've have to have
passed by or through several of these drones. Don't forget that the vehicles are likely to be armed, too.

Even in very rough terrain where the people would be forced to dismount and proceed on foot, they might simply wear backpacks interfacing them to the drone's comm-net and acting as a relay to the rear echelon drone controllers. Still, the majority of the offensive punch might be drone supplied.

I note that there's already considerable work on armed and unarmed robotic military ground systems, e.g. google "robot mule" or "Special Weapons Observation Reconnaissance Detecting System"

Or, of course, I could be completely wrong. :-)

-- Pat

Byron said...

One interesting possible twist is to have the space war be backdropped by a larger surface war. Again, I'll use DOTB as an example. It was started by the US invasion, but the war quickly spilled over into India and the US vs. China on Earth. The space war was sort of a sideshow for everyone but Luna. That's how they won. The US could only devote a little bit of resources, while practically their whole budget went to the Navy. This allows a much weaker colony to take on a major Earth nation and survive.

adam_grif said...

Powered armor is plausible, whereas mecha are not. Since these are the same things at different scales, it's necessary to decide where one ends and the other begins.

General rule of thumb is that it stops being powered armor if it does not fit through typical doorways. So this limits them at about 2ish meters high.

Beyond this threshold, PA can no-longer serve in the same battlefield niche as infantry (which they have an edge over), and are forced to engage AFV's, which will be mounting 20mm cannons and up. When they reach 4 meters and up, they start becoming targets for significantly larger tank main guns, which they most certainly will not be able to armor against.

As a complement to PA, expect to see remote piloted, humanoid robots. If PA are plausible, then they are too, since they are essentially PA minus the squishy human pilot (so, more internal volume, no funerals when they get destroyed etc).

Unknown said...

i have to say the general outline presented by the blog post and my fellow commentors is one that i agree with as the most likely one to evolve from current trends. I would like to recommend the book the next hundred years by george friedman as having a number of interesting ideas about the trends in precision weaponry, drones and powered armour though he come to a rather different conclusion about the scope of military operations. his belief seems to be that precision weapons married with hypersonic scramjets and drones will lead to truly globalised warefare with no rear areas. he also presents with out going into much detail a vision of powered armour as nearing the crosscountry mobility of mechanized forces so landbased combat is not largely static. all in all it is an interesting look at near future planetbased combat with many of the same assumptions in regards to technical trends but with a rather different conclusion. if this is not going on to long one book i think of when i hear about large naval surface combatants with massive defences and hardening to allow their continued use would be firelance by david mace. it is a rather dated coldwar technothriller that is also one of the more depressing pieces of military fiction i have ever read, but it does outline a fairly belivable and interesting scenario that justifies a large and essentially unstealthy large surface combatant as a viable combat platform. on the other hand i have to say i believe the technological trends will lean towards a continuation of current naval design with its emphasis on small and realtivly cheap surface combatants with a greater emphasis on stealth and active defence such as decoys and PD. also they will probably move to being sumbmersible if not completely submarine.

Citizen Joe said...

In the future, armour will be soft... and squishy. Because precision weapons will be so deadly at range, defenses will be social and involve mixing military with civilians or hostages so that those ranged weapons will not be used due to the collateral damage. That will continue to be the case until a 'glitch' suddenly appears in all the robotic weapons and they just shoot through the hostages while the general point fingers at the arms manufacturers.

Rick said...

Welcome to a new commenter!

In terms of weapon reach there will be no rear areas, but I think that is exactly what will make logistics so difficult.

One place I might differ from Friedman: Paratroops delivered by hypersonic scramjets seem like easy targets for theater defense missiles. Generally, big transport planes are extremely vulnerable. Air transport for troops needs to be small and stay close to ground cover.

If soldiers are controlling battlefield weapons rather than holding them, small arms quickly take on a semi-ceremonial role.

Anonymous said...

Ok, let's see; space war between Earth and an Earth-like world (natural or terraformed), and between Earth and a non-Earth-like world (I think we already did colony vs. colony). So, the conditions would be:
1. Earth able to field a couple dozen command/bombardment/support mobile space bases, a hundred regiments of 1200 troopers each, hundreds of aerospace fighter planes, and scores of transport shuttles, and scads of tactical orbital spacewarships against:
a) an Earth-like rebellious colony world presenting a united front with roughly the same capabilities as the invaders from Earth.
b) an Earth-like colony world with many compeating colonial states, some at war with Earth, some neutral, and some allied with Earth. Each are roughly on par with any one nation on Earth, but without the depth of resources of the invaders.
c)a non-Earth-like world with united colonies, roughly with the same capabilities of the Earth forces.
d) a non-Earth-like world with seperate colonies, still on par with any Earth nation's state or provence.
e) any of the above that DON'T have a rough parity with the Earth forces.
2.A united Earth that isn't able to field more than one or two mobile space platforms, only a dozen regiments, and a few tactical orbital warships, against:
a throu e from above.
3. A single Earth nation (or a few), send an invasion force to a colony world. Same as a thou e above.
As I see it, things could go from colony gets stomped quickly, to colony wins outright, to a draw.

pspinler: I see a draw back with your sceinario: snipers and electronic warfare. Drones need operators, so you need a comm link to make it work; disrupt the comm link and you disrupt the drone. Snipers operate from as far away as a mile, so to cover that range would require a huge number of drones just to secure against these snipers. Shoot the local operator and the drones he was controling will be useless until a new operator can take control of them. It may be long enough for a successful attack on the enemy unit.

Rick: just thinking; if those underground/underwater bunkers/forts weren't known to the invading forces before they got there, then they would have to be discovered and neutralized...and that might be impossible before the war is won or lost.
Also; tanks may not be used in urban warfare, but that doesn't mean that they will become obsolete any time soon; in real life, several nations are developing point-defence systems for tanks and should be fielded within a decade on the outside.

One last thought: except in cases of young or underdeveloped colonies, any serious conflict between Earth and a colony world will favor the colony. The colony has a world-sized logistics train and the Earth has an astronomical sized logistics train. Also, the colony can build artilliry, planes, ships, subs, and tanks on-site; Earth has to ship everything across the sky to fight the war. It isn't impossible for Earth to win, but it seems to be much more difficult than people seem to think it will be. Of course, it all depends on the ratio of forces and how easy it is to resupply those forces. If the Earth forces can't adapt to the local enviornment, they it gets even tougher.

Ferrell

Thucydides said...

I suspect that as drones become more prominent as weapons platforms, they will also have to become smaller and more autonomous. This is the idea of "fire ant warfare".

The only thing even more frightening than that would be to give the weapons the ability to live off the land and reproduce.

There would be two levels of weapons systems; delivery systems to bring the "fire ant" weapons to battle, and the "fire ants" themselves. Delivery systems could range from familiar (submarines, rockets or artillery projectiles) to new (hypersonic cruise missiles, space delivery) or the simple stratagems of bringing them in by suitcase or other underhanded means. Personal protection will resemble vaccination rather than the gunfight at the OK Corral.

The more interesting stories will be in the intermediate period when people will still be able to take personal action, either by themselves or as the operators of drone devices. The other sort of story would have to involve people creating weird and illogical stratagems to outwit drone weapons (deciding to attack a certain target based on the draw of a card or the roll of a dice?)

Byron said...

I think that ECM argues against drones in many ways, far more than in orbit. I've read that the age of mass industrial war is over. I'm not sure, but for what we're dealing with today, drones are not a great answer. The decision to shoot and what is the most important part of what goes on, and people will have to make that decision. Drones can be jammed, but soldiers can't.
In all honesty, though, I don't really see any big land battles in the colonies, except for factional fighting. This is for two reasons:
1. The colonies have every incentive to spend on their navy if trying to defend against off-planet powers. If the navy works, then the army doesn't have to risk it, and nobody can get at me without going through the navy. That's if you don't worry about power projection. I can actually see two different land forces. The Army is responsible for defending the planet, while the Marines do power projection.
2. Landing troops requires orbital superiority. To put people on the ground, I am going to have to rule orbit to the point at which nobody threatens me. That includes surface-to-orbit weapons. It's going to be a lot easier to take out a dropship than it is to take out a warship. If I have enough superiority to land troops, I also have a fleet in low orbit, which can enforce my demands with lasers and kinetics, like in Footfall. The troops will be there for occupation, not for capturing the area.

jnutley said...

The future of land warfare is Transformers! ;)

You want to evade detection from orbit and from aerial cameras? Look like an ordinary local car. Pack a big gun, one or more swarms of small special purpose bots, some arms to break or build barricades, as much armor as you can bolt on inconspicuously, and enough smarts to complete your mission even if the comm is unreliable. Station them in garages near but outside the borders of your potential trouble spots. Defcom goes up, deploy them to or over the borders.

Leverage your car building tech to cut costs and make hundreds of them.

Milo said...

The problem with going to ground is how are you supplying your troops? Even if your soldiers are well-camouflaged in the Venusian rainforests, your factories and supply trains are going to be more visible. I'll just target those instead.

For an effective campaign, you need to either be able to live off the land by hunting-gathering, or you must have the support of the local citizens who are willing to provide you with supplies (thus strongly decentralizing your supply chain - if I napalm-bomb a farm on suspicion that they're helping you, you'll just go to a different farm, after filing an accusation of war crimes against me). And ammo is harder to come by in civilian villages than food, although this alone could justify using lasers - they can be recharged in any ordinary electrical outlet.

Guerillas are also generally poor at holding territory. They're good at wars of attrition and killing lost of invading troops until they've had enough and up and leave, but they're bad at keeping the enemy from simply marching past you and grabbing your cities. (Case in point: the US army is still having some trouble with Iraqi terrorists, but that didn't stop them from entering Baghdad and toppling the government.)

This is not at all like World War 1, where soldiers in trenches were actually capable of holding a line.


"It is easier to have a Ho Chi Minh Trail in the jungle, so one thing timeless will be the supreme importance of ground and the physical ecosystem. This of course gets interesting on habitable planets other than Earth."

Heh. Dedicating a lot of resources to grooming healthy ecosystems sprawling across the planet... so you have somewhere to hide in when the bombs fall. The treehuggers are gonna like that one! (Not.)


"In the colonization era everyone can have their own planet, and later on, even if local fissures develop - and they will - any planet that can present a united front enjoys a huge advantage in interstellar power politics."

In a setting where the majority of politically relevant governments have a full planet, a more Balkanized world might come over as something like Somalia today - a place full of small-scale warlords individually too weak to pose a meaningful threat to the big boys, and probably too busy fighting (or glaring at) each other to bother trying in the first place.

Of course, as Somalia showed, even warlords are dangerous if they decide to gang up on the foreigners. As the Bedouin proverb goes: "I against my brother. I and my brother against my cousin. I and my brother and my cousin against the world.".

Milo said...

adam_grif:

"As a complement to PA, expect to see remote piloted, humanoid robots."

You might be able to make humanoid robots, but why would you? Robots don't have the ingenuity to take full advantage of opposable thumbs or bipedal agility.

Even if they're completely remote controlled - that is, all movements directed by a human, rather than just a human saying "go this way and start shooting" - it'll need to have a pretty impressive control system to make that work comfortably. Having to enter individual servo movements is a non-starter, so you would need a full-body harness that records all the operator's movements and transmits them faithfully. Even then, unless you also have very good holograms / virtual reality, people are going to fumble trying to interact with objects that they're only seeing on a screen. Oh, and what if the robot gets knocked over by the enemy? Should the harness inflict the blow to the operator so he can react properly? And what if the enemy jams your communications? Close-range laser communications can't be jammed, but that exposes your operator to risk.



Ferrell:

"Snipers operate from as far away as a mile,"

Snipers can only hit soldiers on the move or in hastily-set tent camps. A sniper can't hit someone in a fort - and you can make a fort anywhere provided you have the time to shovel dirt.


"One last thought: except in cases of young or underdeveloped colonies, any serious conflict between Earth and a colony world will favor the colony."

Planetary assault favors the defender, as in every other case of attacking fortified positions in history. This is a conclusion we've all come to long ago.



Thucydides:

"I suspect that as drones become more prominent as weapons platforms, they will also have to become smaller and more autonomous."

I wonder about that. Generally miniaturization is expensive - often, small products are less effective, pound-for-pound, than larger ones. Even when people had the technology to build corvettes, they still used ships-of-the-line.

Small drones will be used for special operations, but I don't think gun-toting ants are going to be the mainstay of any army anytime soon.


"The only thing even more frightening than that would be to give the weapons the ability to live off the land and reproduce."

Wow, and here I thought minefields were a problem when you lose control of them!

Yeah, umm, let's not do this. Please.

Milo said...

Byron:

"I can actually see two different land forces. The Army is responsible for defending the planet, while the Marines do power projection."

The question is what differences in training and equipment do those two branches have? The most important thing is being prepared to fight in different gravities and to rapidly adjust to reentry from zero-gravity into a hostile territory, which I'll grant you is no easy thing. Still I'd rather train all my soldiers for that if I can - that way I can deploy them wherever I need to rather than having a bunch of people who are stuck at home. Force projection has more complicated logistics too, but that's mostly the domain of the navy rather than the surface grunts to get supplies to the right planet.

Also, given the fact that you will only ever have a land war on your home planet if things are already going very badly for you, and that at this point civilian-supported guerilla movements are more effective than a conventional centralized army, I can easily see monogovernment planets (that aren't expecting internal rebellion, etc.) only bothering with force-projection training for their surface grunts. In a pinch they'll also be able to fight at home, but having a large number of people trained only for being a sore loser seems like a drain of resources.

I see surface forces being divided more along the lines of whether they're made for surface-to-surface operations (grunts) or surface-to-orbit operations (coastal artillery). And of course, fighting on a non-atmospheric world would be very different, there's probably a specialized branch for that.


"If I have enough superiority to land troops, I also have a fleet in low orbit, which can enforce my demands with lasers and kinetics, like in Footfall. The troops will be there for occupation, not for capturing the area."

I agree. Although... someone... suggested heavily fortified bunkers that are too tough to easily take out from orbit, which, if possible, would provide an incentive for ground forces to try to flush them out.

Sabersonic said...

Time for my two cents in this little topic if discussion and debate.

Killer Robot Ants: Though there is an advantage in having a surveillance and reconnaissance drone/droid small enough to hide from even the eyes of soldiers not equipped with Reality Enhancement and Augment equipment, but one can only go so small. Too small and a single puff of wind would cause it to tumble worse then a leaf. This could be solved by solely ground-based micro-drones but then there's the little problem of being stepped on. And something that small would transmit data rather poorly and equally more vulnerable to electronic warfare.

Combat Drone/Droid: Armed robotics would not be the end-all answer that many futurists both civilian and military have envisioned. Drones are still dependent upon the human operator even if it's given limited intelligence and autonomy for complex tasks and direction. And said operator is dependent upon the security of the link itself which can be disrupted by electronic warfare. Such tactics can potentially be lessened with either more powerful transmitters or moving the drones closer towards the operator. Transmitters would obviously be bulky and power hungry if it needs to be either powerful enough to defeat ECM or generate ECCM and moving the drones closer to the operator will simply make them more vulnerable to enemy fire. And let's not forget that the old electromagnetic pulse can defeat even AI driven droids unless they are hardened against them and even then that protection is dependent upon its distance from the epicenter of the pulse and these protective upgrades would undoubtedly make them expensive enough for their core ability to be "expendable" to be questioned. The best solution (so far) is to have a hybrid deployment of grunts, fire support drones and recon droids.

And while we're on the subject:

"As a complement to PA, expect to see remote piloted, humanoid robots." - adam_grif

"Even if they're completely remote controlled - that is, all movements directed by a human, rather than just a human saying "go this way and start shooting" - it'll need to have a pretty impressive control system to make that work comfortably." - Milo

Somehow I'm reminded of that Surrogates Movie.

Power Armor: Despite the popular notion of the war tech, the primary reason to deploy Power Armor is for mobility and endurance. Armor protection from small arms is merely an after effect and provide moderately better ballistic protection then combat helmets and bullet resistant vests. That is if one wishes to make the armor as economically viable as possible yet provide moderate amount of protection against general purpose rounds (forget specialized rounds such as armor piercing) and yet not sacrifice mobility.

And let's not forget that even though they're built for endurance, depending upon how they're powered they'll still need to be transported within their operational range of any objectives and/or mission zone so APCs and IFVs for PA will likely have a place in this future surface force.

- Hotmail Address
Gmail Address

Hugh said...

I think there's room for skepticism about the effectiveness of precision weaponry when two post-industrial powers go into battle against each other.

What we've seen in the past two decades is mostly precision weaponry being deadly against opponents with a lower "tech level." But my reading of history suggests that against equal opponents, precision weaponry rapidly loses its effectiveness. German guided bombs in WWII got jammed; Israelis came up with countermeasures against Egyptian guided anti-tank missiles in 1973; the Serbians used decoys with great effect in Kosovo. Other commentators have already mentioned the possibilities for jamming.

And, as I believe the US military is starting to think about, there's this whole business of post-industrial nations not being actually able to manufacture a lot of stuff themselves. If there's a couple of thousand T-72 or similar low budget tanks rolling over the border, your outsourced just in time manufacturing chain may not be able to produce enough missiles to keep up with demand.

Byron said...

Milo, deep bunkers do not a planet hold. While I can see them being used, if anyone has enough space superiority to successfully land troops, they also have more then enough to smash any conventional force mustered against their force from orbit. Remember Footfall. If you don't, read it.
And before anyone brings up the Starship Troopers argument, there's a world of difference between landing special force and men, and landing a modern fighting force. I can see special forces being landed early on, assisted by lots of decoys. They would serve in a defense-hunting role, similar to the SCUD hunters in the first Gulf War. However, special forces can't take a planet.
I guess I have to agree that Armies (local defense) won't really exist on unitary worlds, except as guerrilla cadres. If there are other powers on the planet, then a conventional army will exist. The question then is what it would look like. That is dictated by, as much as anything, orbital support. It's possible that both sides would agree not to use it, in a MAD situation. I personally think that orbit may be separated from ground campaigns by mutual interest. If someone shoots one way, pretty soon, everyone's orbital infrastructure is dead. If they do, then forces will have to be exceptionally stealthy to avoid orbital targeting. Still, I find the idea unlikely, particularly as it benefits neither side to destroy their orbital defenses, which protect them against outside invaders. It's possible that space weapons could be under the control of a joint force that is neutral and is there to keep other planets out, even despite the war.

Albert said...

EMP destructiveness is overrated. Shielding isn't really hard to do and military hardware is supposed to be immune due to shielding/engineering tricks.
Look for "faraday cages" for more details.

Radiation is another concern alltogether.

Anyway, ECM and ECCM are the real show-stoppers.
Relay drones may be useful in this regard, they stay close to "frontline" so the other drones don't need very powerful transmitters and designers can focalize more on making them effective at fighting while the relay drone's whole power supply and armor is dedicated to the transmitter able to defeat ECM/generate ECCM.

This way you still have "command bots" like Star Wars's droid motherships, but unlike SW, it would be good to have some more than one per "squad" of drones.

Hugh said...
I think there's room for skepticism about the effectiveness of precision weaponry when two post-industrial powers go into battle against each other.
Yup. I believe most worthy opponents will probably be able to negate about anything from laser weapons (with decent armors and smoke), to GPS-guided bombs (either jamming or outright killing GPS system) and laser-guided ones (frying their sensors with lasers), to air support on any height (lasers, missiles, fighters), to spysats and Predator-like drones.

If there's a couple of thousand T-72 or similar low budget tanks rolling over the border, your outsourced just in time manufacturing chain may not be able to produce enough missiles to keep up with demand. Reassuring. Zerging an enemy with very large numbers of cheap mediocre-to-low quality units should always remain an option.

-Albert

Mangaka2170 said...

Something people don't seem to be considering is the human factor, and I don't mean funerals for dead troops.

If, as some commentators have mentioned on here, armed infantry are largely or entirely replaced with drones, what might have been a conflict people on either side could support essentially becomes a real-time strategy game with real-world consequences, where the victor is the one who can afford to throw more resources into manufacturing than his enemy can. As long as either faction has a manufacturing base remaining the war would not end as the population is largely detached from the conflict since they're not actively engaged in the war effort (look at the apathy many Americans have towards our occupation of Iraq and Afghanistan). One of the things that prevents wars from getting out of hand is that people are risking their lives to defend or improve their country. Take that away and international differences might as well be settled by multiplayer StarCraft.

Even though I'm sure that drones will become more prevalent on the battlefield, there's no way that the good old infantryman will be replaced for long before people realize that wars didn't used to last for decades.

Milo said...

Sabersonic:

"Armor protection from small arms is merely an after effect and provide moderately better ballistic protection then combat helmets and bullet resistant vests."

A powered exoskeleton can carry heavier armor than a human could carry normally. However, it'll be an uphill battle to keep up with the increasing destructiveness of weapons.


"APCs and IFVs for PA will likely have a place in this future surface force."

Agreed.

Powered armor has good agility (compared to tanks), can enter places an unenhanced human would have trouble with by breaking down doors and clearing rubble, and might reduce fatigue and so give troops more endurance - but while it has good mobility, it doesn't have particularly good speed. It also can't carry larger equipment such as a long-term power supply on its own.



Byron:

"I can see special forces being landed early on, assisted by lots of decoys. They would serve in a defense-hunting role, similar to the SCUD hunters in the first Gulf War."

If you can pull off such a landing, this has the potential of being one of the more exciting scenarios for a surface warfare story. Probably with the invaders being the heroes, since they get the benefit of being a small team surrounded by hostiles.


"It's possible that space weapons could be under the control of a joint force that is neutral and is there to keep other planets out, even despite the war."

Possibly, but "neutral" forces with a lot of weapons have an annoying tendency to exploit their political power at an inconvenient moment. Maybe you broke a treaty but as long as you have all the space guns, who cares?

Rick said...

Welcome to a couple of new commenters!

Yes, the recent experience of high tech weaponry has mostly been

Whatever happens, we have got
The Maxim gun, and they have not.


When both sides have it, it will be a different story.

On the human factor, that is also reminiscent of the 18th century. Wars were fought by long-service armies, not mass conscription, and Frederick the Great believed that civilian populations should neither know nor care that there was a war on.

So wars DID drag on for years, and after a few years of peace they were back at it, all without the society actually being very militarized.

The problem with going to ground is how are you supplying your troops?

That IS a problem, which is why the op tempo of future wars might be slower than we expect, due to the need for logistics to be stealthy, which has to make it less efficient than truck convoys or transport planes.

Zerging an enemy with very large numbers of cheap mediocre-to-low quality units should always remain an option.

It is never too late to read Clarke's "Superiority."

Politically unified planets don't have much reason for an army, beyond the surface 'coast artillery' component of a layered space defense. A planet might plausibly have a constabulary, a sort of RCMP to keep order across the vast outback, but in spite of paramilitary training and equipment that is far from being an 'army.'

One trope that literally does not travel well in space is the land power with an all conquering army. Because it doesn't conquer anyone without an equally all conquering space force, and the latter is far more critical.

If you want an interstellar scenario with land centric war, your option is a wormhole style FTL that allows surface gates, so that getting from planet to planet does not involve space travel.

kedamono@mac.com said...

Ground forces face many problems: Landing on a defended world is like ducks landing on a pond guarded with Vulcans miniguns. :o

But in those cases where you can land forces, living, breathing people in the mix will not be in the majority. Soldiers will be in power armor accompanied by functional combat drones. There will be armored vehicles, but no one will be at the wheel or, more importantly, holding the trigger. Warfare through robotic proxy will be the norm.

Military arms will change, from man portable to power armor portable. And they will be designed to penetrate power armor, which means un-power armored combatants go 'sploding like meat filled balloons if they get hit by these weapons.

Still, there is a need for human-scale armor and weapons for those situations where you can't clomp around in power armor, such as we see in Afghanistan. You need to have a human face when you field an occupation force. However, you might see much more in "less than lethal" weapons in these situations. A squad of soldiers may have switchable magazines, LTL in one, and full metal jacketed in the other. And the default setting is LTL.

Byron said...

I'm still not sure about robotic warfare. It works in theory, but there are a number of issues. Complex machinery is usually less rugged than a human, particularly the types of command and control systems you're proposing. If they go, then the whole system does. Also, in many ways, people are less expensive than complex equipment. Take a look at the M1. It was planned to include an autoloader, but it was deleted because the military decided that a soldier was cheaper and more reliable. The Soviets use them, though also to keep the size of the tank down. While that's a limited example, the principle holds true. It's often easier and cheaper to put a human in the machine than to automate the machine. We could probably set the F-16 up to fly on computer control relatively quickly, but would we want to? It'd likely be less effective, and not a lot cheaper than a pilot.
I'm not ruling drones out, but land is an even worse environment for them then low orbit, and we've decided that they're not always practical there.
Also, how is this less-than-lethal to lethal system supposed to work? Just curious.
And I'm not advocating a neutral force. Just raising the possibility that space weapons might be separate from ground weapons.

Tony said...

A couple of things...

A soldier in a powered armor suit is going to be heavy, at least as much as a good sized horse. If you don't believe me, consider this: the weight of protection similar to a SAPI plate, spread over a surface area twice that of an average man (so that everything vital is inside the armored envelope) is over 300 lbs (~= 150 kg). Then you start adding supporting structure, actuators, power systems, fuel, etc. If you want protection against more than machine gun bullets, you start building a trooper the size and mass of a small car.

And this trooper, even the light version, is going to have all of the mobility problems of a horse. Particularly difficult will be operating in muddy environments, gaining traction on steep slopes, and going upstairs in a typical residential structure.

Second, when we start talking about robotic "warriors" we start taking the necessary human element out of war. That not nostalgia or conservatism talking. One of the most fundamental features of war is it's risk-reward nature -- the "unlimited liability", as Peter Young called it, of the commitment made by the soldier/sailor/airman. In a normal civilian job, if it gets to be too personally dangerous, the worker can always quit, or strike for safer conditions. In war, when it gets too dangerous, that is usually precisely the time that one is expected not to quit or go on strike, but to keep on going.

Yet remote, robotic warfare eliminates or overwhelmingly diminishes the risk to the so-called combatant. At that point, conflict ceases to be war and becomes high tech predation.

kedamono@mac.com said...

@Bryon
Also, how is this less-than-lethal to lethal system supposed to work? Just curious.

Well, the easiest way is to have them attached to different barrels. One barrel fires LTL baton rounds/pepper rounds/whatever is in the LTL arsenal in the midfuture.

The lethal rounds fire through a second barrel mounted on top/to the side/underneath the LTL barrel.

The main purpose is to reduce the number of civilian casualties when a soldier reacts and fires off a couple of baton rounds at kid throwing a rock and not bullets. Better PR, better relations.

But you still have the lethal option for someone who isn't a kid throwing a rock.

A thought crossed my mind about the fate of the AK-47. It is the most popular assault rifle out there, mainly because its easy to use, clean and keep firing in situations where FN SCARs would jam solidly. 200 years from now, I'd wager we'll still be seeing AK-47s or a weapon near enough to one out there in the hands of untrained combatants. It's not the best assault weapon in the world, and doesn't pretend to be. It's a assault rifle for a peasant just off the steppes with little or no military training. And there are lot of those out there.

Milo said...

Rick:

"When both sides have it, it will be a different story."

The problem is that Mutually Assured Destruction is inherently preventative of wars between superpowers where both sides have the big guns. Unless some way is found around it, all future wars will either be small-scale "safe" operations against not-so-threatening enemies that don't risk escalation into nuclear armageddon, or feuds between tribal warlords who have technology roughly equal to each other but vastly below that of society's more impressive locales.


"A planet might plausibly have a constabulary, a sort of RCMP to keep order across the vast outback, but in spite of paramilitary training and equipment that is far from being an 'army.'"

Which doesn't mean they can't be pressed into service as one in an emergency. If it comes to total war, a lot of people will find themselves doing things they never meant to.

Mostly though, a constabulary would be trained for occupying territory and keeping order (the things you typically want to do when invading a planet), as opposed to destabilizing it and causing trouble for the people in charge (the things you typically want to do when resisting an invasion). So their usefulness as a home defense force would be limited, except to catch enemy special forces and spies before they're able to launch a full invasion.


"One trope that literally does not travel well in space is the land power with an all conquering army. Because it doesn't conquer anyone without an equally all conquering space force, and the latter is far more critical."

Yes. On Earth, naval travel is often faster even when a land route is available, but it's not the ONLY way to get from point A to point B unless one of those points is on an island (most notably Britain, which did indeed exploit its location in this manner). In interplanetary or interstellar warfare, though, spaceships are mandatory. Unless you have ground-based wormholes, of course, but most settings don't for the simple reason that they're not as cool.

On the other hand, unlike naval travel on Earth, when a land route is available, you'll never bother with spaceships - except maybe a suborbital flight for really quick deliveries, but those aren't really spaceships.

Thus space navies and nautical navies have in common that they're the most important form of travel in their setting, but for very different reasons.

Milo said...

kedamono@mac.com:

"Ground forces face many problems: Landing on a defended world is like ducks landing on a pond guarded with Vulcans miniguns. :o"

Hey now. Water is a pretty good defense against bullets. If the ducks dive down quickly enough it'll be hard for those miniguns to hit them. Depends on the size of the pond, of course...

And then they have to figure out how do actually do something since those ducks can't stay underwater forever. Are those ducks armed?


"You need to have a human face when you field an occupation force."

You can pull that off with bulletproof-glass visored power armor, if it isn't too bulky.

Speaking of which, is there any way to make transparent armor that's effective against lasers? It seems like an oxymoron - if it's transparent then it can't stop light - so maybe some kind of reactive armor, that's normally transparent but changes color/releases smoke/etc. when hit by an overly strong light pulse? We have sun-sensitive glasses now, but those adjust way too slow to stop a laser weapon.

You could just have it be transparent to a few frequencies and lock out the rest, but that only works until the enemy figure out a way to adjust their lasers to shoot at the correct frequency to get through your armor.


"A squad of soldiers may have switchable magazines, LTL in one, and full metal jacketed in the other. And the default setting is LTL."

Another score for energy weapons, which (at least for some designs) can be easily tuned in output rather than having a bunch of separate ammunitions clogging your logistics. Electrolasers are particularly well-suited for tunable variable-lethality shooting, and I expect them to have relatively reliable effects regardless of which part of the body they hit (since the electricity will conduct), although they do poorly against anything with insulating armor.

Milo said...

Byron:

"And I'm not advocating a neutral force. Just raising the possibility that space weapons might be separate from ground weapons."

Handled by a separate branch of the military? Sure. Loyal to different political structures? Less likely.



Tony:

"If you want protection against more than machine gun bullets"

Protection against machine gun bullets is pretty good.


"And this trooper, even the light version, is going to have all of the mobility problems of a horse."

It needs less food and it's at less of a risk of breaking a leg when handled roughly. That counts for something.


"Particularly difficult will be operating in muddy environments, gaining traction on steep slopes, and going upstairs in a typical residential structure."

Going upstairs is a minimum necessity of power armor. Fail at something that simple and you've lost the justification for using humanoid vehicles in the first place. However, it shouldn't really be difficult as long as the stairs can support your weight (likely, though ladders will be more troublesome) and your sabatons are small enough to fit on the steps, you should be fine. Powered motors will take care of the weight problem of going up.

The traction thing could be a bigger problem though. Rough surfaces (possibly extensible, though you want to keep your equipment as simple as possible) or gecko glue (which you may well have anyway for climbing sheer walls) will help with the slope. I'm not sure what to do about mud, although the main problem here is that you'll sink in deeper than an unarmored human would and so need to pull harder to move your leg. You'll still do better than wheeled vehicles, I'm sure.

Should you happen to be on a muddy slope, you're in trouble.

Byron said...

I'm also skeptical of powered armor. It seems that if we're putting someone in a giant suit, it might be easier to just make it a small vehicle. You get similar mobility, and no real drawbacks, if the numbers are of the sort you're looking at. The only real place I can see for powered armor is in space or other hostile environments, when you need to protect the occupant from the environment. Plus, the mobility isn't as much of a concern in space, because of the lack of gravity. Still, if the engineer was competent, the structure and armor might be one and the same, saving a lot of weight. Power draw would still be a huge issue, though.
And the LTL weapon is just two put together. While it's a good idea, it really is no different from loading an M203 with baton rounds. The only real practical switchable weapon I can think of is a combined laser/electrolaser weapon.
And the AK will undoubtedly be around for a long time. I really doubt that civilian weapons will go much farther than they are now, simply because more firepower is superfluous except in paramilitary situations, which normal people won't get into. That means that smokeless cartridges will be around for a long time, and that AKs should have ammo. The reason nobody uses weapons from before smokeless powder is ammo. It's too hard to get, and isn't as good.

Tony said...

Milo:

"Protection against machine gun bullets is pretty good."

Not at the cost of certain types of mobility. Remember, the power in powered armor is there to move the armor, not the soldier.

"It needs less food and it's at less of a risk of breaking a leg when handled roughly. That counts for something."

A soldier in a powered armor suit is supposed to be an infantryman, yet his mass, bulk, and ground pressure are going to be that of a cavalry horse that he can't run as fast as. I'm not sure I see a mission for such a critter.

"Going upstairs is a minimum necessity of power armor. Fail at something that simple and you've lost the justification for using humanoid vehicles in the first place. However, it shouldn't really be difficult as long as the stairs can support your weight..."

Yet that is precisely the problem I foresee. If a soldier weighs half a ton or more, with all of that weight concentrated into an area the size of a couple of moon boots, he starts to challenge the load bearing capabilities of the average wood frame structures, and would almost certainly collapse the upper floors of many Third World structures.

Byron said...

Milo, you're nitpicking.
The point is that people won't have conventional armies for home defense. A constabulary might be used as one, but it's not maintained to keep the planet safe from invasion.
The pond analogy was that they wouldn't get to the water. Again, nitpicking.
Why would you want transparent armor? A face shield I can understand, and it might be possible to make an ultra fast acting visor. Still, a man in a giant metal suit is only marginally better than a man in a vehicle. He can't come in and sit down for tea. If he did, he'd crush the sofa. Hearts and minds are everything in that business, and I doubt powered armor would make a good impression.
Protection against medium machine gun bullets isn't anything to scoff at now, but that's only because we use a certain size of machine guns. That's because they fulfill the current requirements, which include dealing with unarmored or lightly armored humans. In theory, it'd be possible to arm troops with something like .50 Beowulf rifles, which would be far more lethal. The military doesn't because 5.56 NATO is adequate against most threats, and a lot lighter and has less recoil. That doesn't mean that, if, say, men in powered armor (or demons) attacked, we wouldn't get a more lethal weapon.
The problem with going up stairs is that stairs are designed for normal humans. OSHA requires stairs to be designed for 510 lb. If we assume that half the weight of the suit is the armor, then a man in powered armor will weigh about 750-800 lb. That means he can't use normal stairs. Not to mention that the feet will likely be too big, and raise the ground pressure too much to be workable.
The comparison was to the horse's mobility problems, which don't include food. They include things like ground pressure. And what about tracked vehicles? They might be better. I know the M29 Weasel had a quarter the ground pressure of a human.

Milo said...

So... what do people think about kinetics vs lasers vs other stuff for ground warfare? Assuming that lasers have been made to work at all, and can be miniaturized into a handheld model.

Lasers have no recoil (okay, absurdly tiny recoil), don't need to lead the target (lightspeed lag is negligible when shooting at distances as short as a single planetary diameter), and aren't affected by wind/gravity, all of which add to their accuracy compared to kinetics. You can also make use of laser targeting, obviously.

On ground, you are going to have to worry about weather. Lasers work better in wind, while kinetics work better in fog or rain (or smokeshields, not natural weather but working on the same principle). Ideally you would try to find a wavelength of light that can penetrate fog well, but I don't know if that can be done.

Lasers can't be used in indirect fire, which is important for shooting at long ranges across a cluttered battlefield and the curvature of the planet, so I expect they will not be used for surface-to-surface artillery - shells and missiles will continue to dominate in this area. Lighter hand weapons will typically be used at shorter ranges so this is not an issue for them.

Lasers have simpler logistics - shots of different calibers will simply use different amounts of electricity, rather than needing separate ammunition stores. And this electricity can be loaded from anywhere that has a power supply, rather than dedicated ammunition plants. And it's not just the ammunition that's flexible - the guns themselves could be quite easily tuned between high-caliber, low-rate-of-fire mode and low-caliber, high-rate-of-fire mode. A really good gun might even be able to vary armor-piercing qualities by tuning the pulse profile.

Overall there seem to be enough advantages to lasers that you'd have a motive to use them even if their shots are more expensive, dollar-for-kilojoule, than equivalently-damaging kinetic shots.

What do you think would be the best weapons for various kinds of tactical roles?

Tony said...

Milo:

"So... what do people think about kinetics vs lasers vs other stuff for ground warfare? Assuming that lasers have been made to work at all, and can be miniaturized into a handheld model."

Handheld lasers/phasers/blasters/powerguns are a particularly persistent but essentially impossible expression of magitech. Not merely improbable, or implausible, but impossible. The implied energy densities of batteries that could be fit into the volume of a pistol butt or even a rifle magazine -- or a back pack with an umbilical to the weapon, for that matter -- are so high that the power source would be too unstable and vulnerable to catastrophic failure. The guy with a flamethrower on his back during a Pacific island assault in 1944 would be at less risk, perhaps a lot less.

I don't know about others, and can only speak for myself, but I can't imagine a mission for such a weapon.

Byron said...

I understand the benefits of lasers, but I'm not sure they are all that useful in most combat situations. Generally, infantry combat takes place at ranges under 500m, which makes the long-range capabilities of lasers rather pointless. I don't think that most misses are because of the weapon. It's generally more the operator's fault. I'm also going to assume that lasers are generally more delicate than an equivalent gunpowder firearm. That is always a concern on the battlefield. I'm not claiming that they are useless. Far from it, in fact. They would be useful for engaging high-speed targets, such as aircraft, and artillery shells, and for long-range fire. In fact, I expect the first infantry-fielded laser to be a sniper rifle.
The problems with electricity as a source of ammo are numerous, however. First, rechargeable batteries are going to be heavy, and you can't recharge them if you don't have electricity. More specifically, the right kind of electricity. I know someone who literally melted her hair by using an American curling iron in Switzerland. Also, the electricity has to be working. This could be a problem if you enemy takes out the power plant. For more details, see the section from Hammer's Slammers at Atomic Rockets.
All of the above factors serve to make lasers more useful when mounted on vehicles. If they do make it to small arms, I expect they'll work very like modern small arms, possibly down to the individual disposable batteries. The M1 Garand should have taught us about that. The last problem is bulk. Lasers take a mirror, and it's sort of hard to fit one into a small arm without making it awkward to store and carry. Plus, the mirror isn't safely buried away.
A third possibility is coilguns. They're likely to be more efficient than lasers, and have the advantage of the round not being effected by rain or fog. The power issues are the same, but they should be less bulky.
Tony, if we got the efficiency of the weapon to be similar to a modern rifle, and got a power source of similar energy density, then it's very possible. Nitroglycerin has less than three times the energy density of our best batteries today. It's not beyond the realm of the possible that they could reach parity.

Milo said...

Byron:

"The point is that people won't have conventional armies for home defense."

Which is what I was saying. You were the one who was suggesting they would.

But I also pointed out how a constabulary in your setting might be relevant (or not) to a war story.


"The pond analogy was that they wouldn't get to the water. Again, nitpicking."

No, joking. Come on, I was talking about the military tactics of deploying ducks.


"Still, a man in a giant metal suit is only marginally better than a man in a vehicle. He can't come in and sit down for tea. If he did, he'd crush the sofa."

I figured they'd be doing stuff like manning checkpoints. Where you need to interact with people who pass through but also to present a hardened position that doesn't invite attack.


"OSHA requires stairs to be designed for 510 lb. If we assume that half the weight of the suit is the armor, then a man in powered armor will weigh about 750-800 lb."

Okay, yeah, that would be a dealbreaker. Gotta get that down to something that can handle normal urban environments, even if that means having less armor.


"That doesn't mean that, if, say, men in powered armor (or demons) attacked, we wouldn't get a more lethal weapon."

I'm going to quote Rick here:
"Yes, there are bunker busters and Thor bolts, but the point is that such big powerful weapons are costly to deploy, carried by vulnerable platforms, and can be engaged by defensive fire."

Forcing the enemy to break out the big guns is usually going to provide you with some sort of advantage, since those big guns are bound to have disadvantages - cost, rate of fire, recoil, whatever - that justified them not already using those weapons in the first place. The question is whether or not the advantage you get is worth the price of your armor.

Oh, and speaking of recoil again, that's another count for lasers. Currently recoil is the biggest limiter on handheld infantry weapon calibers - soldiers can physically carry heavier guns, but they'd dislocate their shoulders firing them (hence tripods, etc.). Lasers could potentially allow heavier handheld weapons without the tradeoffs we face today, if you can cram a big enough power source into the available space, and deal with the waste heat.

Tony said...

Byron:

"Tony, if we got the efficiency of the weapon to be similar to a modern rifle, and got a power source of similar energy density, then it's very possible. Nitroglycerin has less than three times the energy density of our best batteries today. It's not beyond the realm of the possible that they could reach parity."

In theory, yes. The problem is that theory isn't practice. In practice, one has to control the discharge of the available energy. With chemically powered firearms, the way we do that is accept that a sufficiently powerful and quick chemical reaction will produce a lot of heat and pressure that have to be contained by brute force. (Which is why the chamber walls are and have to be significantly thicker than the barrel.)

An electrical power source that can support an equivalently speedy and dense discharge of energy is going to be, well, volatile. Lithium batteries, like the one in your laptop for example, are capable of starting serious fires when they malfunction. Ones left out in the sun too long explode. Imagine a battery at even higher energy densities capable of discharging in a few milliseconds. Imagine what happens when something like that is dropped, or worse, hit by a bullet or shell fragment.

I didn't mean to say it was impossible for fundamental reasons, but it is practically impossible, given what we know about the chemistry of materials that would go into making up such batteries. The higher the energy density and the quicker that energy can be released, the more unstable the device. It's really the same thing as chemical propellants, just chanelling the energy release into photons instead of expanding gas pushing a projectile.

Byron said...

Yes, there would be disadvantages to the bigger guns, but they would in no way be worth the cost of the armor. If they have to switch back to 7.62 NATO, then there is greater recoil and lower control on automatic, but those have been dealt with before. Most of the world used 7.62mm rifles for 40 years. The reason they don't is that the nature of infantry combat has changed. Infantrymen don't stand in the open and shoot at each other, or even generally engage targets at long range. When they fight, it's at close quarters, were the greater weight of fire makes a difference against unarmored humans. If the humans become armored, the tradeoffs change. What I'm trying to say is that it's easily possible to equip an infantryman to deal with your powered-armor trooper. It would make him slightly less effective against normal humans in close quarters, but if I'm facing troops in powered armor, that's not too much of a concern.
So what has your powered armor accomplished? I've had to rearm my men, but once that's done, they can kill you despite your fancy, expensive armor. Modern infantry weapons are the result of a series of tradeoffs based on current conditions. If the conditions change, then so do the weapons.
Tony:
What about capacitors? Or armoring the batteries? We generally don't panic about people dying because a round hit their ammo. I know, there are some differences, but I don't think it's totally impossible. What if we used fuel cells instead, for instance? Or some other means of generating electricity? Molten-salt batteries, maybe? I can understand Lithium-derived versions not being a good idea, but that doesn't rule them all out.

Milo said...

Tony:

"The implied energy densities of batteries that could be fit into the volume of a pistol butt or even a rifle magazine -- or a back pack with an umbilical to the weapon, for that matter -- are so high that the power source would be too unstable and vulnerable to catastrophic failure."

Nonsense.

A laser that can fire shots doing comparable damage to a pistol (which is sufficient for killing a human) with a number of shots comparable to a pistol (which for current semiautomatics is pretty good - we're not stuck with sixshooters these days) would only need to carry total energy comparable to a pistol. Current lasers aren't managing it because they have very poor efficiency compared to present-day kinetics. An efficient laser (no more of a stretch than an economically feasible interplanetary merchant ship) could give you a pretty good hand weapon without blowing up on you, as long as you remember that it's a hand weapon and doesn't need to carry enough energy for vaporizing 100 main battle tanks between reloads. You can still afford to carry more energy than current guns do, as long as you don't try to carry multiple orders of magnitude more.

Gunpowder, if you'll recall, is an explosive, but that doesn't make guns impossibly dangerous to carry. Batteries would actually be safer if their design isn't conductive to releasing all their power at once (depends on the design, really - the best batteries of today are at risk of catching fire but not, I think, of actually exploding).



Byron:

"I'm also going to assume that lasers are generally more delicate than an equivalent gunpowder firearm."

Likely. That is indeed a problem.


"In fact, I expect the first infantry-fielded laser to be a sniper rifle."

Also likely.

Although I'll point out (woo! nitpicking!) this could be problematic if your laser has a visible beam the enemy can use to trace your sniper. Yes, in vacuum lasers are invisible, but air causes a little scattering and misty air causes a lot of scattering. You could deal with this by using invisible wavelengths (even if some enemies have sightscopes, not all of them will) or just by only deploying those snipers on clear days, which is a pretty sensible idea for snipers anyway.

Milo said...

Byron:

"First, rechargeable batteries are going to be heavy,"

Not that bad. According to this, a present-day high-end battery has an energy density comparable to a low-end chemical explosive (like gunpowder). Midfuture tech could plausibly be a little better if you want it to be.


"and you can't recharge them if you don't have electricity."

True, but a portable generator isn't going to be much harder to carry than multiple crates of ammunition, and is more flexible in usage.


"I know someone who literally melted her hair by using an American curling iron in Switzerland."

That can be solved by installing an adaptor. It's only a problem if you aren't aware of the local voltage or of what voltage your equipment uses.

I expect that most planets except Earth are going to have a unified voltage (why not?), and militaries that aren't stupid are going to read up on their target before invading.

An adaptor, again, much easier to carry than crates of ammunition.


"Also, the electricity has to be working. This could be a problem if you enemy takes out the power plant."

Or the enemy could take out your warehouses/factories. It's inconvenient but there are backups.

Meanwhile I note you don't mention the one disadvantage of rechargable batteries I actually agree with: namely that recharging a battery takes much longer than replacing a magazine. This could, of course, be dealt with by having spare batteries and recharging one while the other is installed in your gun.


"A third possibility is coilguns. They're likely to be more efficient than lasers, and have the advantage of the round not being effected by rain or fog. The power issues are the same, but they should be less bulky."

I'm not so sure about the benefit of those, actually. They still need to load a projectile even if the energy comes from electricity, so you don't get any of the advantages lasers have when reloading - once you already need individual specially-made bullets anyway, you might as well manufacture them with their own propellant. Since they're still based on projectiles, they'll have more or less the same deployment tactics as current guns.

Coilguns would be useful if you can get them to hit harder or cheaper than explosive-propelled guns, but they have no inherent advantage.

Byron said...

From what I see, the best energy density we get out of batteries is under 50% of nitroglycerin. That's not terribly close.
I'm not sure the beam is going to be a problem. It only lasts for fractions of a second, it's fairly diffuse, and you have to be looking right at it. All it will do is localize the shooter, not pinpoint him.
The problem with using rechargeable batteries for infantry is also cultural. Militaries, when forced to choose between a battery that is rechargeable,and laser bullets, will go with the bullets. It fits into the supply system. The problem with getting electricity on-site is that it can't be counted on. For planning purposes, the military will assume it isn't there. They'll bring anything themselves.
Coilguns could have an advantage over gunpowder firearms in several areas. First, they can fire at much higher velocities. This reduces recoil for given energy, making weapons more lethal. Secondly, they can be made more consistent than rifled firearms, leading to better accuracy. Thirdly, it would be possible to fire a very fast burst of small slugs, increasing penetration of the target, as well as hit probability.

Tony said...

Byron:

"What about capacitors? Or armoring the batteries? We generally don't panic about people dying because a round hit their ammo. I know, there are some differences, but I don't think it's totally impossible. What if we used fuel cells instead, for instance? Or some other means of generating electricity? Molten-salt batteries, maybe? I can understand Lithium-derived versions not being a good idea, but that doesn't rule them all out."

Nitrocellulose and battery (or high density capacitor) chemicals are really, fundamentally just energy storage mediums. Nitrocellulose is relatively safe in its delivered form because the energy is put in at a factory through a series of pretty dangerous chemical processes, then fixed in place through other processes. But the resultant handling safety and shock resistance are a function of optimizing the material to do its job in a very specific way.

Even at that, subjecting nitrocellulose or similar chemical propellants to concentrated heat -- in the form of a nearby fire or hot projectiles penetrating the container -- can be a really enlightening experience. If you've ever seen video of an ammo fire in an armored vehicle, you know what I'm talking about.

Now, the problem with electrical energy stored in chemical form is that when you go to use it, you're trying to take it out as raw electrons, moving at a significant fraction of c, not through a mediating expansion of gaseous combustion byproducts, moving a few thousand feet a second. The way to make a chemical storage medium do that is to get it and keep it in an excited state, ready to liberate all of its spare electrons nearly all at once. IOW, it's not a combination of chemicals ready to burn, it's a single chemical, or mixture, already put into a state where any change in the environment will cause discharge. It's like carrying around an ongoing explosion in a magic bulb that will release the explosion in a desired way through a certain interface if you get the spell right, but which will also just blow up in your face if you do anything to upset the djinni that governs it.

Of course, one could imagine a supercompact explosive current generator, but then why not just use the explosives to propel a bullet? Or, if you really want to use the electrical power, what is you plan for containing the explosion and safely discarding the waste heat?

Tony said...

Byron:

"What about capacitors? Or armoring the batteries? We generally don't panic about people dying because a round hit their ammo. I know, there are some differences, but I don't think it's totally impossible. What if we used fuel cells instead, for instance? Or some other means of generating electricity? Molten-salt batteries, maybe? I can understand Lithium-derived versions not being a good idea, but that doesn't rule them all out."

Nitrocellulose and battery (or high density capacitor) chemicals are really, fundamentally just energy storage mediums. Nitrocellulose is relatively safe in its delivered form because the energy is put in at a factory through a series of pretty dangerous chemical processes, then fixed in place through other processes. But the resultant handling safety and shock resistance are a function of optimizing the material to do its job in a very specific way.

Even at that, subjecting nitrocellulose or similar chemical propellants to concentrated heat -- in the form of a nearby fire or hot projectiles penetrating the container -- can be a really enlightening experience. If you've ever seen video of an ammo fire in an armored vehicle, you know what I'm talking about.

Now, the problem with electrical energy stored in chemical form is that when you go to use it, you're trying to take it out as raw electrons, moving at a significant fraction of c, not through a mediating expansion of gaseous combustion byproducts, moving a few thousand feet a second. The way to make a chemical storage medium do that is to get it and keep it in an excited state, ready to liberate all of its spare electrons nearly all at once. IOW, it's not a combination of chemicals ready to burn, it's a single chemical, or mixture, already put into a state where any change in the environment will cause discharge. It's like carrying around an ongoing explosion in a magic bulb that will release the explosion in a desired way through a certain interface if you get the spell right, but which will also just blow up in your face if you do anything to upset the djinni that governs it.

Of course, one could imagine a supercompact explosive current generator, but then why not just use the explosives to propel a bullet? Or, if you really want to use the electrical power, what is you plan for containing the explosion and safely discarding the waste heat?

Tony said...

Byron:

"What about capacitors? Or armoring the batteries? We generally don't panic about people dying because a round hit their ammo. I know, there are some differences, but I don't think it's totally impossible. What if we used fuel cells instead, for instance? Or some other means of generating electricity? Molten-salt batteries, maybe? I can understand Lithium-derived versions not being a good idea, but that doesn't rule them all out."

Nitrocellulose and battery (or high density capacitor) chemicals are really, fundamentally just energy storage mediums. Nitrocellulose is relatively safe in its delivered form because the energy is put in at a factory through a series of pretty dangerous chemical processes, then fixed in place through other processes. But the resultant handling safety and shock resistance are a function of optimizing the material to do its job in a very specific way.

Even at that, subjecting nitrocellulose or similar chemical propellants to concentrated heat -- in the form of a nearby fire or hot projectiles penetrating the container -- can be a really enlightening experience. If you've ever seen video of an ammo fire in an armored vehicle, you know what I'm talking about.

Now, the problem with electrical energy stored in chemical form is that when you go to use it, you're trying to take it out as raw electrons, moving at a significant fraction of c, not through a mediating expansion of gaseous combustion byproducts, moving a few thousand feet a second. The way to make a chemical storage medium do that is to get it and keep it in an excited state, ready to liberate all of its spare electrons nearly all at once. IOW, it's not a combination of chemicals ready to burn, it's a single chemical, or mixture, already put into a state where any change in the environment will cause discharge. It's like carrying around an ongoing explosion in a magic bulb that will release the explosion in a desired way through a certain interface if you get the spell right, but which will also just blow up in your face if you do anything to upset the djinni that governs it.

Of course, one could imagine a supercompact explosive current generator, but then why not just use the explosives to propel a bullet? Or, if you really want to use the electrical power, what is you plan for containing the explosion and safely discarding the waste heat?

Tony said...

Milo:

"Nonsense."

Please see my comments to Byron. The problem with stored electricity is not that it need be any more energetic than chemically stored energy, but that it's in a state designed to facilitate its release, not its storage. For this reason batteries and capacitor design is most concerned with insulation and discharge control. A round of ammunition, on the other hand, has to be specially designed to get it to go boom. A sensitive primer has to be installed and the discharge contained in small space, or nothing happens. A bullet or fragment penetrating a high-density, quick-discharge storage device breaks the containment of energy already in a releasable form. Oops.

Milo said...

Byron:

"For planning purposes, the military will assume it isn't there. They'll bring anything themselves."

Yes, but bringing it yourselves is still easier if you can just plug your lasers into your ship's power plant, or into a portable generator onboard whatever vehicle you brought with you, rather than carrying crates of single-purpose ammunition. The ability to also occasionally recharge at an unfamiliar outlet is just a bonus for if things don't go according to plan.

Carrying electricity, electricity, electricity, and electricity will greatly simplify your logistics compared to carrying pistol bullets, machine gun bullets, sniper rifle bullets, and tank gun slugs, and that's important when logistics are difficult. Militaries have in the past comprimised on armament versatility in order to simplify logistics.


"Coilguns could have an advantage over gunpowder firearms in several areas. First, they can fire at much higher velocities."

This makes coilguns useful for vehicle-based weapons when you want to shoot harder than chemical-based weapons are reasonably capable of (which is limited by the speed of sound in gunpowder or something like that), but infantry soldiers can't handle that kind of recoil and the equipment is likely to be heavier and more complicated than an AK-47.

You can get the recoil back down by using lower-mass bullets, which could theoretically also allow you to carry more ammunition... except I want to see how you're planning to design a hand-holdable ammunition feed system capable of isolating and firing individual grains of sand.


"Thirdly, it would be possible to fire a very fast burst of small slugs, increasing penetration of the target, as well as hit probability."

Hmm. Now that's an interesting application. I could actually see that being useful.

In this case an individual "cartridge" would be a shell with multiple slugs inside it. The concept is analogous to three-round-burst tactics used with assault rifles today.

Byron said...

What sort of battery are you talking about? I don't understand quite what you mean. A normal battery takes electrons from one side, and pushes them up the other. It doesn't release them at a significant fraction of C. Nor are they that effective as an improvised explosive. They'll explode if used wrong, but the way you describe it, it'd be better to just use them as a warhead. Why don't we just do that, or replace the gunpowder with these batteries?
Milo, I never said the ammo would be that small. There is a lower limit, we just haven't reached it. Plus, it's time for a physic review. Energy is equal to one-half of mass times velocity squared. Momentum is mass times velocity. I can get more energy for the same momentum (recoil) by decreasing mass and increasing velocity. The problem is that you can only get so much velocity out of a gun with a given barrel length. Plus, you run into weird effects with the cartridges and barrel wear. Other mechanisms can get around that. And yes, pretty much true on the cartridge thing.

M. D. Van Norman said...

“What happens when post industrial forces fight each other?”

Someone apologizes.

Rick said...

Welcome to another new commenter!

I encourage everyone to be mindful of language that could be read as jumping on someone.

I'm not informed enough to judge the laser hand weapons question, but one point I'd make is that the actual chemical propellant is a pretty small fraction of the mass of conventional ammo - most of it is in the shell, cartridge, etc. So a battery could be ten times heavier than the gunpowder it replaces and still not necessarily add all that much to the weight of kit.

On power armor, the mobility point is why I'd opt toward lighter armor, designed to protect from small arms fire. After all, when power armor is first introduced, there will be an awful lot of small arms out there.

And as noted, while you can give the grunt a heavier weapon, this has its problems, including a heavier supply chain. Power armor doesn't make you impervious, just harder to kill.

Byron said...

Rick, the 300 lb figure was for protection from non-armor-piercing 7.62 NATO rounds. That's a pretty small arm. I wouldn't feel comfortable with anything less for people I was sending into combat. Those sort of weapons are organic to the platoon level. And the supply chain might be slightly heavier, but we've had 7.62 as the standard round before, and with armor-piercing rounds it would work against that sort of armor. It doesn't really work. Lighter armor isn't worth it from a military point of view, and anything heavier just compounds the mobility problem.

Raymond said...

All this talk of power armor and batteries seems to miss a couple salient points:

- Advanced batteries (prototype stage) such as silicon nanowire cells are expected to have energy densities measured in megajoules per kilogram, and power densities in tens of kilowatts per kilogram. Considering modern infantry rifle rounds have a kinetic energy of 1.5-3 KJ, I really don't think the ammunition supply is going to be much of an issue in any particular engagement, whether laser, railgun, coilgun, or electrothermal gun. The weapon itself will have more mass than the ammunition, which is the inverse of today's infantry gear. The armor itself will have plenty of juice as well.

- The main reason to use rail/coil/electrothermal guns is to get the round into the hypersonic velocity range, around Mach 6 or roughly 2 km/s, where penetration is entirely a question of length and density. For comparision, the APFSDS round fired from an M1 Abrams clocks in at around 1.5-1.7 km/s. Basically, whatever armor your power suit lugs around is going to be tissue paper against a direct hit, but that's not really what the armor's there for, which brings me to the next point.

- We have reached the velocity limits inherent in chemical propulsion of kinetic rounds, which is entirely determined by the deflagration rate, which is tied to the speed of sound in the expanding hot gas. The Interceptor system used by US forces will stop 7.62x51mm NATO, and reportedly 7.62x54mm Dragunov AP-I. Any power suit worth its mass in propellant will be covered head to toe in similar (or better) armor. The colonial steppe wanderers with their future AKs won't so much as dent it.

- Similar protection will be afforded against most forms of artillery, especially the more mobile and less strategically vulnerable weapons such as infantry mortars, hand grenades, and fragmentation from RPGs. Considering artillery is by far and away the biggest threat to infantry on the battlefield (90% of infantry casualties in WWII), this alone makes powered armor indispensable. It will also allow the infantry to carry its own indirect fire support wherever it goes - also a Big Deal.

- On the strategic side, given that surface-to-orbit weapons may be very easy to hide (especially submarine-launched systems), and planetary atmospheres will absorb and/or scatter the wavelengths used in long-range reach-down-from-geosynch-and-tap-your-nose spaceborne lasers, low orbit is going to be a no-mans-land for a significant chunk of a campaign. Orbital fire support against mobile weaponry will be difficult, probably requiring hours of advance notice and exposure to ground fire. Landing forces, however, are much trickier to intercept successfully; the primary vulnerability is in the boost phase, not reentry, especially if you have a whole planet's worth of possible vectors to choose from when invading. I suspect special forces-style precision missions will be the norm, and will be essential to neutralizing enough of the planet's ASAT weapons to bring your main force into low orbit permanently, at which point the fire support becomes overwhelming, and the tide of the war turns for good.

Albert said...

All this talk about lasers...
an expert's laser gun concepts
and about exploding power supplies...
the same expert take on the dangers of his power packs

Generally, lithium ion batteries are more than enough to give you plenty of shots.
And they don't blow up in a so catastrophic way.

-Albert

Luke said...

Since people are discussing the likely behavior of laser weapons, I thought I might as well chime in.

Milo lists several advantages and disadvantages of laser weapons compared to projectile weapons. There are a few other points of comparison which should be added.

First, lasers are likely to be extremely accurate. In order to work at all, they will require active focusing and probably adaptive optics. This means that adding beam stabilization, essentially the same as image stabilization on modern cameras, will be trivial. In addition, since light will take the same path from the target to the laser as it takes from the laser to the target, all the user has to do is aim at where he sees his target, and the laser will go there. Combined with the beam stabilization (which doubles as image stabilization of the targeting scope), the user will be able to hold the beam rock steady and not have to worry about any deflection of the beam (unlike with rifles, where you have drop due to gravity and deflection due to wind).

Another advantage is that any laser which can drill a hole through a person will be able to drill through a surprising amount of armor. The reasons are somewhat complex, and depend on the method by which the beam interacts with the target, but can largely be broken down into two categories. The first is using a relatively long pulse of laser light with a relatively low power. Note the term relative - something on the order of 40 MW and 1 millisecond pulse duration, for 40 kJ of total beam energy. I call it a heat ray. This will be enough to "burn" a 1 cm wide hole through a person. It is not really burning, of course, there is no immediate oxidation of the tissues at the front of the drill hole. Rather, the heat from the laser evaporates any water in the tissues and causes thermal decomposition of organic molecules. The same beam incident on steel will produce a thin melt zone on the order of a micron thick which is continually eroded away by the small portion of the melt that evaporates. The evaporate produces a hypersonic jet of steel vapor that breaks up the melt layer into droplets and throws them out of the hole. As a result of the melt erosion, it does not take much more energy to drill through steel than it does to melt through it. That 40 kJ beam will drill a 1 cm hole through 3 cm of steel - much better armor penetration than an armor piercing 7.62 NATO round.

(due to length restrictions, this message is split into multiple posts)

Luke said...

The second category is what the military has acronymed a Pulsed Impulsive Kill Laser (PIKL), which is described as "chewing" though armor and structural materials. I just call it a blaster. This uses a series of high power, short duration laser pulses. The pulse duration may be on the order of a nanosecond or less, and the total pulse energy could be around 100 J (about as much as a firecracker). A train of perhaps 50 of these pulses will be emitted in short order, probably in less than a millisecond. Each pulse blasts out a crater on the back of the crater of the previous pulse. The performance of this PIKL is rather more speculative than that of the aforementioned heat ray, but the physics of explosions excavating craters is well known. Using values of material strength appropriate to ballistics gelatin, I estimate that it will take about half a kJ total for a pulse train to blast a hole through a person. When applied against armor, an interesting phenomenon occurs - so long as you can maintain a tight enough focus that your laser point of focus is smaller than the crater produced by a point explosion with the energy of one pulse, the volume of armor removed per pulse is proportional to the armor's material strength. This is much the same as with penetration by a projectile when you have ductile deformation of the armor (ductile deformation is when the armor is pushed to the side, rather than forward - when the armor pushes forward you have fracturing and either a plug of armor is broken off and pushed out by the projectile or you get radial cracks leading to 'petals" which are bent aside by the projectile). The difference is that during ductile deformation, the projectile must excavate a hole of constant width, whereas in stronger armor, the width of the hole drilled by a PIKL laser is the width of the crater it excavates which gets smaller as the material gets stronger. Consequently, whereas ductile penetration by a projectile is proportional to the ratio of the material strength, penetration by a PIKL laser at close focus is proportional to the cube root of the material strength. I estimate that a beam composed of 50 pulses of 10 J each spaced 20 microseconds apart and focused into a 1 mm spot size can blast all the way through a person and then some, and can blast through about 4.5 cm of armor steel (RHA). If the energy per pulse is increased to 100 J (for a total beam energy of 5 kJ), the beam will punch through 9 cm of armor.

By comparison, at close range an armor piercing 7.62mm NATO bullet can penetrate about 1.9 cm of armor steel (RHA). Using the Tate model of long-rod penetration (which works quite well for projectiles ranging from rifle bullets to long rod tank penetrators to the hypervelocity jet of a Munroe effect shaped charge HEAT round) I predict that a 3 cm long coilgun flechette of tungsten carbide traveling at 3 km/s with a 20:1 aspect ratio will have a kinetic energy of 6 kJ and will penetrate about 4.9 cm of armor steel (RHA).

Luke said...

Responding to Tony's claim that the power supply of a laser is physically impossible. This is clearly not the case. As previously described, 40 kJ is sufficient to "burn" a 1 cm hole through a man. Modern lithium ion batteries can store on the order of 500 kJ/kg, and are not particularly dangerous (yes, some models from earlier this decade would on rare occasions malfunction spectacularly and catch on fire. This has largely been corrected. Nothing can be foolproof, but Li:ion batteries do not seem appreciably more dangerous than gunpowder or primers). So, if you assume a laser 30% efficient at turning electricity into beam energy (typical of modern diode pumped solid state lasers), a Li:ion battery pack with a mass of a bit over 1 kg would be able to store enough energy for 4 shots. A 10 kg pack (probably worn as a backpack) would be able to store 40 shots. You would probably use the battery to charge up an electrolytic capacitor in order to get enough power to drive the high power, millisecond pulse, but that's quite doable, and electrolytic capacitors are not all that dangerous either. This is a bit limited for a military weapon, but it is quite likely that both laser efficiency and the specific energy (energy per mass) of consumer batteries will continue to increase in the coming decades.

If we use the "blaster" or "PIKL" style laser, it only takes about 1 kJ to hole a man, and a 1 kg Li:ion pack could store 150 shots.

Raymond said...

@Luke:

I'm curious how you got the 4.9 cm RHA number - I'd think a tungsten carbide long-rod penetrator would have a sufficiently greater density than RHA steel to be in the ~6 cm range at hypersonic velocity (by Newtonian HV impact approximation). Obviously, I am not a ballistics expert (IANABE), so this question is neither sarcastic nor rhetorical.

As for lasers, I suspect that small arm sizing would prevent mirror diameters larger than 10 cm or so, which even in the green wavelengths would limit (by diffraction) the effective range to 200-300 m. Not to say it wouldn't be quite the weapon in close quarters, but I think railguns would have a substantial range advantage.

Luke said...

Bryon:

Lasers can use lenses instead of mirrors. For handheld lasers, you will probably see them using lenses for the same reason small consumer optics (like SLR cameras) use lenses while large consumer optics (like amateur telescopes) use mirrors. Lenses are compact and rugged.

There is no upper limit to the efficiency of lasers (well, up to 100%). High powered diode lasers have been demonstrated at about 65% efficiency. Some theoretical microscale lasers have achieved up to 99% efficiency under (extremely constrained) laboratory conditions. On the other hand, coilguns are linear electric motors, and brushless electric motors have demonstrated 95% efficiencies in mass produced consumer electronics, which compares quite favorably to the 30% efficiency of mass produced machining lasers.

It is not clear at this point whether coilguns will be more or less bulky than lasers. Modern coilguns are disappointingly low velocity, and modern lasers are disappointingly poor at antipersonnel roles.

Thucydides said...

A lot of comments so a few general points:

1. Powered exoskeletons are being developed for logistical reasons, not combat. If a soldier can carry a weeks worth of consumables on the march, then he is less tied to roads, helicopters or other vulnerable links in the logistics train. RAH set the stage for military SF writers, but professionals talk about logistics...

2. Most prototype exoskeletons are rather form fitting. A set I saw at the AUSA convention in 2006 looked a bit like a backpack combined with the sort of leg braces used by polio victims. Powered by several Lithium-ion batteries from laptop computers, it still allowed the demonstrator to run down the hall after a friend and I had placed 175lbs of barbell plates in the backpack.

3. A form fitting suit like that combined with boots and gloves with the sticking power of Gecko limbs would overcome most objections to stair climbing. I would not want to go up stairs in an urban fight anyway.

4. Snipers can be used with great effect against troops in fortifications, snipers were justly feared in WW I, when most troops were hidden in trenches. This goes back centuries, people were sniped from barricades and walls with bows and arrows long before the advent of gunpowder weapons.

5. Precision weaponry and high fidelity sensors will certainly be able to distinguish targets down to a very small size. "Fire Ant" warfare is a metaphor for robots and drones working in cooperative swarms, initially they will probably be all sizes, but gradually it will become clear only smaller and smaller ones will be able to survive the "screening" process. In a century they may indeed be the size of ants (or large spiders). Worse yet will be the ones the size of bacteria or virus particles.

6. Transporting tanks and troops is a bit pointless in interstellar war anyway. If the troops arrive on a primitive planet, there is no infrastructure to support tanks and mechanized forces. If the planet has that sort of infrastructure, they can also build a lot more tanks and guns then you can bring to the fight. The clever adversary might invite an invasion to take advantage of the invading combat engineers building a network of roads and refineries...

7 Fights between peer or near peer competitors will not go the way either party expects; I imagine the start of WW IV (WWIII being the Cold War) being something like the start of WW I; lots of dashing manouevre then everyone gets bogged down, with the fight devolving into unexpected directions and desperate schemes floated to break the stalemate.

Luke said...

Milo:

Although I'll point out (woo! nitpicking!) this could be problematic if your laser has a visible beam the enemy can use to trace your sniper. Yes, in vacuum lasers are invisible, but air causes a little scattering and misty air causes a lot of scattering. You could deal with this by using invisible wavelengths (even if some enemies have sightscopes, not all of them will) or just by only deploying those snipers on clear days, which is a pretty sensible idea for snipers anyway.

Visible light undergoes Rayleigh scattering - this is why the sky is blue. A visible light laser weapon will produce a beam sufficiently intense that Rayleigh scattering will make the beam quite visible to the naked eye even on a sunny day in perfectly clear air.

The situation with near infrared light is a bit more complex. I recently had an involved discussion with other very smart people on another forum. In perfectly clean air, the beam will not be visible of course. However, depending on the amount of dust in the air, the dust that is heated to incandescence may make the beam faintly visible under conditions of low light under certain atmospheric conditions (namely, lots of suspended particulates) and larger particles such as lint or pollen in the 0.1 mm to 1 mm range may noticeably sparkle very briefly as they are incinerated.

It is not difficult, however, to make sensors that can let you see near infrared light. Even night vision goggles will do this (although I am unsure if they will be blinded in daylight). These will easily see the Rayleigh and Mie scattering from a near infrared beam.

Luke said...

Milo:

Meanwhile I note you don't mention the one disadvantage of rechargable batteries I actually agree with: namely that recharging a battery takes much longer than replacing a magazine. This could, of course, be dealt with by having spare batteries and recharging one while the other is installed in your gun.

In general, you can recharge a battery or capacitor in about the same time it takes to discharge it. If you can discharge the battery fast enough to be useful over tactical time scales, you can probably recharge it fast enough as well - assuming you have a generator or wall-plug with enough power.

Luke said...

Reading my way down the list ...

Tony:

Now, the problem with electrical energy stored in chemical form is that when you go to use it, you're trying to take it out as raw electrons, moving at a significant fraction of c, not through a mediating expansion of gaseous combustion byproducts, moving a few thousand feet a second. The way to make a chemical storage medium do that is to get it and keep it in an excited state, ready to liberate all of its spare electrons nearly all at once. IOW, it's not a combination of chemicals ready to burn, it's a single chemical, or mixture, already put into a state where any change in the environment will cause discharge. It's like carrying around an ongoing explosion in a magic bulb that will release the explosion in a desired way through a certain interface if you get the spell right, but which will also just blow up in your face if you do anything to upset the djinni that governs it.

You seem to have a serious misunderstanding of the way electrochemical batteries work. The chemicals in a redox battery are not in excited states. It is just that some of the chemicals (or, more typically, atoms) like electrons more than others, so the electrons diffuse from one chemical species to another until there is enough voltage built up across the terminals to pull further electrons back (or more precisely - the rate of electrons diffusing along the electrochemical gradient is balanced by the rate of electrons migrating back under the influence of the voltage).

Your description is not a particularly apt description of capacitors, either. There, you just have separated positive and negative charge sitting on their own plates without moving.

Byron said...

The problem isn't just the stairs, it's the whole structure. If the stairs in my house can't support an exoskeleton, then how many houses in the third world can on the top floor? Plus, Luke's penetration figures clearly show that the amount of armor required to resist our proposed future hand weapons is far too much for an effective exoskeleton. Though those numbers may be slightly overkill. I don't need to punch a hole clean through a person to kill him.
If I have enough power to recharge a battery on a tactical time scale, why don't I just skip the battery entirely?
Also, won't the laser's diffraction still be somewhat limited by the size of the lens?

Milo said...

Luke:

"First, lasers are likely to be extremely accurate."

...Which is what I said?


"That 40 kJ beam will drill a 1 cm hole through 3 cm of steel - much better armor penetration than an armor piercing 7.62 NATO round."

No surprise there, given that a 7.62mm NATO round carries about 3.5 kJ.

40 kJ strikes me as impractically high for a single shot from a hand weapon.


"The performance of this PIKL is rather more speculative than that of the aforementioned heat ray, but the physics of explosions excavating craters is well known. Using values of material strength appropriate to ballistics gelatin, I estimate that it will take about half a kJ total for a pulse train to blast a hole through a person."

That's a much more reasonable energy content for a hand weapon.


"so long as you can maintain a tight enough focus that your laser point of focus is smaller than the crater produced by a point explosion with the energy of one pulse, the volume of armor removed per pulse is proportional to the armor's material strength"

Umm... proportional, not inversely proportional? So the stronger your armor, the easier I can drill through it?


"I estimate that a beam composed of 50 pulses of 10 J each spaced 20 microseconds apart and focused into a 1 mm spot size can blast all the way through a person and then some, and can blast through about 4.5 cm of armor steel (RHA)."

Really? Wow.

If you can pull that off, kinetic weapons are going to be pretty much obsolete. You're doing significantly more damage with significantly less energy.



Also, hey, you're that person who wrote this site, right? You seem to be pretty knowledgeable about lasers. Do you have any suggestions on my questions about transparent laser armor or cloud-penetrating frequencies?

Milo said...

Thucydides:

"A form fitting suit like that combined with boots and gloves with the sticking power of Gecko limbs would overcome most objections to stair climbing. I would not want to go up stairs in an urban fight anyway."

Gecko glue just for climbing stairs? That strikes me as clumsy.

And climbing stairs is important if there are enemies on the floor above you, and you don't want to (or don't have the fire support to) just bring down the whole house.


"Snipers can be used with great effect against troops in fortifications,"

Sentries and people shooting at you through murder holes, maybe. A drone operator who has no need to interact with the outside world in any way except for his radio equipment? No.


"Precision weaponry and high fidelity sensors will certainly be able to distinguish targets down to a very small size."

This would discourage small drones, not encourage it as you suggest. If targetting is so accurate that your 1 cm drone isn't significantly harder to hit than my 200 cm drone, then what's the point of using 1 cm drones?


"initially they will probably be all sizes, but gradually it will become clear only smaller and smaller ones will be able to survive the "screening" process"

Again, I don't think fire ants are going to be that good. Various pieces of equipment can only be miniaturized so much, and start to lose efficiency if you try to scale them down further. For one example, larger drones can carry armor, and due to the square-cube law, the larger you are, the less of your total mass needs to be spent on armor to achieve a given level of protection.


"The clever adversary might invite an invasion to take advantage of the invading combat engineers building a network of roads and refineries..."

Heh. That sounds like an amusing plot premise for a more comedic story :)


"I imagine the start of WW IV being something like the start of WW I"

That depends on the technology in use. I have more to say on this, but that'll have to wait for another post...

Luke said...

Raymond:

I'm curious how you got the 4.9 cm RHA number - I'd think a tungsten carbide long-rod penetrator would have a sufficiently greater density than RHA steel to be in the ~6 cm range at hypersonic velocity (by Newtonian HV impact approximation). Obviously, I am not a ballistics expert (IANABE), so this question is neither sarcastic nor rhetorical.

The full theory was developed by A. Tate and published as "A Theory for the Deceleration of Long Rods After Impact" in J. Mech. Phys. Solids, 1967, Vol. 15, pp 387 to 399, and was further elaborated in "Further Results in the Theory of Long Rod Penetration" in J. Mech. Phys. Solids, 1969, Vol. 17, pp. 141 to 150. It unfortunately involves taking a numerical integral, so it is not easy to put in a compact format. I've got a nifty c++ program on my home computer that I coded up off of Tate's papers that I use to predict these things.

However, we can look at various limiting cases. The Tate theory basically takes a hydrodynamic theory and adds a term describing the material strength of the projectile and of the armor. In hydrodynamic penetration, the dynamic pressures exceed the material strengths by such an extent that they are irrelevant. Rather, the front of the penetrating jet (the hydrodynamic jet corresponds to the projectile) splashes against the fluid it is impacting, pushing aside a cavity. As the jet continues forward, its front is continually eroded away as it splashes to the side, while it burrows deeper and deeper into the target material. The erosion of the front of the jet is what distinguishes hydrodynamic penetration from the Newtonian impact depth, which assumes a non-deforming projectile (among other things - hydrodynamic theory is based on energy exchange while Newton's approximation for the impact depth is based on momentum exchange). In the limit where hydrodynamic theory is applicable, you get a rather odd result that the penetration depth is independent of the jet velocity 9much like Newton's approximation, actually). Rather, the depth of penetration is equal to the length of the jet times the square root of the ratio of the jet density to the target density. Compare this to Newton's approximation, where the penetration depth is the length of the projectile times the ratio of the densities without taking a square root. Tungsten carbide has a density of about 12 g/cm^3, while RHA has about 7.8 g/cm^3. So Newton predicts that a WC projectile will penetrate about 1.5 times its length into RHA, while hydrodynamic theory predicts a penetration of 1.25 times the projectile length.

Now when we add material strength into the picture, things get more complicated. When the projectile is exactly as strong as the armor, or if it is weaker, penetration rises asymptotically from zero to the hydrodynamic limit. However, when the projectile is stronger than the armor, the penetration overshoots the asymptotic limit as the velocity rises before settling back down toward the limit at yet higher speeds. For tungsten carbide on RHA, the maximum penetration is at 2280 m/s where the projectile will penetrate 1.40 times its length - going faster or slower will decrease the penetration. (I seem to have mis-posted previously, it appears I used the value for a composite tungsten carbide-tungsten penetrator with 16 g/cm^3 density for the penetration of the coilgun projectile, which has a maximum at 2000 m/s at 1.63 times the length - a pure tungsten carbide projectile will only penetrate 4.2 cm. However, since the last cm or so of a 3 cm WC projectile at 2 km/s will be traveling much slower than the initial impact velocity when it gets eroded down to that length, and in fact the last 6 mm will be unharmed, you might as well make the back out of tungsten for its increased density.

Milo said...

My observation is that the character of combat - and the politics that preceeds it - depends strongly on the relative strengths of offensive and defensive technologies.

When defensive technology is superior to offensive technology, this makes people more willing to pick fights because they know they'll probably have advance warning if they start to fare poorly, and be able to safely retreat (while shaking their fist and yelling "I'll be back!"). It also tends to have those fights be dragged-out slugging matches, with the sides gradually depleting each others' abilities, while trying to control their own damage and cope with their losses. In extreme cases, primitive tribes still practice endemic warfare, where you nearly constantly go to war with your neighbors as a sporting event, because mortal combat is so safe that you can essentially pick a fight anytime you feel like it at little cost.

By contrast, when offensive technology is superior to defensive technology, this discourages people from picking fights because even someone in a superior position risks dying to one lucky shot. When people do fight, they'll try to avoid standup fights and try to get the first shot. Often the battle will go from "no-one appears to have accomplished anything of note" to "one of the sides is dead" almost instantly. (Compare Napoleonic musketmen tactics, which are about forming lines to shoot at each other in grand field battles, with modern riflemen tactics, which are about sneaking around in camouflage because you'll die the moment an enemy sees you.) In extreme cases, where offensive technology is vastly superior to defensive technology, you get Mutually Assured Destruction where no-one ever fights because on the off-chance that someone is stupid enough to fight, everyone dies. But the rule isn't limited to that, and it isn't anything really new - for example, I recall reading about how the introduction of the rapier led to a decrease in violent crime, even though the violent criminals had just as much access to rapiers as honest citizens.

So defensive technology superiority both encourages people to fight, and makes those fights more fun and heroic when they happen. (Of course, it's a sliding scale, and you can overdo it. Too much defensive technology, and fights take so long that readers get bored.)

As a test of the idea, consider how these rules would be applied if there existed a point defense weapon that could readily kill anything in open combat, but only at relatively short range. Since the weapon is so deadly (offensive superiority), people on a tactical scale will be very reluctant to attack each other, and so will mostly glare at each other from across a safe distance and entrenched positions. But this inability to get through the enemy's positions means that on a strategic scale, defense is actually superior, so governments will be happy to keep declaring war on each other and sending more soldiers to dig trenches all over the place, leading to an ever more costly stalemate that isn't actually accomplishing anything for either side. This will continue until a defense is developed that can stand up to the superweapon. You may have noticed by now that this isn't a made-up example - this is in fact exactly what happened in World War 1, vindicating my theory (the weapon being the machine gun, and the eventual countermeasure being the tank).

This is also why projects to develop anti-nuke defenses were so controversial during the Cold War (and now). If anyone actually succeeded in developing a reasonably reliable anti-ballistic-missile system, then they would then lose their incentive to avoid starting a global thermonuclear war.

On a completely different note, this means you may want to rethink those stories about "pacifist societies" who have mastered purely defensive warfare...

Milo said...

Over the last centuries, though, offensive technology has been increasing faster than defensive technology. And this trend seems to have held consistently, refusing to stop until the balance of technology tilted all the way over to the MAD side. But even ignoring nukes, our warfare is extremely offensive in nature - infantry die to basically anything, missile ships have been described as eggshells wielding sledgehammers, and planes crash and burn when they sustain relatively minor damage to their airframe. Nukes are just the capstone on an already bad situation.

Defensive technology has increased too - no bronze-age chariot could ever stand up to some of the stuff that Chobham tank armor now shrugs off, and atlatls make pretty poor point defenses against ICBMs - but it just hasn't increased fast enough.

But the thing is, you can't un-invent a technology once it's invented. The offensive technologies we have now are here to stay. All we can do is make up new defensive technologies that render them less relevant. Thus to get the fights we want, in space or elsewhere, a massive breakthrough in defense is needed, while offensive technology for some reason starts stagnating. (And even then, even if in your hypothetical future you decide that nukes aren't much use because nations can readily intercept any nuke that's fired at them, it's still the case that nukes will be able to kill anything that you didn't make an active effort to protect. This is important, because those defenses then have to be practically-perfect-until-destroyed-in-protracted-battle, which limits storytelling options. If, for example, you have the heroic commandos use some cunning trick to sneak onto the enemy planet bypassing the normal defenses, then you should realize that someone could also smuggle in a nuke using the same method.)

Luke said...

Milo:

Which is what I said?

Sorry, I must have missed that. Reading through these rather quickly ...

No surprise there, given that a 7.62mm NATO round carries about 3.5 kJ.

Except that they work in fundamentally different ways. A bullet causes damage through mechanical methods, the "heat ray" laser uses thermal methods. You can't just compare energy, since not all energy is equivalent.

40 kJ strikes me as impractically high for a single shot from a hand weapon.

Perhaps it is, but it is always tricky to try to anticipate future technological devlopments. 40 kJ might not be impractical for our grandchildren. Or it might be. Only time will tell.

Umm... proportional, not inversely proportional? So the stronger your armor, the easier I can drill through it?

Good catch. Yes, the volume removed is inversely proportional to the material strength.

Really? Wow.

If you can pull that off, kinetic weapons are going to be pretty much obsolete. You're doing significantly more damage with significantly less energy.


Well, that's my prediction based on the best physical models I could find through research or come up with on my own. These things are kind of hard to experimentally test, especially on the budget a researcher at a national lab brings home (now if I could get a government grant to test this, that woudl be another story).

Also, hey, you're that person who wrote this site, right?

Yup.

Do you have any suggestions on my questions about transparent laser armor or cloud-penetrating frequencies?

For transparent laser armor - if you have a high strength transparent material it should work well against "blaster" type lasers. The pulses are so intense that linear optical properties like transparency cease to be relevant. The leading edge of the pulse will be so intense as to rip electrons directly off the atoms at the surface, forming a plasma which absorbs the rest of the pulse. The thin layer of solid density plasma the explodes just as it woudl against a non-transparent material.

For "heat ray" style armors, it is more difficult. In principle you could dope it with some chemical that undergoes a photochemical reaction, turning opaque when exposed to sufficiently intense light of the right frequency. Alternately, you could have the transparency modulated electrically, and have a sensor that detects if a laser is incident on the armor. You might also make the armor out of something that photo-decomposes into a charred, blackened mess.

The main problem I see is that optimal armor materials are probably not likely to be transparent, and the more things you try to simultaneously optimize for, the worse the armor protection will be. You might get something like glassy diamond or something that is both very strong and very transparent, but it will probably stay transparent when a heat ray is incident upon it.

As far as cloud penetrating frequencies, this is probably not particularly practical. As long as the wavelength of the laser light is shorter than the dimensions of the cloud particulates, the rate of scattering is mostly indepenent of frequency. Since cloud particles tend to be in the 3 to 100 micron region, and practical laser weapon wavelengths tend to be in the 1 micron range (possibly as long as 3 microns, but the longer the wavelength the wider the focus and thus the less intensity and penetration of the laser, and this is still shorter than most cloud particles).

You might be able to tune the laser to a frequency that is strongly absorbed by the cloud particles, so that the laser "burns" them out of the way rather than scattering off them without heating them. This is easier closer to the target, because laser beams start out wide and focus to a narrow beam at the target.

Luke said...

Raymond:

Forgot about this part ...

As for lasers, I suspect that small arm sizing would prevent mirror diameters larger than 10 cm or so, which even in the green wavelengths would limit (by diffraction) the effective range to 200-300 m. Not to say it wouldn't be quite the weapon in close quarters, but I think railguns would have a substantial range advantage.

That's basically correct. For optimum penetration of RHA, you want to to focus the beam to a spot of 3.8 mm diameter or less (according to my calculations, which are probably reported to one more significant figure than is strictly justified). A 10 cm lens focusing a 1 micron wavelength beam at with about 1.2 times the diffraction limit (which is pretty good) would put most of the beam power within a 3.8 mm spot at a distance of 264 meters, so you could expect good armor penetration out to here. Better armors will have smaller craters blown out of them, so they will need still smaller spot sizes and thus shorter ranges. Some nanostructured carbon armor with the strength 9but not the brittleness) of diamond, for example, would require a 2 mm spot size for optimum penetration (of course, it would also offer superior performance against coilguns as well).

Milo said...

...But that isn't SURFACE warfare, eh? Hmm. Well, surface warfare has an even more fundamental problem: you're vulnerable to aerial bombardment, vulnerable to orbital bombardment, vulnerable to ballistic missiles (surface warfare is the one theater where nukes are already proven to succesfully threaten MAD). Even before considering how you do against other surface forces, you're pretty vulnerable. And when we do consider how you do against surface forces... well, Luke's numbers suggest that an infantry laser gun would be readily capable of penetrating the best feasible infantry armors (even powered armors), and lasers don't miss much... provided you're in their line of sight. Concealment is going to be important here. Just like today.

If only or mainly short-range weapons are used, then a WW1-like trench warfare scenario may develop, as Thucydides suggests. But I don't think modern forces are that easy to hold off with some trenches. We just have too many long-range weapons and travel options.

Milo said...

Argh. First part of post got cut off again.


Unfortunately, spaceships are looking to be pretty fragile. Mass limits make it hard for them to carry armor, they depend on a bunch of delicate machinery, and if you poke a hole in the hull the air leaks out. Meanwhile, most authors want offensive technology to get boosted at least a little, to enable zappy ship-carving lasers. (And kinetics onboard ships aren't usually launched with gunpowder, either.)

I think most science fiction writers realize this on some level, which is why force fields are so popular, despite having even less scientific basis than FTL travel, and not being in themselves necessary to get into space. Our offensive technology has gotten good enough that we appear to need magic for defensive technology to catch up.

But do we?

Once spaceship engines are good enough for interplanetary colonization and warfare to become practical, we may be able to afford putting more armor on the rockets. Lighter armor might also be developed, with carbon nanostuff currently being "just beyond the horizon" - granted many technologies seem to stay there for annoyingly long times, but it's more plausible near-future tech than fusion energy... not to mention lasers, rapid space travel, and terraforming. (Actually, boron carbide offers pretty good kinetic, laser, and neutron protection for little mass with existing technology, but it's rather expensive. Modern armies do use it where mass is at a premium, such as infantry armor and some airplanes, but armoring an entire ship with the stuff is bound to hurt your wallet.)

Mobility also isn't useless. Lasers can't be dodged per se (unless you jink constantly, which will rapidly burn through your delta-vee), but kinetic slugs can. And mobility is still useful against lasers in as far as that if you move whenever you get hit, it's harder to keep a continuous beam focussed on you, forcing the enemy to use a more powerful laser that can penetrate your armor in a short pulse - or else fail to do more than cosmetic scarring.

Point defenses can zap missiles (kinetic or explosive) and shells, and might possibly be useful against unguided slugs, although I'm not sure how much. (A weapon that can zap armor can probably also damage kinetics, but even if you shoot a slug in half, then both halves will still continue to hit you with the same combined kinetic energy as before, albeit less concentrated. You could keep shooting them into finer dust until they're too spread out to do much damage, but that rapidly turns into trying to play a game of Asteroids.)

Stupendous range actually helps here. It makes kinetics easier to dodge and reduces the damage by lasers, gives you more time to zap incoming missiles, and depending on weapons technology, possibly forcing enemies to use longer-range weapons despite those being weaker.

Milo said...

Compartmentalization can help against hull breaches, as can putting your habitation modules further inside the ship, implicitly protected by all your unpressurized equipment. They'll still suffer explosive decompression at least as easily as a ship will sink, but future spaceships might not crash simply from snapping a single fragile component, as with airplanes.

And the big plot-killer, nukes, are explained on the Atomic Rockets website to be less effective in space than on ground. How convenient!

But is all of this enough? They're still pressurized cans in an inherently hostile environment, and fighting with projected continuations of present-day technology. It's not like we don't have armor for sea ships - we've used it before during the ironclad era - but it proved to not be enough. How do you reverse the trend of "anything can easily kill anything else"? Especially when you remember about those pesky fragile radiators...


And now the part about how that isn't surface warfare.

Milo said...

Luke:

"Except that they work in fundamentally different ways. A bullet causes damage through mechanical methods, the "heat ray" laser uses thermal methods. You can't just compare energy, since not all energy is equivalent."

True, but I'm just saying that claiming your 40 kJ laser does more damage than a 3.5 kJ bullet isn't very impressive. It doesn't show lasers are better than bullets, at all.


"The pulses are so intense that linear optical properties like transparency cease to be relevant."

That would be rather bad for the shooter, since you still want to be able to shoot through air without losing too much energy to it. (Yes, I know this issue has been examined by people working in the field, and is called blooming. I was kinda ignoring it before...)

I was expecting that while a fraction of a laser's energy would be absorbed even by a transparent material (since it isn't completely 100% transparent), and for a sufficiently large laser pulse even this small fraction would still do meaningful damage, the majority of the energy would still make it through. Am I completely misunderstanding things?


"The main problem I see is that optimal armor materials are probably not likely to be transparent, and the more things you try to simultaneously optimize for, the worse the armor protection will be."

That already applies to bulletproof glass today, but it's still useful for vehicle windows, helmet visors, and the like. People are willing to compromise to some degree on this front, the question is how much they'll have to compromise.


"You might be able to tune the laser to a frequency that is strongly absorbed by the cloud particles, so that the laser "burns" them out of the way rather than scattering off them without heating them."

Hmm. Now that's an interesting idea. What wavelengths would you need for that, and how much energy would you waste on the cloud?


"This is easier closer to the target, because laser beams start out wide and focus to a narrow beam at the target."

Cloud penetration (as opposed to low-hanging fog penetration, although they're probably done with the same techniques) is mainly important for surface-to-orbit and orbit-to-surface weapons, which were what I was thinking about when I first asked the question. Thus the cloud will typically be kilometers above the surface platform, but at least a hundred kilometers below the orbital platform. So if I understand you correctly (I'm not sure), this means orbit-to-surface would have an easier time burning through clouds than surface-to-orbit? Though I doubt either will have an easy time.

You could, of course, try to just have your sub swim out to somewhere that doesn't have a cloud in the way.

Luke said...

Milo:

That would be rather bad for the shooter, since you still want to be able to shoot through air without losing too much energy to it. (Yes, I know this issue has been examined by people working in the field, and is called blooming. I was kinda ignoring it before...)

The interactions of lasers with the air are rather interesting. You do have a range of powers and intensities where you can have "blaster" pulses that go through a lot of air. Past a certain power, the pulses start to self-focus, which can lead to a phenomenon called filamentation, where the non-linear self focusing is canceled by defocusing of a sparse plasma core, and you get a glowing line of plasma surrounded by the radiation field of the light (which, incidentally, gest spread out in frequency to become white). Filamentation can be useful in some cases, although it would be rather visible and would likely reduce the range of the laser somewhat.

The classical problem of thermal blooming is where the laser heats the air, which then defocuses the beam. There are ways to correct this. The simplest is to use a short pulse so the air does not have time to expand. The millisecond pulses discussed so far are unlikely to suffer much from thermal blooming.

There are other non-linear phenomena which affect lasers in air. Stimulated Raman scattering will scatter laser light in other directions at lower frequencies when the laser is intense enough. Stimulated thermal Rayleigh scattering is similar, except that the scattered light is the same frequency. These tend to be more of a problem at long distances.

I was expecting that while a fraction of a laser's energy would be absorbed even by a transparent material (since it isn't completely 100% transparent), and for a sufficiently large laser pulse even this small fraction would still do meaningful damage, the majority of the energy would still make it through. Am I completely misunderstanding things?

For blasters, the intensity is so high that, as mentioned, transparency does not matter.

For heat rays, the issue is whether the damage caused by the early part of the pulse renders the material opaque or translucent, allowing it to prevent the rest of the pulse from penetrating (unless the pulse can burn its way through, of course). The portion that gets through might cause a painful surface burn but no meaningful penetration through flesh.

Luke said...

Milo (continued):

Hmm. Now that's an interesting idea. What wavelengths would you need for that, and how much energy would you waste on the cloud?

As wavelengths go from 1 to 2 microns, the distance it takes for most of the beam to be absorbed in water drops from about 1 cm to about 1 micron. So near infrared beams would seem to be pretty good at evaporating water out of the way.

With some copy and paste from a previous post on SFCONSIM-L:
a cloud has 1 to 4 kg of water per square meter per kilometer thickness, but rarely exceeds 2.5 kg/m^2 per km thickness. Considering only the heat of vaporization, it will take about 5.5 MJ to evaporate a one square meter hole through a kilometer thick cloud.

The rest of the post is here
http://games.groups.yahoo.com/group/sfconsim-l/message/66173

Cloud penetration (as opposed to low-hanging fog penetration, although they're probably done with the same techniques) is mainly important for surface-to-orbit and orbit-to-surface weapons, which were what I was thinking about when I first asked the question. Thus the cloud will typically be kilometers above the surface platform, but at least a hundred kilometers below the orbital platform. So if I understand you correctly (I'm not sure), this means orbit-to-surface would have an easier time burning through clouds than surface-to-orbit? Though I doubt either will have an easy time.

You could, of course, try to just have your sub swim out to somewhere that doesn't have a cloud in the way.


Yes, indeed. Or fly your laser plane above the clouds. Or site your laser defense station on a mountain in a desert.

Hugh said...

Today's paper had an article about the growth of megacities, something not really considered in the original post. Apparently there are already three cities/suburban areas over 20 million already, and a lot more predicted by 2050. Most of them in the developing or third world too.

How do tanks, or at least armoured cars like the Stryker, stack up against powered armour in a city? I'd think much faster tactical movement for one.

Citizen Joe said...

Re: Powered armor. Keep in mind that mass not only causes weight (which can crush the surface your standing on) but also momentum. Blimps are lighter than air, but mass several tons. While they float, they do not turn or stop very fast. Get a suit of powered armor up to a jogging speed and it will likely flip over or slide like a cat on linoleum when it goes to turn the corner.

Re: Megacities. Cities (much like the mixed orbital environment discussed previously) have civilians and soldiers in them. If you're content to destroy the entire city, that is easy. If you're trying to capture it, you lose out on a lot of weapons. That is essentially the soft squishy armor I mentioned upthread.

Casper said...

On powerarmor weight:
Remember those tournament armors for knights that needed winches to get them on horse?
There is clearly an upper limit to how heavy your armor can be. But with enough tech you can build powerarmor that’s practical on the field. As I see it, there’s room for several types: light “endurance” armor for infantry, heavy armor for shock troops who specialize in storming bunkers, etc.

Also on wet Navies- all vessels will probably be submarines. Even air carriers will only briefly surface, launch VTOL craft and submerge again until the aircraft need recovery.
How good Ortillery is against underwater targets?

Byron said...

If the powered armor only provides protection from light arms, and weighs too much for real urban mobility, then it's pointless to have as armor. I'm not claiming that exoskeletons are impractical, but merely that classical powered armor has no real purpose if it can't compete with an infantryman. The versions described here seem to have more in common with armored vehicles, while likely being more expensive, due to the stuff necessary to make it walk.
The various armor-piercing weapons described here seem to make armored vehicles obsolete. The only way to survive would be to be stealthy. However, I have to question that conclusion. The primary purpose of an infantry small arm is to kill other infantrymen. If those infantrymen become armored, then it might be made more powerful. However, adding in anti-armor capability will likely add weight, expense, and the other fun issues of high-powered lasers. Luke, how much energy, both blaster and heat ray, would it take to reliably incapacitate a man? That's what I think that small arms will use.

Rick said...

What have I let loose upon the world?

Meanwhile, welcome to another new commenter!

I think Milo makes a very important meta point. The long term trend has been toward offensive technology and killing power. By the way, the phrase 'eggshells armed with hammers' was coined by Winston Churchill to describe interwar 'Treaty' cruisers, with more armor and MUCH less firepower than modern missile cruisers.

Personal armor was the rule (for those who could afford it) in the pre-gunpowder era. Gunpowder marginalized it by the 17th century. It is making a comeback now, but from discussion here this may merely reverse the 17th century evolution of the musket.

In the 16th century the standard infantry firearm was the arquebus. Because armor could still stop an arquebus round, the musket developed as a heavy arquebus (it had the equivalent of a bipod stand) that could stop anyone.

The heavy musket was so effective that armor was abandoned - and then the musket shrank back to the dimensions of the arquebus, broadly the dimensions of assault rifles today.

If we revert to arquebus dimensions - and a soldier with an exoskeleton can handle it a lot easier - do we cancel out armor all over again? To put it another way, can reasonable power armor protect the wearer against the weapon another power armor trooper can reasonably carry and fire?


I have suggested before that war may be fundamentally obsolescent, because at the post industrial techlevel the offensive advantage is so great that you cannot protect yourself even against a weaker opponent.

Byron said...

No. A tank can't protect itself against a weapon a normal human can carry and fire. Again, the main limit to modern weapons is not inherent, it's just that they do what they need to.
And actually, I think that body armor might become obsolete in the same way as it did then. The next generation of weapons will be designed to defeat it, which renders it pointless, so it'll go away. We might still see flack jackets, but not much more.

Raymond said...

Luke:

Thanks for the ref re:penetration. The square root of the density ratio was the factor I was missing. Still, 5 cm of RHA is nothing to sneeze at.

Re: Power armor:

If the power suit can carry armor sufficient to defeat modern small arms (and, by extension, most artillery fragments) and allow a twofold-or-better increase in equipment carried, it will be standard. Period. The number of weapons useful in infantry combat but restricted to section, platoon or company level because of weight is staggering. Heavy machineguns, sniper rifles, rocket-propelled grenades, anti-armor missiles, anti-air missiles, light and medium mortars, laser designators, recon drones, thermobaric warheads, and things on (or just over the horizon) like railguns and laser point-defense systems. All of which are more useful and more important to infantry tactics than what caliber your rifle is.

Raymond said...

Casper:

Ortillery is utterly useless against submersible platforms unless you have complete and utter dominance of low orbit, and either visible-wavelength lasers in the tens or hundreds of megawatts in sufficient numbers to cover the whole surface simultaneously, or anti-submarine weapons you can fling from orbit within scant minutes. Submarine-launched ASATs are a bigger threat to spacecraft in low orbit than the other way around. They can surface just long enough to lob an ASAT up, then dive again, and if they have sufficient delta-v I can launch them against targets which aren't even overhead to see the launch flash.

I tend to think this will render low orbit a no-mans-land for most or all of a campaign (start up your WWI references, ladies and gentlemen). In fact, I think it might be the primary objective of the invasion campaign to eliminate the enemy's ASAT capability, at which point the large-scale orbital bombardment can commence, and any sensible planet will surrender before that starts.

Milo and Luke:

Fixed laser installations are dead. Gone. If I'm invading your planet, I hit those from high orbit with kinetics launched when the site is facing away. Repeatedly. I don't come any closer until they are dealt with. I can spot them from geosynch, and they might even be worthy of nukes, not to mention the number and size of kinetics I can launch at nominal delta-v. Airborne platforms are more trouble, but atmospheric fighters designed for reentry can hunt them down. I'm worried about the subs (see above).

Now that I mention it, why has no one brought up aircraft? How do you go about bringing your air support, how do you fuel it, where do you stage it from? Assuming we don't have magical SSTO spaceplanes, I'd think you'd need to capture/create an airbase on the surface to support your power-armored troops...

Raymond said...

Rick:

First, stupendous blog. Thanks.

Personal armor for most the 20th century was about shrapnel, not small arms, because that was by far the biggest threat vector. From steel helmets to flak jackets, the armor was there to reduce casualties from artillery (mortars especially). It's only been the last generation or two of personal armor which has stood a chance against rifle-caliber rounds. You may have heard something about the dust-up surrounding Dragon Skin armor and its comparisons to the Interceptor armor officially sanctioned; wherever the truth sits in that catfight, the fact that soldiers are currently wearing armor which can repeatedly defeat common rifle calibers shows the armor balance may be swinging the other way for the first time in centuries.

Byron said...

No, the armor balance is not swinging. The situation today is exactly the same as it was in the 17th century. Modern small arms have the capabilities they do because of the environment they were designed in. All modern militaries use weapons designed during the 70s or earlier. At that time, body armor was not common, so the rounds were not designed to penetrate it. A firearms designer today could undoubtedly design a round that could penetrate most modern body armor. They already have. The FN 5.7 mm round can pierce armor resistant to .44 Magnum rounds. If a war broke out between two countries that both had armaments industries and used armor, both would rush weapons that penetrated armor into production. At that point the armor would disappear or go back to flack jacket level, as it doesn't add to survivability, and it's heavy. Nobody is introducing new small arms today, because none of the countries with armaments industries are likely to find themselves at war with each other soon, and insurgents and third-world armies can't get these advanced weapons.
The exoskeletons could increase the capacity of the soldier, but if they did, wouldn't the new weapons go right through the armor? Wouldn't it be better to add mobility (go up stairs) or more weapons, not armor that is useless against any "modern" weapons? The only time powered armor (or any armor, for that matter) really works is when there is a technological mismatch between the forces. If one side can produce armor that the other side's hand weapons can't penetrate, then it makes sense. However, we can produce weapons that will penetrate modern armor, and probably issue them to our normal infantry. It's just that our enemies don't have them.

Raymond said...

Byron:

The FN 5.7mm round is a nice idea, but we're talking rifle rounds here, not pistols. Interceptor SAPI plates will stop 7.62x39mm AK-47 rounds cold. IIRC it will stop 5.45mm AK-74 rounds as well, and possibly FMJ 7.62x51mm NATO. DoJ Level III armor is for pistol protection, and yes, five-seven pistols and submachineguns were designed to penetrate it.

Armor-piercing rifle rounds, however, have limitations inherent to the caliber and velocity, because they are designed to be fired rapidly, on the move and without mounts. The velocity of shoulder arms (sub-sniper rifle) is too low to reach the realm Luke and I were discussing previously, where hydrodynamic principles take over.

Also, ceramic composite plates in modern infantry armor have substantially reduced shrapnel casualty rates, and shrapnel is the biggest killer of infantry. Even in guerrilla wars (see IEDs). Flak jacket level protection is for REMFs. The effectiveness of high explosives is set by gas expansion rates, and we have already reached many of those plateaus, so it's not like artillery is going to become an order of magnitude more penetrating anytime soon.

Byron said...

Yes. But we have not seen any rifle contemporaries to the 5.7mm round. The protections are against ball rounds. AP is another game altogether. The various armors under discussion are modern. They've come about in the past few years. An M16, or an AK-47 is not modern. The 5.56mm NATO was designed in the 50s. The 7.62R dates to 1943. We're putting modern defenses against half-century old rounds. The problem is that we have no real modern rifle rounds to compare the armor against.
Here's my challenge. If anyone knows why, if faced with powered armor resistant to, say, 7.62 NATO ball, a military wouldn't swap it's weapons for something with better armor-penetrating capability, explain it to me. And yes, that can be armored against, but then the suit gets heavier and less mobile. The question is the general purpose of powered armor. What is it there for? Is it protection against bullets? I can't see that working? Mobility and carrying capacity? That might work, along with fragment protection. Is it to enhance the infantry? Then it has to be capable of infantry missions, like clearing a building, and not falling through into the basement.

Raymond said...

DoJ type IV protection specifies stopping .30-06 armor piercing. The newer Interceptor ESAPI plates are rated at type IV with the vest providing spall protection. We've got it already. It's just too heavy to cover the whole body.

The reason the 5.7mm FN round was so effective compared to other pistols is the nature of the armor used - being 40-odd layers of kevlar weave, wide, slow ball rounds (used in pistols) would disperse their energy across a wider area. Narrower FMJ rounds like the five-seven would push the kevlar threads out of the way.

Defeating SAPI plates and their ilk means pushing aside solid boron carbide. This is not something easily done, if at all, at the calibers and velocities reasonably within the small-arms category.

Milo said...

Luke:

"The millisecond pulses discussed so far are unlikely to suffer much from thermal blooming."

Okay, so what kind of transparent armor does protect against these millisecond pulses? And why does the armor work when air doesn't?


Also, something else I've noticed. We seem to be mostly talking about lasers vs steel armor, but steel is actually pretty poor material for using to armor against lasers. What about boron carbide (in use as body armor today) or carbon nanostuff?



Hugh:

"How do tanks, or at least armoured cars like the Stryker, stack up against powered armour in a city? I'd think much faster tactical movement for one."

As long as you're duking it out in tidy streets, cars will work fine. However if you're trying to make your way through streets cluttered with rubble (including vehicle wreckage), or if you need to be fighting in a building, then cars are unsuitable. Cars are also poor at turning around and going in the other direction quickly, but there are bound to be solutions to that which are easier than bipedal vehicles.

Milo said...

Casper:

"Remember those tournament armors for knights that needed winches to get them on horse?"

Yes. I'll also remind people that those were only used for tournaments, and that in actual warfare knights used more sensible armor that were actually pretty mobile.

Anyway, power armor could in principle be made heavier than its operator can effectively move around in without power... in fact, that's rather the point.

The real problem comes when your armor is so heavy that it crushes your horse.


"As I see it, there’s room for several types: light "endurance" armor for infantry, heavy armor for shock troops who specialize in storming bunkers, etc."

History suggests that what will happen is governments will try all of the above, but only some of them will prove to actually be cost-effective and the rest will be discontinued.

The history of warfare is littered with cool weapons that never got out of the prototype stage because, despite looking cool, they weren't actually that useful.


"Also on wet Navies- all vessels will probably be submarines. Even air carriers will only briefly surface, launch VTOL craft and submerge again until the aircraft need recovery."

Conventional navies are only really useful if you have a Balkanized planet. Ships, except maybe some really small ones, are simply too heavy to carry across space, and space invaders would hardly need them anyway when they can land their forces directly on land without needing to bother controlling the sea.

If you have a Balkanized planet, then while I agree submarines would take center stage (they seem to be doing so today), there will probably be some place for surface warships as a cheaper craft. If you have a unified planet, then the only ships worth having are anti-orbit submarines (descended from today's ballistic missile submarines, now armed with either missiles or lasers), coast guard type vessels (which would be more like a SWAT team at sea), and transports. Neither of the latter two are really warships, despite their ability to be used in war.

Aircraft carriers may well become obsolete if airplanes develop better range. Those awesome fusion reactors you're using to reach the gas giants could probably also allow planes to stay in the air indefinitely. You'd just ship materials for constructing an impromptu airstrip at the destination, then have your planes get there on their own.

Space-based aircraft carriers launching amphibious space/air planes (optimized for atmospheric agility but also capable of escape and reentry on their own) might have a place.


"How good Ortillery is against underwater targets?"

Pretty poor. Your projectile has to deorbit, and has to stay on course across a long distance through three different mediums (vacuum, atmosphere, water), all the while not even being able to see the target. You'd fare better if you're fast-acting enough to hit the sub as it surfaces, when it's about as vulnerable as any other surface target.

This might well end up providing the justification for landing actual surface troops to try to engage the enemy's anti-orbit submarines. Woo, sub-on-sub warfare IN SPACE! Or else a bunch of space-and-air-dropped homing torpedos.



Byron:

"Luke, how much energy, both blaster and heat ray, would it take to reliably incapacitate a man? That's what I think that small arms will use."

Small arms will use the amount of energy needed to reliably incapacitate a man, plus the amount of energy needed to reliably penetrate the amount of armor commonly carried by men on this battlefield.

For that matter, heavy arms will use the amount of energy needed to reliable incapacitate a vehicle, plus the amount of energy needed to reliably penetrate the amount of armor commonly carried by vehicles on this battlefield.

Milo said...

Rick:

"What have I let loose upon the world?"

Heh. As much as I like big explosions, I do find it a little depressing that the warfare post is attracting so many more responses than all the other ones.



Byron:

"A tank can't protect itself against a weapon a normal human can carry and fire."

These are grenades and rocket launchers, of which you have very limited ammunition. No sensible army would tell its soldiers to start shooting antitank weapons at enemy infantry.


"The next generation of weapons will be designed to defeat it, which renders it pointless, so it'll go away."

That is assuming the armor will be measured against enemy superpowers using the next generation of weapons. What if the armor is measured against peasants with AK-47s?

It's interesting to note that today riot police are often better armored than soldiers (riot shields, etc.), both because they more often expect to go up against poorly armed attackers, and because they tend to want to avoid "shoot them before they shoot me" as a defense strategy.



Raymond:

"In fact, I think it might be the primary objective of the invasion campaign to eliminate the enemy's ASAT capability, at which point the large-scale orbital bombardment can commence, and any sensible planet will surrender before that starts."

Definitely. The question is whether eliminating the defender's surface-to-orbit capability would involve deploying hunter-killer troops to the surface while the spaceships wait at a safe distance, or whether it would involve zerging them with spaceships in low orbit and using superior numbers to overwhelm their "fortified position advantage", or a mixture of both.


"Assuming we don't have magical SSTO spaceplanes, I'd think you'd need to capture/create an airbase on the surface to support your power-armored troops..."

Yeah. If you need long-term endurance and docking into orbit for reloading is too expensive, you'd have your dropships land do-it-yourself airstrip equipment, and fuel/ammunition canisters.

Capturing a preexisting airbase is a possible shortcut but would be done opportunistically and sensible invaders won't assume they can always do this.



Byron:

"What is it there for? ... Mobility and carrying capacity? That might work, along with fragment protection."

Yeah, those are what I'm picturing. If it protects against small-caliber bullets that's a bonus, but fragment protection is good enough on its own. Mobility benefits, compared to unassisted infantry, would be in things like breaking down doors and getting through rubble.

Other benefits would be NBC protection - bonus points if you can get the powered armor to work in vacuum. (Not necessarily in zero-gravity, that's a different matter entirely and I don't think there's much justification for soldiers to be fighting there.)


"Then it has to be capable of infantry missions, like clearing a building, and not falling through into the basement."

Yeah. So what kind of mass limit would you suggest?

kedamono@mac.com said...

I think the major issue here that we're all skirting around is not how effective power armor is, or how deadly lasers are. That's not what will decide a military to build them.

It's cost. If one PA'ed soldier costs 5 normal soldiers, and the normals can field weapons that can take out the PA soldier, then power armor might not make the cut. Exoskeleton systems in the depot and fire bases might see a use, since they will be more maneuverable and portable than a fork lift. (Think of the exo-suit Ripley used in Aliens)

Lasers fall into that same category. If your MK 1 Laser Blaster costs 5 times as much as a FN SCAR, and doesn't really get you much more fire power than the ballistic weapon...

Thirdly, as I pointed out with the AK-47, how durable and GI resistant are these technologies? I did my four years in the army, and I've seen GIs do weird and stupid things to their gear. How long will your MK 1 Laser's optics last if a GI spits on them and cleans them with his t-shirt? And what happens if water get's into the MK 1's optics. Not "if", but "when".

Remember, even though the Army touts the intelligence of its recruits, (And I was one), the average GI is not the sharpest bowling ball in the bag. Any parts of your MK 1 Laser that are designated for "depot maintenance only" will be cracked open, cigarette/sand/blood/spit/whatever will be introduce to those parts, and then resealed by the GI who "Just wanted to look" at the guts of his weapon.

The more that a weapon needs to be kept in a pristine state, the more likely it's going to be dropped into a mud puddle, swished around for good measure and then immediately used by the owner to save his life.

A AK-47 will fire in this situation 60% of the time. Will your laser?

Byron said...

What I saw was that it defeated level IIIA armor at 200 m. While it doesn't specify the type of armor, I really can't see Kevlar being that loose of a weave. I'm willing to bet that, given a clean slate, a decent amount of funding, and a requirement to build an infantry rifle cartridge that could pierce Level IV protection, a competent designer could do so. The reason it hasn't happened is lack of demand.

Small arms will use the amount of energy needed to reliably incapacitate a man, plus the amount of energy needed to reliably penetrate the amount of armor commonly carried by men on this battlefield.

For that matter, heavy arms will use the amount of energy needed to reliable incapacitate a vehicle, plus the amount of energy needed to reliably penetrate the amount of armor commonly carried by vehicles on this battlefield.

I know. I just wanted numbers. My point is that A. it's unlikely that handheld weapons will be useful against vehicles, and B. that they will be able to pierce whatever armor is used by opposing infantry. It thus makes sense to keep the armor down, so as to avoid loosing mobility from it and the weapon you're hauling around to deal with it.
And the comparison with AK-47s is exactly my point. Powered armor will not be useful in an industrial war. It will be useful against people at a lower tech level, which is what happens when guerrillas face a government.
And I'm thinking no more than 500 pounds or so all up. That seems a reasonable limit.

Byron said...

Kedamano, we've somewhat already discussed that, but you raise good points. That's why I personally think that lasers will initially be used solely on vehicles and other large mounts. You can protect them more effectively, and give the crew better training in dealing with it. Plus, as was pointed out, lasers really don't live up to their promise as infantry weapons, because of range limitations.
This is something both sides seem to fall into on lasers. The pro-laser side goes on about "speed of light" and perfect accuracy and such. The anti-laser people talk about how unsuitable they are for hand weapons. Both are true, but neither really gets the point. The first operational lasers will be for artillery defense. Then air defense, and so on.

Albert said...

I agree with Byron, current armors only have sense when facing weaker opponents that cannot afford to mantain modern and ass-kicking new weapons.
Any worthy opponent will simply upgrade the arsenal and you will dump all the shiny armors in the trash and fight without underwear like Braveheart.

To me this is another telltale that most nations don't really think about serious warfare against each other anymore.
At least not with infantry, that today is more a "foreign police force", used to enforce rules (yours or theirs, so either an invasion like Iraq or a neutral police force for troubled places) on weaker nations that can't really defend against them.


Anyway, about this armor-vs-bullets contest, I would be curious to know how much ludicrous properties must have a material to be worthy of a power armor capable to stop the best weapon an infantryman can be issued with while still not so bulky/massive to let soldiers enter and fight in buildings.
(so what a powered armor is supposed to be in the first place)


Btw, I still love the concept of a nerdy guy commanding a swarm of flying drones from an armored vehicle over a macho powered-armor guy, even if both seem at the same level of unfeasibility.
Sure that there is no way to implement it?

-Albert

Raymond said...

Byron:

Most IIIA armor is kevlar, not ceramic plates, and the AP strategies used are different. No, kevlar weave isn't exactly loose, but at high velocity it might as well be when compared to high-density ceramics or heavy metals.

Also, my main point is that fragment protection is far more important than small arms, but intersects it at certain levels (such as AP rifle rounds, which have similar penetration profiles to high-velocity fragments from large artillery rounds). If the power armor can walk through a mortar barrage, or survive nearby howitzer rounds, then infantry tactics change dramatically, and the cost of the power suit doesn't matter so much (especially at the far end of an interplanetary supply line, where mass is all and zerg rushes by the invader are all but suicide).

Also also, there is a difference between handheld weapons and small arms. An antitank weapon is handheld by the broader definitions, but is very much not a small arm. If power suits allow every infantryman to pack a Javelin missile, a Stinger missile and a grenade launcher, then non-powered infantry won't even be considered, AP rounds be damned.

Milo said...

Byron:

"Plus, as was pointed out, lasers really don't live up to their promise as infantry weapons, because of range limitations."

For that matter, neither do rifles. In today's confined urban warfare, militaries are rediscovering the value of shotguns.


"The first operational lasers will be for artillery defense. Then air defense, and so on."

Yes. The question is how far the "so on" will go.

Anyway, this thread is supposed to be about surface warfare, not just infantry warfare. When I asked about lasers vs kinetics, I was also curious about not only shotguns/machines/etc., but also tanks, fighter planes, IFVs, etc. Assuming you're still using those at all. (Remember, I did point out the inapplicability of lasers for surface-to-surface artillery.) What kind of non-space vehicles do you expect to see in midfuture warfare, and what weapons would they carry?

If you have ideas for other kinds of futuristic weapons besides lasers and differently-propelled projetile weapons (like coilguns), I'm also listening. Particle beams don't strike me as too useful, though - they tend to leave too many victims alive but terminally ill. (Heavier ions would tend to do only surface penetration and so not have this issue... except the same lack of penetration also makes them incapable of penetrating air.)

As for the ever-popular-regardless-of-whether-it-makes-sense plasma weapons... well. The appeal here is that plasma is really hot and so carries lots of energy, which makes it rather intuitive that it would be painful to get hit by. (If you're merely using plasma to obtain ions for a particle beam weapon, that doesn't really count as a plasma weapon.) The question is what are you shooting at that requires so much heat to melt? Existing termite grenades can already melt steel. Plasma seems like it would be overkill against most things you're likely to ever encounter on a battlefield, and it's hard to work with.



Albert:

"At least not with infantry, that today is more a "foreign police force", used to enforce rules (yours or theirs, so either an invasion like Iraq or a neutral police force for troubled places) on weaker nations that can't really defend against them."

And this is likely to remain the main purpose of infantry in the future. Even when superpowers (on different planets) fight against each other, the real superpower combat will take place in space, and the planetary warfare will be a mopup operation by the side that won in space, with the loser now no longer being a superpower on account of having had its space fleet blasted away.


"Btw, I still love the concept of a nerdy guy commanding a swarm of flying drones from an armored vehicle over a macho powered-armor guy, even if both seem at the same level of unfeasibility. Sure that there is no way to implement it?"

There will be a place for drones on the battlefields of the future, just not as the main line of battle. Stuff like spy planes and such. Possible with use as second-tier "cheap" troops, with the idea being that you deploy actual humans as elite forces to the most troubled spots while letting robots handle the rest.

The real nerdy guys will tend to be found onboard spaceships. Surface warfare is more macho.

Byron said...

OK, maybe you have a point about penetration. Still, all that will happen if suits are immune to normal fragments is that artillery will switch to DPICM. You seem to equate power suit with armor. I'm not against all exoskeleton/power suits, but I see them more for carrying capacity than armor. However, have you been listening? There will be no conventional ground battles with interplanetary invaders. Also, why would giving every man a set of missiles make regular infantry obsolete? If I gave my men light powersuits and antimaterial rifles, which now can be used as normal rifles, they can take your guys out. It's a small arm, at least in that usage.
I can see a fall of the conventional combat airplane when laser defense becomes common. A plane can't dodge a laser, so all that will be left are low-altitude craft which are too low to track well. I can see vehicles becoming more stealthy, and less armored. They aren't able to withstand weapons, so they hide. They also serve primarily as weapons platforms. It's possible that with high-precision systems that kinetic artillery could be possible, particularly from coilguns. So much depends on the space situation. If space support is possible, then that becomes the real battlefield. If it isn't, then the battle is slightly different. There is less fire support power available, and far-back sections might be reasonably safe.

Raymond said...

Milo:

If we're talking a surface war, the invading constellation obviously isn't looking to do a fly-by nuke-and-pave. And even if the defenders' spacecraft are insufficient to intercept and defeat the attackers, I can see a lot of defensive weaponry remaining on the surface (hidden in deep bunkers or underwater) to deny the attackers low orbit. Such weapons may be insufficient to strike high-orbit targets, at which point a surface campaign becomes necessary.

I also think infantry missions (power armor or not) would be the exception, at least until the occupation phase starts. Bunker attacks, mostly. Cities, if they are left standing at all, are not something you want to spend precious mass invading and holding until you absolutely own low orbit. And maybe not even then.

What I see is aircraft and minisubs, tasked with tracking down and eliminating ASAT weapons. Which means establishing bases (air and sea), securing local resources (fuel, propellant, fortifications), and local command and control (high orbit means lightspeed lag - not conducive to dronerunners). I think there's a place here for nuclear-powered aircraft, portable AA lasers, and space-transportable small navies, as well as autonomous mining/extraction equipment.

But dear god, the mass you'd have to bring across deep space. I suspect the length of these kinds of campaigns would be measured in years, not weeks.

Byron said...

I'm going to try to summarize the results of our discussion up until now. If you disagree, then say so. I'll try to restrain myself to what is definite.
1. There will not be any orbit-to-surface invasions in the conventional sense. Nobody will land troops if they don't control low orbit, and if they control low orbit, they can destroy any forming masses of troops, and provide any landing with overwhelming firepower. If they don't control low orbit by having eliminated all enemy surface to orbit weapons, then the landing forces are sitting ducks during the drop.
2. There will be no conventional land battles with interplanetary conquerors. All conventional land battles will occur on non-homogeneous worlds. These include Balkanized worlds, or worlds with defenses only on one point. Think Battle of Hoth in Star Wars.
3. Powered armor is going to be heavy if it is to withstand normal weapons, limiting its use in infantry replacement. It might be possible for peacekeeping/occupation duties. It will likely be useful for carrying extra weapons, though.
4. Lasers seem unlikely as hand weapons in the near future, but if they exist, they can go through lots of armor.
5. The same applies to coilguns.
6. Drones are iffy because of ECM and the lack of a role. The operator is also vulnerable. If necessary, for story purposes, they can be ruled out.
7. Mecha will not exist. Vehicles will likely be stealthy.
That should be enough for now.

Milo said...

Raymond:

"And even if the defenders' spacecraft are insufficient to intercept and defeat the attackers, I can see a lot of defensive weaponry remaining on the surface"

Oh right. When I said space battle, I was including surface-to-orbit defenses as part of the "space forces". Trading surface-to-orbit and orbit-to-surface shots is very different from staging a surface-to-surface battle.

This goes back to what I said about: "The question is whether eliminating the defender's surface-to-orbit capability would involve deploying hunter-killer troops to the surface while the spaceships wait at a safe distance, or whether it would involve zerging them with spaceships in low orbit and using superior numbers to overwhelm their "fortified position advantage", or a mixture of both.". You seem to be favoring the former, while I tend to think in terms of the latter.


"high orbit means lightspeed lag - not conducive to dronerunners"

Only a second or two. Too much to rely completely on human reflexes (which I didn't think was going to work anyway), but enough for most any drone with even a trace of autonomy.

Of course if you're afraid of getting jammed, that's a different matter.



Byron:

I largely agree with your points. But a couple of notes:


"worlds with defenses only on one point"

Why would you have a world like that (besides dramatic tension)?

The only justification I can think of is if the world is a very recent entrant into the space club, and doesn't yet have the funds for more than one surface-to-orbit installation. If you can have more, any sensible military would spread them out.

Alternatively if only a small portion of the planet has been settled at all. In which case landing troops elsewhere is an iffy proposition because why deploy troops to the middle of nowhere? And if you can afford to deploy troops to the middle of nowhere and march them to the enemy dome-city on land and still be in a condition to fight when they arrive, then said dome city can probably afford to supply a series of defense bases that are far from the main industry center.


"4. Lasers seem unlikely as hand weapons in the near future, but if they exist, they can go through lots of armor."

I'm not entirely sure about this yet. See my question above regarding non-steel armor. We think of steel as tough, but its heat of vaporization isn't that good - if lasers become dominant weapons, armor manufacturers will likely switch to using different materials that are more appropiate.

mithril said...

On the issue of "Army vs. Marines", and the idea of two branches of ground combat units. i think this is a fairly pluasible situation in any setting where interstellar conflict is present. what would seperate the two branches would be combat philosophy. 'marine' units have to be transported on spaceships to get to the planet where the fighting is. and unless your using a space-fantasy setting like star wars, your going to have severe constrants on the mass and volume marine units can use. this would drive the development of marine units into the 'smaller, lighter, tougher' paradigm. marine units owuld likely rely heavily on force multipliers. be they the kind we undertand (man portable artillery, advanced C4I capabilities, and so on) or ones we're just now begining to consider (energy weapons, orbital strikes, power assisted suits, etc.) they'd have to rely on lightweight vehicles, which would be built with plenty of gear to make them more accurate/harder to hit/more flexible (look at the Future Combat Systems vehicles for a good exampe of such "expiditionary" design approaches) tactics for marines would be the kind of decapitating strikes that such light units are excellent for. they can't fight a seige or protracted "fight for every inch" battle, but they can zip in and grab specific strategic targets and cripple the enemies ability to fight back, maybe even force a quick surrender.
while conversely, the 'army' is a defensive force. it has to be larger in number, since it has a larger strategic feild of operation. after all, planets are big. rapid strategic mobility is a feature most would likely include, but since the exact targets invaders will hit can't be completely defined before the first dropships come down, a purely reactive force of marine type units wouldn't be very efficent. so you'd need to be able to put sufficent defensive troops in place at any prospective target. and there might be hundreds of those. since the Army has to have many more units with more people, they wouldn't be able to afford the kind of bleeding edge hardware of the marines. so it is likely they would rely on more 'conventional' gear. the main advantage the army units would have though is that with no need to conserve mass and volume like on a spaceship, they can invest in heavier vehicles, like heavy tanks or self-propelled large bore artillery. these vehicles, while probably not using the same level of accuracy/survivability enhancing electronics as marine units, can afford to carry much heavier weapons and armor. (alternately, they could be 'dumbed down' version of marine vehicles, but in far higher numbers)
thus groud combat would become a case of "unstoppable force vs. immovable object", with marine units being highly mobile but somewhat fragile, against the slower but tougher army units. and of course, both groups would be in an arms race against each other. marines looking for better ways to avoid or defeat army gear, and vice versa.

Raymond said...

Much of this hinges on the nature of a) ASAT weaponry (how well can it be hidden, can it hit higher orbits, can it intercept vehicles during reentry), and b) orbital fire support (time to target, how small and/or mobile a target can it hit, how large must it be to be useful, how low an orbit is required). If ASAT weapons can hit anything in low orbit as long as it stays in low orbit, but not reliably hit craft reentering at the equivalent of lunar or martian reentry velocities, then surface war becomes necessary before anything is decided. If orbital weaponry can hit targets as small as tanks from high orbit, then surface warfare is mop-up and counter-insurgency only, and the real battle is determined in space.

I think ASATs will have a really hard time hitting anything during reentry (ballistic missile defense is currently focused on the boost phase, and not succeeding very well at that), but staying in low orbit is death. Which allows for troops to drop without being annihilated, but not for close ortillery support. I also think the kind of warhead or KKV capable of hitting something from high orbit will be certain death to fixed installations, but not sufficiently accurate to hit a tank, and will take hours to arrange, not minutes.

Which leads to an uneasy snake-and-mongoose between the invaders and defenders, and cat-and-mouse games on the surface.

Byron said...

Maybe I should add a 0 to the conclusions.
0. There will be no land battles for control of planets during a space invasion.
The reason for this is simple. Surface to orbit weapons are deadly. Read Space Warfare I if you doubt this, then come back.
As to the possibility of landing troops, I shouldn't have put in the possibility of landing troops through it. It is probably possible to get a few men on the ground, but not in numbers required to invade a planet.
As to the specific concerns about ASAT capability, I believe that ABM capability is a better model. Even then, there are several reasons that hitting an invasion force is easier than ABM work.
The first is size. This shows the size of modern warheads. A personnel pod is going to be at least three times as large. Even then, when they get down, you have the issues that bedeviled the D-Day paratroop drops. Everyone is on their own, and have to link up. Also, there's no heavy equipment. Anything larger is proportionately easier to hit.
Second, any damage will cause the loss of the craft, as happened with Columbia. All you have to do is blast shrapnel out at the target. High reentry speed makes it worse. Also, if they're launched from high orbit, lasers can be used to damage the heat shields before entry.
Third, the pod has to stop at the end. Ballistic missiles are designed to go as fast as possible through the atmosphere. A dropship can't. It has to stop, which opens it up to normal SAMs.
Fourthy, most of the failures in modern ABMs are not "everything worked, but we missed." They occur because of issues with immature hardware. By this time, those should be worked out.
Also, on any realistic colony world, there's not going to be very many points that are critical. There will only be a few potential landing zones for assault troops, which can then be garrisoned with missiles.
All of the above reasons point to a situation in which the Marines are occupation troops, and the Army doesn't exist at all, unless there's an opponent who shares your planet. To repeat, there will not be land battles for control of planets during a space invasion. I suppose you could set it up so there are, but at that point, it's like deep-space fighters.

Raymond said...

Byron:

I'm really not convinced this is a conclusion, as opposed to a proposal. ABM and SAM systems have at best a few hundred km range. These kinds of weapons also have large launch platforms and radar sites, easily targeted by orbital weapons. Are we really so sure either side will be overwhelming?

Byron said...

There are several mobile version, most notably THAAD. Also, it's hard to miss an incoming object like that. A simple passive IR sensor could see it. Plus, if I can see stuff in LO, then why can't I see a reentering object?
Even if the accuracy is the same as ground-based midcourse defense (50%), then you have to double the forces required, assuming I have the same number of missiles that you have dropcraft. Then when they get down, your forces still have to take the planet. I can easily build an army that can protect myself. Then you have to add more troops. Pretty soon, you're hauling enough stuff across space that it'd be cheaper to just build more bombardment ships.
I do have one request, though. Please stop mixing technology levels. If you can move an army across space, your opponent is likely to have ASAT/BMD/SAM systems quite a bit more effective than those of today. Including, possibly, independent, mobile, mature versions.

Raymond said...
This comment has been removed by the author.
Raymond said...

[Post edited. Probably shouldn't be slinging mud-like stuff. Sorry, Byron, didn't mean to be so harsh.]

I know THAAD is mobile. Its launcher is large. Its radar is less mobile, especially during intercept, and even more vulnerable.

You're still working against gravity, radar jamming, decoys (radar and IR), rocks with heat shields, and if we're talking a substantially higher techlevel, reentry-capable nuclear-powered laser-armed aircraft with plenty of maneuver which can disable and dodge SAMs and ABMs as soon as they get out of the reentry plasma. And then spend the rest of the time hunting your SAM sites. As part of the invasion's initial strike, I've taken out all of your communication satellites and hammered your fixed defense installations. I control high orbit uncontested, and I brought along some nice big telescopes, so I can see where your big fat ABM launchers are, and can pick my landing zone appropriately.

For the sake of argument, lets go with double the SM-3's range - say, 1000 km. You have a large surface area to defend in depth, with interlocking coverage. How many sites do you need to cover the whole surface? How many missiles at each site? How many rocks with heat shields can I make from nearby asteroids or moons to make you waste ABMs before I actually send the drop pods?

Do you really think it will be that easy to stop? If it is, then I never bother sending ground forces. I nuke. Then wait. Then re-terraform. At which point this entire discussion is moot.

Byron said...

I apologize for sounding condescending, but you seem to consider an effective ground-based defense impossible. What's to stop me from shooting your aircraft down before reentry? Have you even read Space Warfare I? It deals with this exact issue. A V2, fitted with the appropriate seeker, can shoot down a spacecraft in low orbit. I don't see why it couldn't also do ABM duties, too. Plus, the ground troops are unnecessary complexity. If you can sneak stuff down into the atmosphere, and take out my ABM/ASAT sites, then you can move your ships into low orbit, and threaten to kill me from there. This spares the expense of using ground troops, which have a number of disadvantages in this situation.
When they land, they still have to be combat-effective. I've read that during the drop at Normandy a lot of the troopers seemed to think that the jump was their mission. "We're here. You should just roll over and die." The same applies here. You have to put men on the ground, then have them accomplish their objectives, while separated from support, because as soon as they've landed, I'm going to move all my mobile ASATs around them, though not too close.
Also, why would a drop pod be any more survivable against defenses than a spacecraft in low orbit? It can't dodge too much, it's on a fixed path, and it can't shoot back. Also, it's far more vulnerable to damage. Dent the heat shield and up it goes.

Raymond said...

I don't consider a ground defense impossible, but it will have a point of saturation. That point will be at the invader's choosing, assuming there is mass anywhere close to the target planet's orbit.

Low orbit is still an orbit, with the predictability that implies. It still goes all the way around, multiplying exponentially the area in which to hide weapons. And it keeps coming around, several times a day, allowing the defender to choose the timing.

With reentry vehicles, you have one opportunity (however many ABMs you use) in a narrow cone of space, and once it gets into the atmosphere it gets unlimited maneuver (unlike orbital craft). And if a lot of stuff is coming down, how do you know which is the drop pod and which is the rock? Or if they're all rocks, just to make you use up ABMs? Just as you said earlier about bullets, countermeasures can and will be attempted.

That's why I think low orbit is harder to hold than to simply go through on your way to the surface - the attacker/defender dynamic is inverted, with all the problems of the strategic case (defense-in-depth, attacker's initiative) and without the benefits of the tactical case (fortifications, advance warning, stockpiled supplies).

Anonymous said...

Ok, here's a scenario for you:
The Dominion of Bob (DoB) controls the largest moon of a Neptune-sized planet (Arnie); they have massive industry, but their agroculture is primative (but adiqute), so they decide to invade the Serine Republic of Gottsfield (SRoG), on the next planet over (Lana), an Earth-like world with several other nations on it (as well as several unclaimed territories)that have an overabudance of farm-goods. DoB has the capacity to ship 80 regiments of about 1200 troopers each. SRoG has an army of about 25 regiments, but a dozen ASAT submarines and another dozen quartets of land-bases ASAT mobile launchers in the mountainous spine of the country. They can defend low orbit over the SRoG, but not the rest of the planet.
So, does DoB land all it's troops in the unclaimed territory closest to the SRoG outside of the ASAT bubble, and attack like a conventional army, or:
Does the army of DoB try a stealth landing of commando teams to take out the ASAT control/sensor centers then drop the rest of the army directly on the SRoG?

The DoB army has handheld weapons that can disable light vehicles (and people), but have no heavy vehicles of their own.They do have large numbers of light-weight power armor, however.
The SRoG army has a small number of tanks, IFV's, and SPA's, as well as combat helicopters and small boats armed with guns, rockets, and torpedoes they use for coastal defense.
So...Can the DoB successfuly invade the SRoG? Can the SRoG repel the invaders? Will the other nations on the planet of Lana react to the war between the DoB and the SRoG?
Go ahead and debate it...

Ferrell

Rick said...

Welcome to yet another new commenter!

I'll make the argument again for space-defense missiles mounted on trucks and dispersed. You can preposition them because you know the regions of the planet you're covering (whether one city and its back country or big swathes of continents), and what orbital tracks an attacker will be on.

From space, a missile truck just looks like another truck. Once it has launched you can pinpoint it, but by then it's nothing but an empty truck, a pretty much worthless target.


An important relevant point here: In Starship Troopers it is implied that ships can come out of FTL already in low orbit. That makes surprise landings credible, because the defender has very little warning time.

If your space tech involves extended travel through normal space to and from planets, there is no way you can show up by surprise, and the defender can watch the targets come in.

A tradeoff to keep in mind.

Byron said...

I guess I should make my position on this whole surface landing thing more clear. I'm not saying it's infeasible, I'm saying it's implausible. While Raymond's scenario could work, it seems to require an inordinate amount of work as opposed to orbital bombardment. If the objective is to conquer the planet without destroying it's civilian infrastructure, then there are two ways to do this:
1. Have your ships engage in duels with the ground defenses, seeking to eliminate them. Land Special Forces teams to help hunt ASAT launchers. Gain control of low orbit, then threaten to bombard key targets unless the planet surrenders. When it does, send ground troops to occupy.
2. Do part of 1, but before the ASATs are all destroyed, throw enough troops at the target to saturate the defenses and fulfill the objectives on the ground.
I obviously consider 1 superior. My big problem with Raymond's plan is that it seems inconsistent. If ASAT/ABM sites are so vulnerable to orbital strikes, why not take them all out that way? How is this nuclear-powered aircraft supposed to slip through? What's to stop the defender from shooting it down with cheaper fighters that don't have to be shipped nearly as far? The same applies to all of the tactics suggested. Landing troops only works if yours can overwhelm theirs, which is going to be tough to do, as I can't see any opposed landing not taking heavy losses from ASATs.
Ferrel, how is this "Stealth landing" supposed to work. We're talking about sending people in pods streaking across the sky at Mach 25. That's sort of hard not to notice, particularly if the defenders suspect something. If you could land the commandos under cover, then have them target the ASATs for orbital strike, I find that the best solution.

Raymond said...

Byron, we're actually closer than you think. I'm not talking landing the whole army, just enough small teams to help out. Maybe a few aircraft and minisubs to hunt the more hidden or dispersed ASAT sites (the kind which can close the bunker doors or drop well below the waves), and enough personnel and weapons to defend a small base for same.

It's the subs I'm most worried about, actually, because while they're not the best for providing coverage against landing craft, they are extremely hard to target from orbit, due to the brief time they spend surfaced, and are great for taking quick potshots at low-orbit craft.

Byron said...

I guess that makes some sense, but the ASAT sites will be hard to kill from the ground. Landing large objects (anything larger than a man) is going to be very difficult. Even men will be tough, but they decoy easier than a minisub. There's a reason that full-sized submarines are more effective than minisubs. They can carry more gear, and are generally better at what they do. Also, modern SSBNs are very hard to track, and I don't think it'd be easier if the hunter was being shot at. If his point of entry is fairly well know, then dispatch all SSNs to the area, and send the SSBNs away. Hunt it down at all costs.
Plus, why couldn't submarines break up landing forces? Just refit an SSBN for the job.
Actually, now that I think about it, the use of mobility vs. camoflage for defense will be governed by the balance of kinetics vs. lasers for orbit-to-surface weapons. If lasers are largely useless for whatever reason, then ships and laser submarines become plausible. They just move out of the way in the half hour it takes for the weapon to hit. If lasers are the primary weapon, then that time disappears, leaving missile subs, launching underwater, as the best weapon.

Raymond said...

One advantage (possibly the only one) is that the landing forces only need to clear out a small area, and they have their landing corridor. It takes a long time to move ASAT weaponry over land or by sea, at which point the landed troops have moved, and another corridor opens up. Bear in mind, too, that in the absence of satellites in low orbit (sitting ducks to invading forces), the attacker can go from launch to landing in a few hours - much less than rotation time - and thus defenses on the other side of the planet are, well, useless.

As for subs, they would either need to be surfaced for tracking, or somehow be fed telemetry from another (more vulnerable) site. Not impossible, but far from ideal (I said "not the best", not "useless").

In the general case of submersible surface-to-orbit weapons, I honestly don't know how best to deal with them. Maybe it's just a matter of having enough big-ass lasers to form interlocking fields of fire around the vast majority (if not the entirety) of the planet, and hope you catch everything. Maybe someone can come up with space-transportable small SSNs quiet enough and well-enough armed to make current SSN designs look like Zepplins compared to F-22s. (Compact liquid-metal fast reactors coupled to magneto-hydrodynamic generators and propulsion? Small supercavitating torpedoes? Any sub heads in here?)

Milo said...

Ferrell:

"They can defend low orbit over the SRoG, but not the rest of the planet."

Actually, their subs can operate from a pretty large stretch of international waters (assuming Lana's oceans are big enough), just as ballistic missile submarines do today.

A line-of-sight weapon will have a maximum range of arccos(planet's radius / (planet's radius + height of low orbit)), or about 10 degrees or 1000 km on Earth. For comparison, this would allow the US to deny landing over a significant portion of Canada - all of the southern half, plus a bit in the northwest which is too close to Alaska. Lasers-based weapons might have shorter ranges if shooting at oblique angles would cause them to lose too much energy to the atmosphere, while missile-based weapons might have longer ranges if they have good homing mechanisms.


"So, does DoB land all it's troops in the unclaimed territory closest to the SRoG outside of the ASAT bubble, and attack like a conventional army, or:
Does the army of DoB try a stealth landing of commando teams to take out the ASAT control/sensor centers then drop the rest of the army directly on the SRoG?"


If there is a "safe" landing place within a reasonable range, then they would probably land there. The safest place would be an unclaimed territory reasonably far (depending on the lasersubs' range) from both the SRoG's territory and the sea. Hopefully the SRoG is on a large continent with significant inland areas. (Inland areas are often less fertile than coastal ones, due to getting less rain, so it quite possible that those areas would go unclaimed until people run out of space elsewhere.)

Also consider what other nations are doing. If there are any other nations on Lana which are willing to lend the service of their spaceports to the DoB, that would greatly simplify the landing. Conversely, if there are other nations which are willing to lend the service of their anti-orbit weaponry to aid the SRoG against the DoB, then that would limit the amount of safe area we can fly over.

If no adequately safe landing place can be found, using the above methods, then I would attempt to take out the anti-orbit installations from orbit (by zerging, if necessary) before attempting a landing. One option would be to bunch up the DoB's ships, zerg locally, punch through a few of the SRoG's defenses to make a small area safe that wasn't previously, land forces there, and then have those forces march out and attempt to take out the SRoG's other defenses. Unfortunately unless the SRoG is very large, the ranges discussed above mean that you would need to clear practically the entire country anyway. I note you didn't mention how many orbital bombardment ships the DoB has...


"Will the other nations on the planet of Lana react to the war between the DoB and the SRoG?"

Well that depends on them, doesn't it?

What is the political situation on Lana? Does the SRoG have any kind of alliance or trade interests with other nations? Are there any lingering tensions? Are there any other nations which might try to take advantage of the situation to get a piece of the pie for themselves in the confusion? Are there any nations who fear that if Lana doesn't stand together against foreign invaders, the DoB might come for them next?

If there are nations that have poor relations with the SRoG but don't want to act openly against them for whatever reason, then they might welcome the DoB's actions and secretly help them in plausibly-deniable ways.

Milo said...

Rick:

"An important relevant point here: In Starship Troopers it is implied that ships can come out of FTL already in low orbit. That makes surprise landings credible, because the defender has very little warning time."

Obviously, almost any military strategy can be justifed using appropiate magitech.



Byron:

"1. Have your ships engage in duels with the ground defenses, seeking to eliminate them. Land Special Forces teams to help hunt ASAT launchers. Gain control of low orbit, then threaten to bombard key targets unless the planet surrenders. When it does, send ground troops to occupy.
...
I obviously consider 1 superior."


So you do think that landing surface hunter-killers to take out anti-orbit defenses is useful?

Whether or not it is useful is a big deal, since these hunter-killers, if viable, are going to see the most exciting surface combat of the war - much better story fodder than an occupation. If they're viable, then the SRoG will also need a home defense army to guard important locations against hunter-killers.

What kind of missions do you see these special forces doing? Do you expect them to actually engage enemies on the ground, or just try to locate defenses and then radio their position for orbital bombardment, skipping the killer part of hunter-killer?


"Plus, why couldn't submarines break up landing forces?"

Because submarines need to surface before firing, making them relatively poor at hitting targets that suddenly deorbit from high orbit and only stay at low-orbit altitude for a short time. If they stay surfaced permanently, they're much more vulnerable and you lose the benefit of using submarines in the first place.


"If lasers are largely useless for whatever reason, then ships and laser submarines become plausible."

If lasers are useless, then I do not expect laser submarines will see much use.


"They just move out of the way in the half hour it takes for the weapon to hit. If lasers are the primary weapon, then that time disappears,"

Not necessarily. A good sub can surface, pop off a shot, and resubmerge in a fairly short timeframe. While sensors can detect it in that timeframe if you happen to be looking in the right direction, you need to recognize what you're seeing in time to fire off a shot. You don't have time for a leisurely scan of the surroundings.

It might even stop just short of surfacing, with its laser barrel poking out of the sea like a periscope. Actually, that could take care of the problem I described above...



Raymond:

"As for subs, they would either need to be surfaced for tracking, or somehow be fed telemetry from another (more vulnerable) site."

I was assuming the telemetry one. Sensors are vastly cheaper than weapon platforms - you can afford to simply have so many of them that the enemy will never shoot them all out.

Byron said...

ASAT weaponry can move at rates better than newly-landed forces could. When I see the forces moving, the stuff sets off. An Ohio-class can do 25 knots. They can't move far enough to really evade unless they have an insane degree of mechanization. That requires bigger drop pods, and so on. I really don't know how to deal with SSONs. (ASAT subs). You just have to wait them out. They have limited deterrence value, and eventually, they can't deny you control. They either launch, and you win or lose, or they don't and the leaders surrender.
One thing we forget in this is that weapons will not just have to target the landing site. The pods should be vulnerable for at least a quarter-orbit before they reach the planet. During that time, any ASAT systems can shoot at them.
Yes, my idea is more of hunters than hunter-killers. I believe that it's going to be far harder to find the launchers than to kill them. If they're hard enough to avoid being killed from orbit, it's going to take an army to destroy them. Thus, there's no point in sending special forces to attack a site that they can just kill from orbit if they find it. To protect against ground attacks, the launchers will be guarded, and it's a lot easier to slip in a dozen men than a hundred.
My comment about lasers being useless was inspired by heat reasons. If they're not very efficient, then it stands to reason that it's far easier for a submarine to use one than it is for a spaceship. Also, haven't we discussed laser barrels before? The mirror will be close to the size of the sub's hull diameter. It's sort of hard to hide that. Actually, I'm not sure lasers subs are good at all. The mirror has to be dry, which means that surfacing is necessary, and the day has to be calm. Missiles are much easier, and can be launched underwater.

Milo said...

Byron:

"The mirror has to be dry,"

So did Age of Sail cannons. They solved the problem with an ingenious technology called "gun ports".

If that doesn't work... lotus effect?

Sabersonic said...

As I was reading the latest posts on the subject of Power Armor, I can't help but be reminded of this one quote by George S. Patton:

"No bastard ever won a war by dying for his country. He won it by making the other poor dumb bastard die for his country. "

That is to say, the purpose of any form of infantry protection, weather they be flak jackets, anti-ballistic weaves, ceramic plates or whatever, isn't to make the soldier invincible. But rather to have the soldier survive and recover fast enough to pursue one of the following options.

A) Get to cover so you can shoot at the guy(s) who attacked you.

B) Help any wounded personnel.

C) Retreat.

As I had stated in my first post in this blog entry, armor isn't the primary reason for the use of PA. The minimum requirement, as many have suggested, for any type of armor protection for the PA would be shrapnel. Especially the stuff that comes down from above.

And even then the armor plates don't have to absorb unholy amounts of lead like rain off a ducks back. They may only need to absorb a few number of rounds for the poor sap to get to cover and recover/fire back and may even be designed to be segmented for easy replacement back at the depot. Such a design could, theoretically, be lighter than the invincible "Tank" armor that many media depict them.

And speaking of tanks, one could argue that PA could be engineered in a similar fashion. That is the thickest sections of armor will be the front, at the expense of protection to the rear. Even have the armor sloped like tanks, though honestly that's just pushing the tank analogy logic a bit too far.

Though to be honest, I had forgotten about the possibility of NBC protection for those who don the PA.

And now for my other two cents.

Surface Warfare Vehicles: I had a second post entry right after my first on this subject, but somehow it no longer exists. But I think by now nearly everyone has said their peace on the subject that is quite similar to my own opinion on the subject.

Anyway, conventional vehicles that exist on a balkanized world will most likely reflect not only the military doctrine of that world's nation-states, but also the environment as well. One could argue that stealth would be the primary design philosophy on many vehicles such as tanks, APCs, IFVs, mobile artillery, aircraft, even surface vessels since even modern weapon systems are powerful enough to take out these complex machinery in a handful of impacts that the best defense (besides offense) is to not be easily seen, if not at all. Stealth may have no place in deep space and requires an existing infrastructure and/or orbital settlements for low planetary orbit, but upon the surface of a planet Stealth has a definitive future even if there are possible countermeasures to find them.

If lasers and other such DEW are the norm, particularly those that could adjust their damage profile dependent upon the target and mission profile at hand, then this greatly simplifies logistics of various vehicles who require line-of-sight firing of targets. Those that require ballistic, over-the-horizon weapons and/or air-burst effects will more then likely stick to solid projectiles for some time to come even if the launch mechanism may change from chemical ignition to electromagnetically launch systems and whatever in between.

There has been recent talk of jet fighters fitted with solid-state lasers which would make the need for stealthy fightercraft that much more essential to their overall survival in addition to new tactics that may focus upon low-altitude and terrain-hugging to enhance said stealth techniques. Arguably the best form of stealth is the magitech cloaking devise, which conversely makes them ideal against laser-based weapon systems that utilize optical frequencies of light.

- Hotmail Address
Gmail Address

Sabersonic said...

Continued from previous post

Air Bases to Orbit-to-Surface Troop deployment: I'm not sure if this is mentioned in any previous blogs, but from what I've been told, air fields are notoriously difficult to defend against. Though this means that it makes the job of "commandeering" an air base that much easier for space-borne commandos and the like (that is if they're able to get past that rather nasty network of anti-orbital and anti-air defenses in the first place) but that also means that they're that equally vulnerable to either Liberation or Liquidation by the previous owners.

I recall some stories about development of "Mobile Offshore Bases" whose primary purpouse is for the deployment of military forces when land bases are unsuitable due to "diplomatic" reasons. These are potiential solutions to the need of an airfield for surface deployment. However, considering that I have yet heard of any new development in this idea, one could assume that it's not an idea worth the pursuit. That and how easily destroyed the modular building blocks of the off-shore airfield would barely be an attractive a solution, let alone any reprisals from surface naval forces or even submarines that would take out said fortification.

- Hotmail Address
Gmail Address

Byron said...

That might work. My point is that setting it up will take some time, and it won't work when it's windy or anything, as the spray will still be on the mirror. It won't build up, but it'll be there.
And saber, how are we to get this base of yours down? When I admitted that people might come down, I didn't expect people to start throwing around ideas involving submarines and airfields. The problem is that as stuff gets bigger, it gets easier to detect. A drop pod might be made stealthy to radar and chilled before entry. It probably won't matter if its plasma trail can be seen in broad daylight. At the very least, the defender will know something has come down, and be hunting it. Plus, it's much harder to make decoys for those types of objects. Hauling all that mass is difficult, and you don't know they won't guess right.

Raymond said...

I'm not sure on the quarter-orbit time. A reentry vector from high orbit would more closely resemble a geostationary transfer orbit than reentry from low orbit - much steeper angle, less reaction time and a smaller cone of defensive fire for SOMs. Someone with actual orbital simulation software (anybody know of good stuff around, preferably open-source?) could tell me if I'm out to lunch?

The troops you land? As attached to powered armor as this thread is, I'm actually thinking something closer to a Tachikoma (from Ghost in the Shell). Legs, wheels, firepower, small size? Sounds better to me.

Also: aircraft? Reentry-capable, obviously. And I'm partial to nuclear-powered designs, myself, as fuel ceases to be a concern. It would obviously need VTOL capability; I'm thinking superconducting lift fans, mass penalty be damned. They'd allow you to design high-flow/low-speed ducts a la the F-35, and leave your main engines optimized for supercruise. I don't think I'd bother with trying to make them capable of orbit, though - absent something like metallic hydrogen, I have no idea how you'd fit them in the performance envelope required to fight, nor how or where to load them with remass.

Against subs, and if we have aircraft available, do we drop a cache of torpedoes and sonar buoys, and if a sub surfaces and pops off a shot, task a fighter to hunt them down?

Hugh said...

A new (?) argument for surface warfare vehicles (from M1 Abrams to M3 Bradley to Stryker) over powered armour: better situational awareness. I believe that most ground attack aircraft and choppers have two crew because that's more effective despite the cost, and a fair number of successful air to air fighters have had two crew as well.

Human beings can consider the tactical environment, or focus on taking out one particular target, but they're not able to do both at once. Even if your powered armour infantry have a tactical data net like in John Scalzi's Old Man's War or David J Williams Autumn Rain trilogy.

(Which isn't to say that powered armour infantry are useless either - I can envisage squads of PA infantry accompanied by AFVs just like current armies have.)

Milo said...

Byron:

"When I admitted that people might come down, I didn't expect people to start throwing around ideas involving submarines and airfields."

Submarines are the hardest kind of anti-orbit defense to detect from orbit. Therefore, (anti-)submarine warfare would be one of the most important roles for surface hunters. You need to be able to get in or near the sea and find out what's lurking there.

Even once found, you probably need specialized weapons to deal with them. Normal orbital bombardment stuff can't penetrate water well.

Thucydides said...

I think the most plausible near future war novel was Ralph Peter's "The War in 2020"

Although rather dated in a political sense (the enemy is Japan, then near the peak of their economic dominance when the book was written), the technology is plausible near term "future" tech.

The story is exciting because it concentrates on the human factor in war. Many of the alleged superweapons being deployed in 2020 end up canceling each other out, so in the end, the most adaptable, most creative and most determined soldiers end up being the one who win (in this case, the Americans, who end up highly outmatched by the Japanese and betrayed by their Russian allies).

Some other points (given the rapid rate of posting here, I am being outmatched!)

Gecko gloves and boots allow the soldier to climb walls, avoiding stairs and other choke points.

Heavy weapons like .50 cal sniper rifles or 20mm cannons have large and bulky ammunition, which limits the amount you can carry. A soldier in powered armour armed with a .50 machine gun might only be in the same position as an unarmoured soldier with an M-4 carbine in terms of how much ammunition he can carry.

Submarines vs spaceships is an interesting trope, but the same water shield which protects the submarine also prevents easy sensor access to space; the sub will need some sort of relay to see its targets in space which of course is the weak point the constellation will try to target.

SLBMs and cruise missile subs (like Soviet era Charlie and Oscar class subs) don't need to surface in order to fire their weapons, but they do need target data and some reliable form of communications to function properly.

Byron said...

In many ways I see orbital assaults as a form of siege. It's a contest to see if the defender runs out of ASATs before the attacker runs out of ships and political/economic capital. I do know that submarines are hard to deal with, but I don't see any easy solutions. It's going to take patience, not dropping a 30 m submarine, and then sending it after them.
The minimum vulnerable distance I can see is still a long ways. During reentry on DGIV missions on Orbiter (Raymond, there's your sim), when headed for Canaveral, I'd start heating over Texas or New Mexico. While you could shorten it, there's a limit on that.

Sabersonic said...

"And saber, how are we to get this base of yours down?" - Byron

One can potentially make the whole base modular and easy to transport. But I wasn't really advocating such a solution. You've already clearly stated how incredibly idiotic such a proposal is in terms of orbit-to-surface warfare. In fact, the only way this could really be achieved is if there's friendly territory that'll play host and allow such a drop in the first place. And even then, why do that when it'll be easier to just have the local bases be on loan?

PA with APCs and IFVs:
From what I can tell, there's a general consensus that the combination of Power Armor with APCs/IFVs are a viable combination as the infantry with APCs/IFVs are now. However, I'm actually kind of surprise that no one else has brought up the following point: Why put all the heavy weapons and armor on the PA, if its that costly mass wise, when the IFV has such defenses and weaponry by default? That's basically what APCs (and arguably IFVs) are: Battlefield Taxis. Meant to transport soldiers to the battlefield protected and fresh enough for a prolonged encounter as opposed to a march or more vulnerable transport vehicles. They have the armor to defend against non-AP rounds wielded by soldiers and the weaponry to keep an enemy PA down for those favored flanking moves. The APC/IFV unit and PA can also be integrated into a network-centric "Battlenet" for lack of a better word.

The only downside is that, for economic, logistic, and mobility sake, the APC/IFV will not have armor capable of withstanding against Tank Killing rounds. And even if there were such armor systems that could defeat conventional Tank Killing rounds, then there'll obviously be countermeasures against such systems, especially if its magitech with loopholes not typically found in traditional space operas.

- Hotmail Address
Gmail Address

Teleros said...

Whew, one hell of a thread here; just going to focus on the more recent comments I think :) ...

"Human beings can consider the tactical environment, or focus on taking out one particular target, but they're not able to do both at once."

How much genetic engineering and such are you willing to allow? It's not really something for this blog I know, but perhaps it's worth considering for another post some time...

"SLBMs and cruise missile subs (like Soviet era Charlie and Oscar class subs) don't need to surface in order to fire their weapons, but they do need target data and some reliable form of communications to function properly."

If said cruise missiles are smart enough to acquire their own targets once they leave the ocean (and perhaps even travel underwater a bit before emerging, for the sub's safety?), then that should certainly increase their effectiveness. Hardly perfect, because there's a lot of atmosphere to travel through whilst the orbiting fleet takes pot shots at the missiles or uses ECM, but it may help.

"Why put all the heavy weapons and armor on the PA, if its that costly mass wise, when the IFV has such defenses and weaponry by default?"

Leaving aside the issue of power supplies, one reason why you're likely to see infantry & APCs (or their robotic equivalents, and space carriers whilst we're at it) is that they provide a handy means of covering a large area. If you're patrolling through the streets of a city, a squad of soldiers with armoured support is usually going to be better than just the armoured support.

Geoffrey S H said...

Arrrgh! I can barely read all of this! That’s alot of detail...

I’ll refer to some points, but there’s so much I might not be able to respond to them directly, and I’m very late post-wise anyway.

As concerning naval warfare, I’m somewhat leaning towards some sort of trimeran-hydrofoil-flying wing ground effect vehicle, with morphable wings (and thus a small ornithopter capability) which are also semi-submersible (500 years from now anyway). They have the coil-weaponry to duel with craft in orbit, and support ASAT subs if need be, and the speed to avoid all but seeker weapons (p.s.: If you want to flush out sibs, fusion nuclear depth charges might be the way ;) ). While I know that transporting a massive fleet to a planet might not be a good use of mass, to have a floating platform for resupply and pitched combat that cannot be shot down, but which can travel almost anywhere where there are seas, (as opposed to a land equivalent that would have terrain as a far more limiting factor) would be a massive asset in my opinion.

As regards large ground battles, landing a large army, then retreating from orbit might in my opinion, not be the most implausible option- take as much as you can, (retreating if necessary- the constellation would blaze in with only defence and pds in mind and extract all it could), and when the constellation comes back in, you can hit the silos that fire on you quicker than when you had just orbital assets- as you have friendly forces covering as much ground as possible, and a greater chance they are near hidden hostile anti- orbital assets. I do believe that stealth is one day going to be completely obsolete as a form of defence though, barring hiding in a city.

As concerning whether orbital cover will decide a conflict6, I remember reading about Saddam’s “supergun” and the capability of the DD(x) 1000 Zumwalt class theoretically having the power to shoot into orbit from rail-guns. Now imagine the development of that technology through a few centuries (we’ll have moved on by then, but I think its slightly possible that it will still be around). Essentially I see many tanks, artillery batteries, and ships being able to duel with spacecraft in orbit- i.e: the best way to remove such forces to land your own, and then thin them out to allow spacecraft to enter low orbit and try to defeat the defending force. Of course this might rely on a balkanised planet so that one nation doesn;’t have access to the entire hemisphere as a defence area, but I think there is a small chance of this happening even if they do.

(continued)

Geoffrey S H said...

(continued)
As concerning whether orbital cover will decide a conflict6, I remember reading about Saddam’s “supergun” and the capability of the DD(x) 1000 Zumwalt class theoretically having the power to shoot into orbit from rail-guns. Now imagine the development of that technology through a few centuries (we’ll have moved on by then, but I think its slightly possible that it will still be around). Essentially I see many tanks, artillery batteries, and ships being able to duel with spacecraft in orbit- i.e: the best way to remove such forces to land your own, and then thin them out to allow spacecraft to enter low orbit and try to defeat the defending force. Of course this might rely on a balkanised planet so that one nation doesn;’t have access to the entire hemisphere as a defence area, but I think there is a small chance of this happening even if they do.

Adding in submarines (and remembering that the battleships and cruisers theorised above and also submerge for a time, or race on the surface out the way of incoming THOR rounds), and I think that surface forces will have a great deal more resilience to spacecraft than is usually thought. If soldiers could be equipped with small shoulder-fired recoilless rifles that expel chem.-pumped nuclear laser weapons to fry the mirrors of space craft then there is the possibility that the best chance of taking a planet is to present a small profile of the massive constellation coming in, then to deploy as many troops in one small area as possible, to maximise pds coverage. Then find cover for them and build some fortifications,. As soon as possible the constellation leaves, and focus on deep-system assets and killsats. The army/navy/airforce then proceeds to close with the defenders (even if the orbit-range of most ground weapons means that they can shoot anywhere, the other side will have pds as well and an atmosphere can do much to reduce accuracy- not least the wind) and attempt to thin them out. Then the spacecraft will be in a position to land reserves and secure the world. I try as much as possible in my settings to take spacecraft out of a land war- saying a battleship can out-duel or at least match a spacecraft of the largest size would help explain this- not to mention a battalion of tanks or an artillery battery.

Geoffrey S H said...

(continued)

The army/navy/airforce then proceeds to close with the defenders (even if the orbit-range of most ground weapons means that they can shoot anywhere, the other side will have pds as well and an atmosphere can do much to reduce accuracy- not least the wind) and attempt to thin them out. Then the spacecraft will be in a position to land reserves and secure the world. I try as much as possible in my settings to take spacecraft out of a land war- saying a battleship can out-duel or at least match a spacecraft of the largest size would help explain this- not to mention a battalion of tanks or an artillery battery.

With one setting, I fiddled around with a NERVA drive and worked out that it would need a 200 engine craft to carry ¼ of a 200,000 ton battleship to a world- not bad given the fact that I have massive hohmann barges in that setting and 800 foot long military space craft. Very difficult to do but not impossible.

While it is VERY soft sci-fi, I do occasionally look to Warhammer 40k for a little inspiration, given the attention they turn to ground combat. Settings that do not assume that space warfare is the be all and end all of combat are few and far between.

Geoffrey S H said...

Apologies for writing so much for a war article- I think I was the one that wrote in asking about ground warfare, did not think it would be THIS long!

Ps: I see tanks that can roll over completely if flipped over, with multiple turrets and small anti-orbit capability eventually being developed- for them, mobility is king. The armour for them would be tough. Aircraft are morphable wing hyper-sonic capable, with pilots in acceleration tanks with Brain control interfaces, working alongside drones (that with the advent of BCI’s might actually be slower to react!) and high tonnage military air-ships. One aircraft can change form a missile like shape (fast transit- strike fighter) to a more delta-winger vehicle (air-superiority fighter and ground-attack).

Pps Someone talked about armouring heat-radiators: perhaps ballenite? I know I always mention that when armour comes up, but it does seem the best bet with radiators...

Milo said...

Byron:

"In many ways I see orbital assaults as a form of siege."

It's certainly possible to hurt enemies by blockading them (remember, if there's something worth trading, then it's also worth blocking), but a planet is still bound to be more self-sufficient than a walled city. I don't think setting up in high orbit and just waiting for the enemy to starve to death is going to work very well.

Furthermore, the longer you wait before attacking, the longer the planetside force has to crank up their military-industrial complex, build new fortifications, manufacture weapons, etc. While a fleet in orbit doesn't have the resources to engage in construction of any kind, except for waiting for reinforcements (which take time and money to ship in). So the longer you wait, the bigger the defender's advantage grows.

So I expect that attackers will opt for "storming the castle" rather than "siege", significantly more often than they did back in medieval times.



Sabersonic:

"One can potentially make the whole base modular and easy to transport. But I wasn't really advocating such a solution. You've already clearly stated how incredibly idiotic such a proposal is in terms of orbit-to-surface warfare."

I think it's viable later on in the occupation stage.



Byron:

"Why put all the heavy weapons and armor on the PA, if its that costly mass wise, when the IFV has such defenses and weaponry by default?"

The PA will carry any weapons that potentially need to be used inside a building, or when a conversation suddenly turns sour. The IFV will carry stuff for use in other situations such as duelling with enemy IFVs, as well as weapons which sometimes need to be carried by PA but aren't relevant to their current mission, and spare ammo for the PAs' weapons (including a portable generator or large battery for lasers).



Teleros:

"If said cruise missiles are smart enough to acquire their own targets once they leave the ocean (and perhaps even travel underwater a bit before emerging, for the sub's safety?), then that should certainly increase their effectiveness."

If your missiles can comfortably travel from water to air to vacuum blind and still kill their target spaceship, then counter-torpedos would probably also be able to comfortably travel from vacuum to air to water blind and still kill their target submarine.

Milo said...

Geoffrey S H:

"(p.s.: If you want to flush out sibs, fusion nuclear depth charges might be the way ;) )"

Well sure. If you exterminate everything in the ocean, that takes care of subs nicely.


"Essentially I see many tanks, artillery batteries, and ships being able to duel with spacecraft in orbit-"

I think anti-orbit capacity is too difficult and expensive to build onto stuff meant for use in surface-to-surface combat, unless it's something that already has a really long range like an ICBM. I doubt you can even fit an anti-orbit weapon on the size of a tank. Furthermore, surface-to-orbit and surface-to-surface are such very different tactical environments that it would be tricky to design a generalist weapon that is efficient both ways. There is also little point to - a spacecraft up in low orbit is just as threatening to you as it is to someone several kilometers from you, and vice versa, so there is no need for each individual surface-to-surface unit to bring its own surface-to-orbit defense with it.

I see surface-to-surface forces and surface-to-orbit forces as being entirely separate branches of the military.


"I think that surface forces will have a great deal more resilience to spacecraft than is usually thought."

What is usually thought, here?

We've already discussed how surface-to-orbit "coastal artillery" would do pretty well against spacecraft. However, surface-to-surface forces are simply sitting ducks for spacecraft - they'll get picked off unless they have some way of shooting back. They can hide, go guerilla, and stick around long enough ot be a thorn for the occupation forces, but they're already essentially on the losing side at that point - air support is a massive US asset in Iraq, and space support will be a massive DoB asset in SRoG.

Raymond said...

Teleros:

I'm actually with Byron on this one - APCs are large, and massive, and make for bigger targets on the way down. I have slightly more faith in an aircraft surviving a drop, since it actually has a chance to maneuver, but an APC is asking for trouble, at least until a large ASAT-free corridor is achieved.

Byron:

I've heard or Orbiter before - now upgraded to must-get. But what's DGIV? And as a test, could you try a reentry from GEO via geostationary transfer orbit, and let me know what the cross-range is?

Tony said...

Luke:

"You seem to have a serious misunderstanding of the way electrochemical batteries work. The chemicals in a redox battery are not in excited states. It is just that some of the chemicals (or, more typically, atoms) like electrons more than others, so the electrons diffuse from one chemical species to another until there is enough voltage built up across the terminals to pull further electrons back (or more precisely - the rate of electrons diffusing along the electrochemical gradient is balanced by the rate of electrons migrating back under the influence of the voltage).

Your description is not a particularly apt description of capacitors, either. There, you just have separated positive and negative charge sitting on their own plates without moving."


I understand all of that. The problem is that you're describing batteries that discharge their energy very slowly. But as I understand things, a battery that discharges energy quickly enough to generate an effective laser pulse would have to be highly charged all of the time. Upset it's balance, and all of those free electrons start doing what free electrons do.

As far as capacitors are concerend, they're simply not practical for small arms. You don't want to be waiting for a capacitor charge when you need to shoot right now.

Geoffrey S H said...

I concede the point Milo abut weapons designed for both S-S and S-O- I would be willing to be surprised though.

Nevertheless, The capability to get to orbit might be miniturised to allow small projectiles fired from massive battleship-grade weapons to reach orbit, the Orion/Ares rocket is much thinner than the Appollo/Saturn V. It may take about 500-600 years for this to be possible, or longer, but IMHO it should be faintly plausible.

Milo said...

Tony:

"As far as capacitors are concerend, they're simply not practical for small arms. You don't want to be waiting for a capacitor charge when you need to shoot right now."

You can keep the capacitor charged at all times (or at least charged whenever the gun's safety is off), much like current guns keep a bullet chambered separately from your magazine. After firing a shot, you immediately start recharging the capacitor in preparation for another shot.

Even if you need to charge the capacitor before a shot, that's no worse than having to cock a gun.



Geoffrey S H:

Just keep in mind that weapons which can shoot into orbit probably also do a pretty good job of shooting very far away on the surface. This would basically be a setting where everybody has ICBMs. So much for conventional land battles.

Turbo10k said...

Bookmarking.

Raymond said...

Uh, where did this inversion of batteries and capacitors come from? Capacitors are electrostatic; their charge/discharge rates are symmetric, and an order of magnitude (or more) greater than electrochemical batteries. The capacitor will be where the energy for any given shot is stored, so it can be discharged fast enough. The rate of fire will be limited by the discharge rate of the battery. Also, electrochemical batteries tend to store less energy when charged quickly than when charged slowly (exact properties vary between battery technologies). Ultracapacitors are generally electrochemical, with energy storage properties of batteries, but with charge-discharge rates approaching electrostatic capacitors.

Milo said...

Raymond:

"Capacitors are electrostatic; their charge/discharge rates are symmetric, and an order of magnitude (or more) greater than electrochemical batteries."

Regardless of how good your capacitor is, it can't charge any faster than the battery it's connected to can feed it.

Raymond said...

Milo:

That's why I said "The rate of fire will be limited by the discharge rate of the battery." The reason you'd have a capacitor at all is the greater discharge rate, which allows several kilojoules to be dumped in a few milliseconds, which translates to specific power on the order of a few megawatts, which is well out of reach for any battery tech you can name.

Geoffrey S H said...

Milo:

Even with infinate range, the further away you are, the more time to get into cover, the closer and the less time till your projectiles strike your targets.

You make good points, and thus I will stop digging myself into a hole.

Nevertheless, the DDX potential does raise the very question of whether surface force in a century or so will acheive infinite range. The capability of US cruisers to shoot down satalites in low orbit brings this point into even greater focus.

Milo said...

Raymond:

"The reason you'd have a capacitor at all is the greater discharge rate,"

Yes. Duh. Did anyone claim otherwise?

I already told Tony how to get around the capacitor's lower charge rate, so it isn't really a big issue, but it's still a concern that you have to actively design around.

Raymond said...

No, capacitors don't have a lower charge rate than batteries. Their charge/discharge rates are symmetric; the higher discharge rate implies a higher charge rate.

Milo said...

*sigh*

Regardless of how good your capacitor is, it can't charge any faster than the battery it's connected to can feed it.

Raymond said...

*sigh*

That's what I'm saying. We're arguing over misunderstandings (again). I was just trying to correct the misapprehension about capacitors being a limiting factor in an energy weapon's power system, instead of a necessary component.

Milo said...

Raymond:

"We're arguing over misunderstandings (again)."

No we're not, we're arguing over misunderstandings! ;)


"I was just trying to correct the misapprehension about capacitors being a limiting factor in an energy weapon's power system, instead of a necessary component."

As far as limiting factors go, the capacitor's maximum energy capacity will be more important than its charge rate. That's what determines how big a shot you can fire off.

Raymond said...

Other way around: you put in as big a capacitor as you require per shot, and design the battery system from there.

Milo said...

Your capacitor and optics determine how big a shot you can fire. From there, your battery determines how many shots you can make before needing to reload, and what your rate of fire is (the latter also depends on cooling mechanisms).

Byron said...

NO!!!!!!
There is no chance of landing large conventional ground forces on a world with intact defenses. Here's the conversation in modern times.
"So how are you going to take out the SAM/AAA site? It's preventing the entire wing from doing it's job."
"We're going to hit it with paratroopers."
"Like a HALO landing?"
"Nope. We're gonna fly a couple of C-130s right over, and land right on it."
The mass ratios argue against it. The defender can, for the same price, outfield you ten to one. Plus, the losses on the way down will be terrible. Geoffrey, I'm sorry, but if the ground troops have bomb-pumped lasers from bazookas, then why don't the orbital attackers use them. Plus, I'm not sure what you heard about the Zumwalt's railguns. It doesn't have any, and even if it did, they probably couldn't do that. I'm not sure of the feasibility of ASAT coilguns at just over Mach 4, to say nothing of Mach 30.
The fusion depth charge idea is good, except that you won't be able to see the submarine unless it fires. Then, it's useless to them anyway, at least mostly.
How is this battleship supposed to land, anyway? There's those pesky defenses that keep getting in the way of your ground battles, which would be so cool!
Sorry, but I was venting.
Milo, I wasn't speaking of starving them of food. Reread the comment. Any known missile factories would be my first targets, followed by the submarine bases. Also, why not just transmit a launch order via ELF, then send a data burst to the missiles as they clear the surface?
Mobility could be a defense, depending on if the attackers use lasers or not. If they do, then you also have to hide. If they don't then planes and ships become viable, as they can leave the area.
DGIV is the Delta Glider IV. It's a winged spaceplane, so I'm not sure how relevant crossrange results would be.
Also, weapons fired from the surface technically can't reach a stable orbit unless they have an apogee motor. They could kill something in orbit, if that's what you mean.

Byron said...

Now to original material:
Submarines have serious limitations as defense weapons. While they are highly useful, they will be unable to stay on station for extremely long periods. 90 days is the limit for current SSBN cruises, and that includes a couple of steel beaches. Wartime will justify higher workloads, but there's a limit on how long people can be kept in a can before going crazy. Also, consumables will be an issue. It can be gotten around, but major maintenance can't. And the sub pins are high on my list of targets. Once they go, it's just a matter of time before the subs break. That's what I meant about siege. Can the attackers stay until the defenders run out of defenses, through use, attrition, and attacks?
Also, another idea. Spotting drones might be useful. If you can slip in a small drone that has better spatial resolution than your starship sensors, then it could serve in the same role as my hunter teams, besides being cheaper.
The nice thing about drones and hunter teams is the uncertainty they produce in the defender. If a missile truck is annihilated by laser or kinetic, then the defender has to wonder if:
A. I need to put more camo netting on them, and hope their starships don't see any more.
B. I need to start hunting special forces.
C. I need to start hunting spotter drones.
D. I need to start hunting spies.
There would be ways to figure it out, but it can't be certain. If, on the other hand, the launcher was wiped out by small arms, you know you need to start hunting special forces.

Raymond said...

HALO landing sounds much closer to what I had in mind. But not unarmored troops - I'm sorry, they're just too damn squishy and slow. At least with power armor you wouldn't be pinned down by the first mortar team in range. (Also, I once knew a guy who was a forward observer for the Canadian Forces in the Kosovo campaign in '93. Taking out SAM sites was something they had to do on occasion.)

Winged spaceplane is in fact what I had in mind to test - I wouldn't necessarily want my drop pods to be entirely ballistic. The particular design, I'm not sure. Maybe spaceplane, maybe just some flaps on a capsule.

Small spotting drones I consider organic to the teams on the ground, and whatever weapons they carry will probably resemble their operators'. Larger, autonomous drones would probably be dropped simultaneously with the hunter teams - same rough size, less concern if they get scragged on the way down.

I think drones would perhaps be most useful for sub hunting. Faster reaction times to sub locations than orbital weaponry, and with the possibility of rearming afterwards for less delta-v. Perhaps the secondary mission of the hunter teams would be to rendezvous at supply caches and rearm/refit drones?

Mangaka2170 said...

A couple points:

Firstly, regarding the battery's charge lag, it seems that we're talking about a single large battery charging the capacitor (thus limiting the time between shots to the charge rate of the battery). Instead, how about using a bank of smaller batteries, all feeding into the one (or two, if a backup is warranted) capacitor? Not only would the multiple points of entry help mitigate the charge rate limitation, but it also adds a level of redundancy to the system as losing a battery or two would still allow the weapon to fire, albiet at a slower rate.

Regarding the submarine endurance limitation, keep in mind that we're also assuming spaceship crews that can stay aboard ships for months (possibly even years, for interstellar travel) without running into these problems. I see no reason why this can't also be applied to submarines.

Raymond said...

Mangaka2170:

Battery packs are already arranged like this internally, because the voltage is limited by the specific processed used. Overall discharge rates and energy density are mostly a function of mass and volume.

The months-long endurance of subs (and certain types of bunkers) is why I think they'll be active and dangerous long after surface defensive SOM sites are found and eliminated. How much that delays the landing of the occupation forces depends on numbers and detection capabilities. How well will passive sensors which can be easily hidden detect and track incoming missiles/drop pods/spaceplanes coming from high orbit? Lower orbits are easy - a human with a sextant and stopwatch can feed targeting data. But if your stationary telescopes are slag, your radar sites are either rubble or hiding, and your remaining weapons platforms are buried or submerged, how much of the surface can you cover, and for how long?

I'm tempted to say that any invasion force should have a second wave arriving perhaps weeks after the first, and loaded with mining/construction craft to use any local moons or asteroids for additional orbital weapons. If a siege will take months and many tons of mass sent downrange, I'd want to get as much of that as possible from resources close by.

Geoffrey S H said...

I'd like to state that the whole fusion chasrge was a light-hearted comment.

Raymond said...

I'd like to point out how much of this discussion of surface warfare depends entirely on the characteristics of space warfare. Which should be a point of reference as we (unfortunately) look ahead to the future of warfare on Earth.

Geoffrey:

Oh, but it does work. If I were sub-hunting a long ways from home, I'd bring as many of them as I can carry. As a bonus, I could repurpose them for surface bombardment if things went appropriately pear-shaped.

Tony said...

Raymond:

"No, capacitors don't have a lower charge rate than batteries. Their charge/discharge rates are symmetric; the higher discharge rate implies a higher charge rate."

A capacitor can be charged as fast as it can be discharged. But the reason you put the capacitor in a pulse power circuit in the first place is that the supply current doesn't provide enough energy over time to do the required work. So you charge the capacitor at a relatively slow rate, then discharge it at a higher (usually much higher) rate.

This becomes a problem in multi-shot tactical weapons. Yes, as suggested, one can keep a capacitor charged for a quick first shot, but then if you need a second shot right away, you have a dilemma. Backup capacitors could of course be installed in the system, but then you start adding mass and bulk to the system, as well as increasing hazards consequent to damamge or failure.

Raymond said...

Tony:

Then you and I are thinking along the same lines, and I misunderstood your earlier comment. My apologies.

I regard the charge time as a simple rate-of-fire problem. Unless there is a fundamental limitation in the firing mechanism of the rail/coil/laser gun, Your sustained rate of fire will be a factor of battery discharge rate and cooling. The question of whether to size the capacitor for a single shot, or multiple, depends on the specifics of the capacitor tech you're using. Is the cap a major mass factor in the weapon? One shot at a time. If not? Make it bigger until it is, and enjoy your long bursts. Perhaps it would lead to a split similar in nature to the rifle/machinegun divide in modern weapons (well, mid-20th century, anyways)?

Milo said...

Byron:

"Milo, I wasn't speaking of starving them of food."

You said siege. The idea of siege is that you block the enemy's traffic and wait while they exhaust important resources without being able to replenish them, whether they be food, helium-3, or whatever. Actual combat in a siege is very low-intensity, mostly to keep people on their toes.

If your battle strategy involves actively targetting and destroying their defenses and factories, then you aren't performing a siege. This isn't something where hanging out in high orbit and waiting will really help you - you need to rush in and hit the enemy, as hard and fast as you can, and the battle will be very intense but has a good chance of being over in hours or days.


"Mobility could be a defense, depending on if the attackers use lasers or not."

Mobility (especially high-agility planes) is a defense at ranges of more than a light-second or so, meaning it'll help your anti-orbit planes from being lasered from high orbit. Granted, that much jinking will put a number on your airframe, and no amount of jinking will protect you from a laser in low orbit.

These planes have less atmosphere between them and space, so they can engage and be engaged by higher orbits than other anti-orbit defenses. Thus winning duels against those high (or at least medium) orbits becomes an important tactical concern.

Of course, you could have your plane hide under a cloud until it's ready to shoot. Concealment isn't just for submarines.


"Also, weapons fired from the surface technically can't reach a stable orbit unless they have an apogee motor. They could kill something in orbit, if that's what you mean."

Yeah. You don't want to waste energy matching speeds with the target's orbit. It's the difference in your speeds that gives you your killing power.

Many people mix up "orbit" and "orbital altitude".


"Submarines have serious limitations as defense weapons. While they are highly useful, they will be unable to stay on station for extremely long periods. 90 days is the limit for current SSBN cruises, and that includes a couple of steel beaches."

Why would you need more than 90 days, unless it's a siege?

Since space itself has no stealth, you know approximately when the enemy will be arriving. You can save up and release your submarines to the far reaches of the planet several days before that happens.

Even if you want to keep most of your submarines deployed continuously, then you can manage that as long as you have brief stops every now and then, which can be done safely during peacetime (not so much during wartime). Current ballistic missile submarines already need to stay out at sea for extended periods.

Anyway, the time limit is largely because of limitations in food supply and the amount of time humans are willing to spend trapped in a tin can and such. The submarines' nuclear reactor is capable of lasting longer. Our current fission reactors can last for a submarine's entire projected service time, nevermind what fusion reactors could do.


"Can the attackers stay until the defenders run out of defenses, through use, attrition, and attacks?"

I expect that unless you actively target and destroy their factories - which is bound to be less profitable at first than targetting and destroying their defenses (after which you will have time to blow up their factories at your leisure) - they can build new defenses faster than they will wear out from natural wear and tear, meaning their defenses will actually increase, not decrease, the longer you wait.

Milo said...

Byron:

"Also, another idea. Spotting drones might be useful. If you can slip in a small drone that has better spatial resolution than your starship sensors, then it could serve in the same role as my hunter teams, besides being cheaper."

Likely. Again, the sea is one of the most important places to deploy such drones. (You can't do sonar from space.)

The surface forces are rapidly becoming less and less glamourous, though... but that's reality for you.



Raymond:

"and loaded with mining/construction craft to use any local moons or asteroids for additional orbital weapons"

Oh right. If you can build a moonbase near the enemy planet, that would greatly simplify your logistics. This is why the presence or absence of moons, and the degree to which they are fortified by factions friendly/unfriendly to one or the other side in a conflict, are an important "terrain" concern.



Tony:

"Yes, as suggested, one can keep a capacitor charged for a quick first shot, but then if you need a second shot right away, you have a dilemma."

Yes, the gun will have a limited rate of fire, but so do today's guns. Today's guns actually have a rate of fire higher than is tactically useful - soldiers have to be taught to take it slow and aim their shots, rather than spray and pray.

You don't need to be able to unload your entire magazine in seconds.


"Perhaps it would lead to a split similar in nature to the rifle/machinegun divide in modern weapons"

The split isn't as large as you suggest. Assault rifles already do both. In lasers, the split would be even smaller - any capacitor that can hold energy for one big shot can also hold energy for a series of smaller shots.

Byron said...

I was using an analogy. There is no equivilant to HALO in this scenario. To get to the surface, you have to go through the guns. Think Charge of the Light Brigade.
Again, why do you need killer teams on the ground? Orbital weapons provide all the firepower needed, and I've already made the point about detection of killer teams. What is the benefit over hunter teams? If that's the case, the drones don't need to be armed.
The 90 day figure for submarines is for current deployments. The difference between them and other ships is air and shore leave. Plus, the Navy keeps two crews and switches them out. I didn't think of the deployment thing, but you just made all of my other points. I'm not sure about sub-hunting drones. There's a lot of ocean out there, and if the balance remains as it is today, then the chances of a patrol aircraft finding an SSBN in the deep ocean is only very fractionally above 0.
Milo, I said like a siege, not an actual siege. In that, I meant that it wouldn't be a quick battle. You start with a blockade and strikes on immobile targets like factories, then insert special forces hunter teams (if they weren't already in place). Occasionally, you venture down into lower orbit, and threaten to bombard something important. When they fire back missiles, shoot them down, and hopefully kill the platforms. Eventually, you'll run out of ships, or they'll run out of missiles. Whoever runs out first, loses. Eventually, they won't have enough weapons to be able to keep you from bombarding important targets, like leadership. At that point, they have to surrender. Then, you tell the important targets not to move, and send in the troops. When the troops are down, you've won.
The siege aspect is that you don't win it in an afternoon, or a week. It's a slow process of chipping away at defenses. Sieges have never been cases of showing up and sitting outside the walls. They usually involve mining, siege engines, maybe a breach in the wall, etc. The point is that they are slow compared to battles.
On capacitors v. batteries: The capacitors are only there to allow greater peak power. If I decide to build a weapon that can fire one shot a second, then the battery needs to be able to put out one shot energy (se)/second. However, I need to release it on microsecond timescales. Thus, the capacitor. One shot/second is a decent fire rate in combat. Please hold the argument. I'm just trying to put out more clearly what the capacitor does.

Milo said...

Byron:

"Milo, I said like a siege, not an actual siege. In that, I meant that it wouldn't be a quick battle."

And I disagree with that.

What you suggest sounds like performing the Charge of the Light Brigade to get close to an artillery emplacement, having your surviving cavalry shoot the artillery unit's cook, and then falling back and doing the same thing later. Once I'm in range, I'd rather shoot the artillery.


"If I decide to build a weapon that can fire one shot a second, then the battery needs to be able to put out one shot energy (se)/second. However, I need to release it on microsecond timescales. Thus, the capacitor. One shot/second is a decent fire rate in combat."

Correct.

For reference, probable energies for a single shot from a hand weapon are likely to be in the kilojoule range. So use that to estimate how much your battery and capacitor need to be capable of.

Tony said...

Milo:

"Yes, the gun will have a limited rate of fire, but so do today's guns. Today's guns actually have a rate of fire higher than is tactically useful - soldiers have to be taught to take it slow and aim their shots, rather than spray and pray.
You don't need to be able to unload your entire magazine in seconds."


Ummm...not quite. There are many situations in which automatic fire is not advisable. But when you need full automatic fire, you really need it. Also, if your ability to buy pre-packaged technology is much greater than your ability to train skilled riflemen, automatic fire is a valuable combat multiplier. (Think about the effect that the AK-47 has had on warfare in the last 50 years.)

"The split isn't as large as you suggest. Assault rifles already do both. In lasers, the split would be even smaller - any capacitor that can hold energy for one big shot can also hold energy for a series of smaller shots."

I think Raymond made the comment that this is in response to, but I have to agree with him. Regardless of overlapping capabilities, small arms are optimized for certain uses, as you yourself pointed out with your comments on slow, aimed fire. That's the default method (or at least should be) with a service rifle, and it's optimized for it, even if it has a full auto fire capability. On the other hand, a machine gun could be fired at a very low rate, even single shots, but what would be the point?

Byron said...

What you suggest sounds like performing the Charge of the Light Brigade to get close to an artillery emplacement, having your surviving cavalry shoot the artillery unit's cook, and then falling back and doing the same thing later. Once I'm in range, I'd rather shoot the artillery.
No. Factories will be immobile, and their locations will be known before the war. The first weapons launched, from high orbit, will be at them. The reason to venture into low orbit is to gain better targeting data and shorter response times. While you're there, you try to take out the enemy's remaining defenses, which are mobile, and can't be seen from farther out. The objective of the campaign is to remove the defender's ability to deny your ability to freely operate in low orbit. You seem to be suggesting doing it all at once. I'm not sure that's possible, particularly when we bring SSONs into the picture. The problem is that they have to deny you freedom to operate, which means expending missiles. Eventually, they'll run out.

Luke said...

Byron

The various armor-piercing weapons described here seem to make armored vehicles obsolete.

There is also the possibility of better armor. Perhaps using some form of nanostructured carbon or boron nitride for extreme strength and extreme thermal resistance, plus the low density means less mass (although greater thickness) as defense against hypervelocity penetrators.

Luke, how much energy, both blaster and heat ray, would it take to reliably incapacitate a man? That's what I think that small arms will use.

I tend to look for 20 cm to 30 cm of penetration through flesh - that's about what is considered the minimum for police pistols. Lasers seem to have issues with drilling holes with aspect ratios of more than 20:1 to 30:1, so this indicates you need about a 1 cm hole. For heat rays, this gives a 40 kJ pulse with a 1 cm diameter for 20 cm of penetration. For blasters, a 0.5 kJ pulse train would do the trick, according to my calculations - this seems to be the equivalent of a sidearm, where several kJ would be more like a longarm. Higher overall energies may be used because a higher energy per pulse means each pulse carves out a larger crater in armor, and so you don't have to focus as tightly to get optimum penetration, which means you can use the laser at longer range.

Milo said...

So what are you doing during your siege?

For there to be a point to waiting, you have to either be expecting something to happen on its own (like the enemy starving to death), or be in the process of doing something that inherently takes a long time (like digging a tunnel).

It's not clear to me what would make the enemy's defenses easier to kill if you take your time doing it.

If your idea is to simply fight until your enemies run out of ammo (not likely to work on laser-based defenses, and pretty bloody even on missile-based ones), then you can also do that in one intense battle with said ammo being launched at a much higher rate, with the bonus that you'll take out some enemy defenses before they expend all their ammo.

Raymond said...

Byron, despite our disagreements on some of the details, that was exactly the scenario I had in mind: surface warfare entirely as a continuation of space warfare, and with the sole objective of cementing control of low orbit. Which, to me, makes for more exciting settings for stories or video games - you (and maybe a few friendlies) against the world (literally), concentrating on specific, focused objectives, always on the run, always outnumbered.

Raymond said...
This comment has been removed by the author.
Raymond said...

[Slight edit - damn Blogger.]
It's about the hide-and-seek game, and waiting until, say, your enemy subs have to surface for supplies or new crew. Even after the initial blitzkrieg by space forces to establish control of higher orbits, destroy the planet's communication and sensor platforms in orbit, and destroy all the weapons and factories you can find on the surface, some will escape (either underground or underwater) and bide their time. These forces can make low orbit difficult to occupy, and control of low orbit is what you need to threaten targets at will, land occupation troops, and otherwise force the enemy to surrender wholesale.

Byron said...

That means that blasters are more likely to see use, right? Just from a battery point of view.
Milo, I though up Charge of the Light Brigade analogies to each of our plans:
Situation: Attacking dug-in enemy artillery (1850s).
Raymond: Have cavalry charge guns, then attack with sabers.
Milo: Bring own artillery, and launch all-out attack from range.
Byron: Cut off enemy supplies, and then harass them until they run out of ammo.
Pleas don't over-analyze these. It's just my best understanding of how they work.

Milo, as to the siege, it's that I'm not sure that a rush will manage to take out all of the defenses. I suppose you could force them all to fire, but even then, they'll likely extract a heavy butcher's bill. It gives you time to find the launchers, and starve out the subs. Raymond made this point excellently.
Raymond, I'm not sure about this. While it makes a good story, I'm approaching this not as an author but as a strategist. I can understand that you want the option of special forces for stories, but is it really the best way? That's what I'm looking for.

Milo said...

Luke:

"There is also the possibility of better armor. Perhaps using some form of nanostructured carbon or boron nitride for extreme strength and extreme thermal resistance, plus the low density means less mass (although greater thickness) as defense against hypervelocity penetrators."

So how well would your blaster do against these materials (or boron carbide)?

According to Byron, power armor would top out at about 500 pounds, which according to my estimates gives you about 4.5 cm of boron carbide.



Raymond:

"It's about the hide-and-seek game, and waiting until, say, your enemy subs have to surface for supplies or new crew."

When they do, wouldn't you want to be in a position to whack-a-mole them as they pop up? If you're in high orbit, they'll be able to finish their business and resubmerge before you can do anything.


"some will escape (either underground or underwater) and bide their time"

If they're still biding their time while you're in low orbit, then they're not stopping you from landing troops. If they stop biding their time when you get into low orbit, then congratulations, you got them to show themselves!

Byron said...

And then, when you're landing troops, they pop out and kill them, and keep you busy enough for whatever you were threatening to get away. You'll have to kill enough to make that impractical.

Raymond said...

Byron:

Actually, once a hole in the lines could be found or made, a flank attack by cavalry on dug-in artillery positions was often the best tactic. Dependent on terrain, of course.

As a strategist, I'd much rather avoid the taking of planets by force from interplanetary distances in the first place. Sabotage, guerrilla tactics, insurgency, fomenting revolutions, espionage, and infiltration are much better bets. But if you have to, I'd like to have at least a few people on the ground, perhaps in case my siege must lift prematurely, and those boots can begin the above methods of making trouble. Plus, y'know, Romance is what this blog is about, innit?

Byron said...

Over-analyzing!!!!!!!
There is no way to make a hole, short of one of the above methods. Oh, and there's no flank, either. It's a circle. Please don't take it too far.
And while I agree with you on the preferred method of taking a world, I don't see the point of these killer teams. What you need are Green Berets in that case, and they're too valuable to waste thrown against launcher defenses when they can call in an orbital strike. I agree with you otherwise.

Milo said...

Byron:

"Milo, I though up Charge of the Light Brigade analogies to each of our plans:
Situation: Attacking dug-in enemy artillery (1850s).
Milo: Bring own artillery, and launch all-out attack from range."


Sounds accurate enough.

My own artillery are at a disadvantage because they're not dug-in, but I aim to overcome that with superior numbers - that's just the cost of attacking a fortified position.


"Byron: Cut off enemy supplies, and then harass them until they run out of ammo."

The thing is that I don't see why you can make them run out of ammo more efficiently if you attack them slowly, then if you attack them quickly. In the former case the number of your troops they can kill will be constrained by how much ammo they have, while in the latter case they'll be constrained both by how much ammo they have and by their rate of fire.

Cutting off enemy supplies is useful if the battle will take a long time, but first you need to justify the battle taking a long time. In a short battle, the enemy won't be receiving supplies simply by virtue of not having the time to make or ship any.

I guess your approach might make sense if the enemy has significantly better rate of fire than you do (perhaps because of the difficulties of dissipating waste heat in space) - specifically, if your rate of fire is slow enough that you have time to move out into high orbit, wait a while, and move back into low orbit between shots.


And I'll let Raymond answer the analogy directed at his plan.

Raymond said...

Bait: taken.

At the beginning of this wonderfully entertaining thread-jacking I seem to have pulled off, I was mostly curious what exactly the force composition would look like. How many soldiers, how many drones, how many aircraft, would or could you insert naval forces of any type? I'm not terribly tied to any one solution. I'd think Green Berets using the LLRP model from Vietnam would be par for the course. Then again, you might need someone to storm the more hardened bunkers. Open question for me, though.

Flanks, though, do exist. Between the curvature of the planet (not an insignificant concern, given that defensive orbital or even high-altitude sensors are most likely gone) and the distribution of defensive forces and civilian population, in theory anything can be flanked. Spherical surface, after all.

Byron said...

Milo:
The problem is that you can't get everything, and what you don't get will keep you busy. I'm not going to try to force them to surrender until I'm fairly certain that I can take them in a final fight. They'll have to harass me to keep me from threatening them. If I say "surrender" they have to launch, or not have a vital target in my sights. The objective is to get them to the point at which they have no capability to stand me off like that.
In the analogy, I'm trying to take the position by skirmishing with it, and digging bastions towards it. Eventually, they can't challenge my next bastion for want of ammo. Your method would work, but it'd take more ships than my method.
Also, the ships in orbit will have to rotate fairly often. It's going to be exhausting when you could have to go to General Quarters on a few moments notice. The ships can't stay too long. Plus, they'll need to do things like extend radiators and empty heat sinks. It's not so much rate of fire as duration of continuous fire.

Raymond said...

LRRP, not LLRP. Sorry.

Byron said...

Raymond:
As I recall, LRRP stands for Long Range Reconnaissance Patrol. That's sort of what I proposed. Checking the Wikipedia article, it mentioned that they were used in combat, but it also says:
The tactical employment of LRRPs was later evaluated to be generally used far too dangerously by strategic commanders...
That's pretty much what I'm suggesting about your teams. They could do it, but it's easier to just call down artillery than to do it by hand.

Luke said...

Milo

Okay, so what kind of transparent armor does protect against these millisecond pulses? And why does the armor work when air doesn't?

It's tough to say. I could imagine something like a hard transparent plastic, where the beam chars the plastic and makes it black so it has to burn through rather than shine through. Unfortunately, you would need pretty thick plastic to provide much protection from a beam that could hole a man.

You might make something out of diamond layered with an easily vaporized plastic. When the plastic vaporizes, it produces enough pressure to shatter the diamond. The shattered diamond acts as a diffuser, scattering the beam. When the scattered light reaches a low enough intensity, it just chars the plastic layer without burning through, and the beam stops.

I am somewhat skeptical of transparent armor - but something like this would be better than nothing if it was thick enough.

We seem to be mostly talking about lasers vs steel armor, but steel is actually pretty poor material for using to armor against lasers. What about boron carbide (in use as body armor today) or carbon nanostuff?

Steel, RHA (Rolled Homogeneous Armor) in particular, is the standard against which armor piercing rounds are measured. So it makes sense to look at how well steel will protect against lasers and hypervelocity penetrators so we can see how much better those are than modern armor piercing projectiles.

Steel really is a wonder material. But it was the wonder material of the 20th century. You are quite right that there are better armor materials out there. Boron nitride (either toughened cubic BN or BN nanotube weaves) and nanostructured carbon (either toughened diamond or fullerite/nanotube weave) look very promising as the ultimate in what a homogeneous armor material can do.

Against some sort of diamond or cubic BN toughened by nanostructuring, you would need to be closer by a factor of about two in order for your blaster-style laser to efficiently drill it (meaning that the pulse lands within the crater - the crater in something as strong as diamond or cubic BN will be about half as wide as the crater in RHA). If you are within the optimum range, you will drill only about half as far through diamond as you would through RHA. Since diamond is half as dense as steel, you can get equivalent protection with about one quarter the weight.

Black diamond (perhaps what they call DLC - Diamond Like Carbon - an amorphous form of carbon with short ranged diamond-like bonding) would work extremely well as armor against a heat ray style laser. Since carbon does not melt under any reasonable temperature, you don't get the melt ejection which allows heat rays to perform so well against most armor materials. Instead, you need to sublimate all of the diamond in the way. The same heat ray that can burn through 3 cm of steel will only burn through 0.3 cm of diamond. Because diamond is half as dense, you would get twenty times the protection for a given mass.

Against hypervelocity penetrators, for the same protection diamond will be 1.4 times as thick, but only 70% of the mass, as RHA. Toughened diamond would put any modern armor to shame against ball or armor piercing rifle rounds.

Fullerite (which is sort of like sintered carbon buckyballs, except that you can't actually sinter them) or woven nanotubes, would not be quite as strong as our hypothetical tough diamond, but would be even less dense. You wouldn't get quite as good of protection against blasters, but would get better protection against hypervelocity penetrators and equivalent protection against heat rays.

Byron said...

That's an interesting analysis, but wouldn't blasters be the laser weapon of choice, because of the lower energy requirements for a given penetration? For that matter, might the heat ray be significantly overpowered, and have another wounding mechanism? Thermal shock or something like that? If you're vaporizing that hole, might the steam do something? I just find it hard to believe that there's an 80-fold difference in power for a given effectiveness.

Milo said...

Byron:

"In the analogy, I'm trying to take the position by skirmishing with it,"

Skirmishing is generally about hitting light targets, then scooting before heavy targets show up. I'm not sure what you "light targets" here are and how you plan to stay out of reach of the "heavy targets" for the full duration it takes to fight them - at any time, you're either in low orbit or you're not. There's, as you yourself just said regarding flanking, little room to engage some targets of your choice while staying safely away from others.


"and digging bastions towards it."

I don't see what this analogy is being applied to, either.



Luke:

"Steel, RHA (Rolled Homogeneous Armor) in particular, is the standard against which armor piercing rounds are measured."

And that makes sense, because steel is a pretty good armor against current munitions, so it's what you'll be struggling to penetrate. If lasers become commonplace, however, armor technologies will change. You wouldn't measure lasers against present-day ballistic armor any more than you measure present-day rifles against cuir boilli.


"Steel really is a wonder material."

Yes. It's awesome and cheap at lots of stuff. Laser defense doesn't seem to be one of them.


"If you are within the optimum range, you will drill only about half as far through diamond as you would through RHA. Since diamond is half as dense as steel, you can get equivalent protection with about one quarter the weight."

Hmm. You said a PIKL blaster would penetrate about 4.5 to 9 cm of steel. With your new numbers, that reduces to 2.25 to 4.5 cm of diamond. According to my calculations and Byron's recommendations for armor weight, you can afford to wear up to 3.24 cm or so of diamond armor.

It seems your ability to penetrate the armor or not will depend on some rather fine assumptions in technology. It's not a foregone conclusion either way.

Raymond said...

I can see blasters used as primarily short-ranged weapons, with the added advantage for spaceborne usage of easy logistics, additional safety inside spacecraft and habs (different focal ranges between target and hull will mean less accidental penetration), and the ability to carve through bulkheads or blast doors at short range without additional special equipment. Can't honestly seem them in general use planetside, though - too short a range. Except for point-defense, of course.

Byron:

IIRC, the main job of LRRPs was in fact finding targets for artillery and/or aerial bombardment, and not direct combat. It was when strategists started treating them like paratroopers that their usage became too dangerous.

Byron said...

The analogy is breaking down. By skirmishing, I was referring to attacks where I was trying to wear down the enemy slowly, with minimum risk to myself. The bastion is my ability to successfully threaten a vital target. IE, eventually, they can't stop me from destroying what I want to, because they don't have enough missiles. Also, aren't most launch platforms going to be single-shot? Trucks in particular. There's no point in putting two missiles on a truck, if the missiles cost a lot more, and once the truck launches, it's dead. You could mount multiple missiles, and launch them all at once, but that means the truck is still empty.
BTW, Luke's energy numbers are for weapons with the equivalent damage of pistols. I can get a battlesuit that barely protects me against pistols.
Just to be sure, that's not 500 lb of armor. I would guess the armor is limited to maybe 150 lb, if that. 100 is probably a better number, if you want to carry a lot of stuff.
Raymond, that's what I meant the whole time.

Luke said...

Milo

If you have ideas for other kinds of futuristic weapons besides lasers and differently-propelled projetile weapons (like coilguns), I'm also listening. Particle beams don't strike me as too useful, though - they tend to leave too many victims alive but terminally ill. (Heavier ions would tend to do only surface penetration and so not have this issue... except the same lack of penetration also makes them incapable of penetrating air.)

The more I look into them, the more I am swinging toward electron particle beams making sense in an atmosphere. I did some simulations of how much radiation they will spit off in air, and it is not too bad for the operator - certainly within OSHA guidelines for radiation workers. People near the beampath but not actually hit by the beam don't fare quite as well (much of the radiation is forward-directed) but it is not in the realm of acute radiation sickness. Still, the risk to a soldier from the electron beams of other people in his squad might be unacceptable. Anyone hit by an electron beam will be in bad shape from acute radiation poisoning (as well as from the hole the beam vaporizes through their body).

Electron beams perform extremely well against armor if they can punch through a person. A beam that can pass through the 20 g/cm^2 of a person's body can (to first approximation) pass through 20 g/cm^2 of anything else. That's about 2.6 cm of steel, or 5.7 cm of diamond. It is also about 170 meters of air, so you will need to have beams capable of serious over-penetration of a person at close range to have them perform well at longer ranges.

The biggest problem I see with electron beams is that they have funky interactions between the beam current, the beam magnetic field, and the beam plasma that makes the beam unstable. There may be ways around this, but electron beams will probably have inherent inaccuracies so they will never be as accurate as lasers. In the worst case scenario, these beam instabilities cannot be overcome and the beams will veer off in random directions, zig-zagging across the battlefield.

Protons and ions can have significant penetration at higher energies - beam voltages of 100 MeV or so. However, because they also interact via the strong nuclear force, they tend to have unpleasant reactions with atoms in the air, producing showers of radiation. No matter what beam voltage they run at, even their radiation showers will be stopped by about 5 to 8 km of sea-level-pressure air. The initial beam will be able to pass through at most about a km of air before being shattered into radiation showers.

As for the ever-popular-regardless-of-whether-it-makes-sense plasma weapons... well. The appeal here is that plasma is really hot and so carries lots of energy, which makes it rather intuitive that it would be painful to get hit by. (If you're merely using plasma to obtain ions for a particle beam weapon, that doesn't really count as a plasma weapon.) The question is what are you shooting at that requires so much heat to melt? Existing termite grenades can already melt steel. Plasma seems like it would be overkill against most things you're likely to ever encounter on a battlefield, and it's hard to work with.

Plasma weapons seem to be unphysical - you can't confine plasma from within, and if you shoot out plasma with enough energy to go boom, it will go boom as soon as it leaves the barrel of your gun. The exception is beams of relativistic plasma used in space warfare - essentially a particle beam of protons neutralized with electrons to avoid space-charge effects.

Milo said...

Luke:

"The biggest problem I see with electron beams is that they have funky interactions between the beam current, the beam magnetic field, and the beam plasma that makes the beam unstable. There may be ways around this,"

Electrolasers. However, those aren't properly particle beam weapons.


"In the worst case scenario, these beam instabilities cannot be overcome and the beams will veer off in random directions, zig-zagging across the battlefield."

Homing weapons are good. Homing weapons that pick a random target to home in on rather than the thing you were aiming for are bad.

Random zig-zagging is how natural lightning behaves, so I figure it's a good first approximation for how artificial electron weapons will behave as well.


"if you shoot out plasma with enough energy to go boom, it will go boom as soon as it leaves the barrel of your gun"

The plasma wouldn't be expected to go boom, but rather to melt/vaporize/overheat targets with thermal transfer.

Still improbable, I know.


"The exception is beams of relativistic plasma used in space warfare - essentially a particle beam of protons neutralized with electrons to avoid space-charge effects."

I'm not counting that as a real plasma weapon, just a plasma-based particle beam weapon.

Raymond said...

If you have ideas for other kinds of futuristic weapons besides lasers and differently-propelled projetile weapons (like coilguns), I'm also listening.

I think guided personal-scale missile weaponry may have uses - think OICW rounds with rockets and seekers. Wouldn't necessarily require a heavy launcher, can engage power armor, small vehicles, and airborne drones, and can be used as indirect fire weaponry. Not very mass-efficient, so I don't think it'd be a primary weapon, but if available it would allow better standoff engagement of a greater range of forces.

Luke said...

Byron

The mirror has to be dry

I've thought about this a fair bit. Milo already mentioned superhydrophobic surfaces. Another possibility is to make your window out of cubic boron nitride. Your first laser pulse then vaporizes any water and other contaminants but the CBN is tough enough and refractory enough to stand up to that. You could combine the two with the right surface nanostructure of your CBN - get superhydrophobic CBN.

Anonymous said...

Byron:
"Ferrel, how is this "Stealth landing" supposed to work. We're talking about sending people in pods streaking across the sky at Mach 25. That's sort of hard not to notice, particularly if the defenders suspect something. If you could land the commandos under cover, then have them target the ASATs for orbital strike, I find that the best solution."
'Stealth' in this case, means 'misdirection'...either wait for a conveniant metor shower or provide your own via a load of scrap and missiles; in both caes the commando's capsules splash down just off shore or land in remote areas so they won't be targeted. The commando's then have to make their way to their objectives without being detected until they attack...again, 'stealth' is involved. Sorry for not being clear earlier.

Ferrell

Luke said...

Tony

A capacitor can be charged as fast as it can be discharged. But the reason you put the capacitor in a pulse power circuit in the first place is that the supply current doesn't provide enough energy over time to do the required work. So you charge the capacitor at a relatively slow rate, then discharge it at a higher (usually much higher) rate.

This becomes a problem in multi-shot tactical weapons. Yes, as suggested, one can keep a capacitor charged for a quick first shot, but then if you need a second shot right away, you have a dilemma.


The design I envision is to have a capacitor with enough energy for one shot. This is charged by a battery. A modern Li:ion battery can get something like a third of a kW per kg of specific power. So if we are limited to modern battery technology, and if our laser requires 1 kJ per shot, then a 10 kg battery pack would provide enough power for three shots per second. You have to be a pretty good shot to be able to accurately aim more than three shots per second.

In practice, battery technology is advancing rapidly, what with the current focus on electric vehicles. I've seen lots of proposals for new substrates and new electrolytes that would allow Li:ion batteries to charge and discharge faster. There may also be better options out there for rapid discharge batteries.

Raymond said...

Luke:

My money's on silicon nanowire, personally, and that should be capable of a couple MJ/kg and at least 1 kW/kg (probably more). The capacitor sizing, however, could be part of the difference between small arms and support weapons. Assuming the capacitors required for a single shot aren't already a major part of the system mass.

Luke said...

Byron

That's an interesting analysis, but wouldn't blasters be the laser weapon of choice, because of the lower energy requirements for a given penetration?

Oh absolutely. The big question is whether you are technologically capable of producing a blaster. If your pulsed laser technology isn't up to producing multiple joule pulses, you're stuck with heat rays.

For that matter, might the heat ray be significantly overpowered, and have another wounding mechanism? Thermal shock or something like that? If you're vaporizing that hole, might the steam do something? I just find it hard to believe that there's an 80-fold difference in power for a given effectiveness.

The pressure from the steam is likely to expand the hole in meat, but meat is very elastic and can expand quite a bit. I am unsure if you will get expansion to the point that the meat will tear. You are unlikely to get tendon to tear, or bone to shatter, from the steam produced. You will still need to vaporize through something like 20 g/cm^2, and that's going to take about 50 kJ/cm^2.

We have quite a bit of experience with drilling steel with high power lasers, and for high powered continuous beams (as opposed to nanosecond or shorter pulses) melting and melt ejection via a hypersonic vapor jet is the primary means of removing material.

The issue here is that there are very different energy scales for mechanical damage (cracking, tearing, shattering) and thermal damage (melting, vaporizing). Mechanical damage takes about an order of magnitude or two more energy.

Tony said...

Luke:

"The design I envision is to have a capacitor with enough energy for one shot. This is charged by a battery. A modern Li:ion battery can get something like a third of a kW per kg of specific power. So if we are limited to modern battery technology, and if our laser requires 1 kJ per shot, then a 10 kg battery pack would provide enough power for three shots per second. You have to be a pretty good shot to be able to accurately aim more than three shots per second."

With a modern service rifle, I can get 600 rounds into ten kilos worth of ammo plus magazines, and I can send them downrange three times as fast. (Not all at once, of course, but in bursts when I need to fill up the target area with projectiles.) Even with the best batteries, using the most efficient of foreseen technologies, I can't simply can't imagine doing any better with a laser rifle, either in rapidity of fire or quantity of shots. Why accept the complexity of a laser when a chemical energy rifle will do?

Albert said...

So, there are two lines of tought worth thinking about:
-attacking from long range and blasting away defenses (milo)
-setting up a show, aim for supply line and then take your time to kill them off. (byron)

Both of you guys are right, but your thoughts depend from assumptions about who you are fighting.

I find obvious that if you want to do the first against a planet with Earth's population and industrial capability, you need a few Star Destroyers to pull it off correctly.

Milo's option has some sense when you are battling a colony with limited resources (probably the majority of the times then), so that your fleet isn't PWNED instantly by buckets of KKVs any time it gets in low orbit.

The second option is really the only way for anyone foolish enugh to try capturing a homeworld-class planet (like Earth).
Your constellation is going to be greatly outmatched by ground defences. Period.
So you camp in high orbit and act smart when you can.


And the part about landing drop pods is not different (Raymond). Landing drop pods on homeworld-class planets (Earth) would be plain idiotic. They knew that you were coming months before your arrival, and you cannot hope to saturate the defences to let people slip through unless you have the abovementioned Star Destroyers covering them.

In (more common) warfare against colonies that have more limited defences, you may be able to slug it out enough to slip people through and do their work with acceptable losses.
(being mostly go find the darn C&C centers and call fire support on them)


And now some thoughts about travel times...
Hell, we are talking of months for enemies coming from Mars or Venus, but of years or more if from Jupiter or the outer system.

You enemy will be very bunkered up by the time you arrive.

You expect to find missile fabs with a big red arrow painted on them? Wrong. They will have moved the fab in more safe places. Or even split it up in hundreds of tiny dispersed and redundant laboratories and just carry around relatively small pieces that are then assembled in the launcher (an unsuspicious truck trailer)

You expect subs to have supply problems or crew problems? Wrong. They had plenty of time to set up decent makeshift undersea bases (or hidden places) where they can dock to resupply and maybe (if they are truly cool) even refit / do mainteneance where a dock is required.

-Albert

Albert said...

Also, PR generally won't let you shoot on civilians (a reason why I prefer Evil Empires), so the better the enemy disguises itself as one, the more fun your work will be.

And any time you do wrong and nuke a bus full of children, Interplanetary Press will make sure that anyone in the system knows it.

Also, don't forget the whole megacorp stuff. To kill fabs you need to hit corporations also present in your own nation that may not be happy and will do their best to avoid this by political pressure on your command.

-Albert

Milo said...

Wow. We're up to 200 posts already?


Albert:

"The second option is really the only way for anyone foolish enugh to try capturing a homeworld-class planet (like Earth)."

Trying to conquer something that powerful is going to be very bloody no matter how you do it. I suggest bringing a really, really large fleet. We're talking a space armada here. Battleships filling the skies.

If you don't have those, then give it up and apologize profusely to the governments of Earth for wasting their time.


"Also, PR generally won't let you shoot on civilians (a reason why I prefer Evil Empires), so the better the enemy disguises itself as one, the more fun your work will be."

PR also generally won't let you disguise troops as civilians or use civilian shields.

If you get caught.

Luke said...

Tony

With a modern service rifle, I can get 600 rounds into ten kilos worth of ammo plus magazines, and I can send them downrange three times as fast.

The best modern Li:ion batteries can store nearly a MJ/kg. At one kJ per shot, a 10 kg battery pack would give you 10,000 shots.

Even with the best batteries, using the most efficient of foreseen technologies, I can't simply can't imagine doing any better with a laser rifle, either in rapidity of fire or quantity of shots.

I can easily foresee an improvement of specific energy or specific power of batteries by an order of magnitude or more (maybe you will have a choice of one or the other, maybe you can do both at once. It is hard to say). A one order of magnitude increase in specific power would let us get off 30 shots per second and 10,000 shots from a 10 kg pack.

And of course you can go beyond electrochemical batteries. My favorite sci-fi super battery is a toroidal room temperature superconductive inductor supported by a carbon nanotube backing. The physical limit on this is the ability of the nanotubes to withstand the magnetic "pressure" of the field-current self-interaction. This lets you get somewhere in the 40 MJ/kg to 50 MJ/kg range. You would probably want to engineer in a safety factor of a factor of two or so, but this gets you an energy source with a specific energy of around 20 MJ/kg, a specific power rivaling that of capacitors, an extremely rugged casing, and a fairly fault-tolerant system in the sense that its armor can absorb more energy than would be released by a catastrophic failure.

Why accept the complexity of a laser when a chemical energy rifle will do?

Perhaps because a laser can do it better? Perhaps because it will not end up being all that complex? A phase-locked laser diode block might be able to be mass produced cheaply, it could be rugged and robust. Combine with an adaptive optics/active focus/autostabilizing flexible mirror using cheap piezo-electrics and a fixed cubic boron-nitride lens, and you could have a rugged service longarm.

There is no way of knowing how technology will pan out. For all we know, laser weapons will never be more than very large vehicle or fixed installation based anti-missile and artillery pieces. Or maybe they will end up being better than projectile weapons in terms of lethality, cost, armor piercing, ruggedness, ammunition capacity, rate of fire, and accuracy.

Luke said...
This comment has been removed by the author.
Luke said...

Byron

Submarines have serious limitations as defense weapons. While they are highly useful, they will be unable to stay on station for extremely long periods. 90 days is the limit for current SSBN cruises, and that includes a couple of steel beaches. Wartime will justify higher workloads, but there's a limit on how long people can be kept in a can before going crazy. Also, consumables will be an issue. It can be gotten around, but major maintenance can't.

Wouldn't the same limitations apply to the spacecraft of the attacking forces? This would imply that unless they can rotate out their space crews, they will need to take the planet within 90 days.

«Oldest ‹Older   1 – 200 of 828   Newer› Newest»